You are on page 1of 436

F3 Course notes

Chapter 1
An Introduction to Financial Accounting – Its
Scope and Purpose

1.1 ACCA SYLLABUS GUIDE OUTCOME 1:


Define financial reporting – recording, analysing and summarising financial
data

Financial reporting is a way or recording, analyzing and summarizing financial data.

Transactions are recorded in books of prime entry. The totals of these books of
prime entry are posted to the ledger accounts. Finally, transactions are summarized
in the financial statements. We will be study in detail these procedures later on in
Chapters, 4, 5 and 6.

1.2 ACCA SYLLABUS GUIDE OUTCOME 2:


Identify and define types of business entity – sole trader, partnership, limited
liability company

Businesses exist to make a profit. There are three main types of business entity:

1. Sole Traders

Sole traders are people who work for themselves. Examples include a
hairdresser, the local stationer, a plumber.

A sole trader has unlimited liability, i.e. if the business runs up debts that it
is unable to pay, the proprietor will become personally liable for the unpaid
debts and would be required, if necessary, to sell his private possessions to
repay them. For example, if a sole trader has some capital in his business,
but the business now owes $50,000 which it cannot repay, the trader might
have to sell his house to raise the money to pay off his business debts.

2. Partnerships

Partnerships occur when two or more people decide to run a business


together. Examples include an accountancy practice, a legal practice and a
medical practice.

In general, the partners have unlimited liability although there may be


circumstances when one or more partners have limited liability.

___________________________________________________________________________

2
3. Limited Liability Companies

Limited liability companies are incorporated to take advantage of ‘limited


liability’ for their owners (shareholders).This means that the maximum
amount that an owner stands to lose in the event that the company
becomes insolvent and cannot pay off his debts, is his share of the capital
in the business.

In all cases, we apply the separate entity concept, i.e. the business is regarded as
being separate from the owner (or owners) and the accounts are prepared for the
business itself.

1.3 ACCA SYLLABUS GUIDE OUTCOME 3:


Recognise the legal differences between a sole trader, partnership and a
limited liability company

In law, sole traders and partnerships are not separate entities from their owners. A
partnership ceases and a new one starts whenever a partner joins or leaves the
partnership.

A limited liability company has a separate legal identity from its shareholders. In
fact, it can issue contracts in the company’s name. It continues to exist regardless of
the identity of its owners.

Lecture Example 1

Which of the following are differences between sole traders and limited liability
companies?
1. A sole traders’ financial statements are private; a company’s financial
statements are sent to shareholders and may be publicly filed
2. Only companies have capital invested into the business
3. A sole trader is fully and personally liable for any losses that the
business might make; a company’s shareholders are not personally
liable for any losses that the company might make

A. 1 and 2 only
B. 2 and 3 only
C. 1 and 3 only
D. 1, 2 and 3

___________________________________________________________________________

3
1.4 ACCA SYLLABUS GUIDE OUTCOME 4:
Identify the advantages and disadvantages of operating as a limited liability
company, sole trader or partnership

Table 1 : Advantages and Disadvantages of a Limited Company

Advantages Disadvantages
 Profits have to be shared
 Limited Liability
out amongst a potentially
 More capital can be raised
larger number of people
as no limit on number of
 Detailed legal procedures
shareholders
must be followed to set up
 Control of company can
the business – consuming
not be lost to outsiders –
time and money
shares only sold if all
 Financial statements have
shareholders agree
to comply with legal and
 The business will continue
accounting requirements
even if one of the owners
 Financial information can
dies, shares being
be inspected by any
transferred to another
member of the public once
owner – separate legal
filed with the Registrar,
identity
including competitors

Table 2 : Advantages and Disadvantages of the Sole Trader

Advantages Disadvantages
 Personal satisfaction  Limited sources of finance
 Secrecy  Restricted growth
 Personal Control  Full personal responsibility
 Enjoyment of all profits for the decisions and due
 Absence of legal to unlimited liability the
formalities when  debts of the business
establishing business
 Financial advantages in
terms of low taxes, longer
period to pay taxes and
lower accountancy fees.

___________________________________________________________________________

4
Table 3 : Advantages and Disadvantages of a Partnership

Advantages Disadvantages
 There are no legal  Partners are jointly and
formalities to complete severely liable for the acts
when setting up the and omissions of the other
business partners
 Each partner can  Profits have to be shared
specialize amongst more owners
 Partners can share the  Partners may disagree
workload  The size of a partnership
 Financial advantages in is limited to a maximum of
terms of low taxes, longer 20 partners, however
period to pay taxes and there are exceptions to
lower accountancy fees. this general rule
 Any decision made by one
partner on behalf of the
company is legally binding
on all other partners
 Partnerships are
unincorporated, resulting
in unlimited liability for the
partners, making them
personally liable for the
debts of the firm.

1.5 ACCA SYLLABUS GUIDE OUTCOME 5:


Understand the nature, principles and scope of financial reporting

Financial accounting is mainly a method of reporting the results and financial position
of a business. It is not primarily concerned with providing information towards the
more efficient running of the business. In fact, financial accounting provides historical
(past) information.

Management need to plan for the future. They require detailed information as they
are responsible to plan and control the resources of the business. Management (or
cost) accounting analyses data to provide information as a basis for managerial
action.

___________________________________________________________________________

5
1.6 ACCA SYLLABUS GUIDE OUTCOME 6:
Identify the users of financial statements and state and differentiate between
their information needs

Why do businesses need to prepare and produce financial information?


A business should produce information about its activities because there are various
groups of people who want or need to know that information.

The “Framework for the Preparation and Presentation of Financial Statements”


states that, “the objective of financial statements is to provide information about the
financial position, performance and changes in financial position of an entity that is
useful to a wide range of users in making economic decisions.”

Lecture Example 2

What are the needs of these different users and how would they use the financial
information?

a. Managers of the company:

b. Investors/shareholders:

c. Trade suppliers:

d. Trade customers:

e. Providers of finance:

f. Government and its agencies:

g. Employees of the company:

___________________________________________________________________________

6
h. The public:

1.7 ACCA SYLLABUS GUIDE OUTCOME 7:


Explain what is meant by governance specifically in the context of the
preparation of financial statements
Describe the duties and responsibilities of directors and other parties covering
the preparation of financial statements

Corporate governance was defined by the Cadbury Committee 1 as: 'The system by
which an organisation is directed and controlled, at its most senior levels, in order to
achieve its objectives and meet the necessary standards of accountability and
probity.'

Corporate governance is not solely about introducing systems of control, it is


fundamentally linked to directing the organisation in order to achieve objectives. This
is critical to the success of the organisation and is a central part of the role of the
board of directors.

1.7.1 Board of Directors

The most prominent group of actors in corporate governance are the company’s
directors. They can be either executive or non-executive directors (NEDs).

The UK Companies Act sets out seven statutory duties of directors. Directors should:
1. Act within their powers
2. Promote the success of the company
3. Exercise independent judgement
4. Exercise reasonable skill, care and diligence
5. Avoid conflicts of interest
6. Not accept benefits from third parties
7. Declare an interest in a proposed transaction or arrangement.

When exercising this duty, directors should consider: -

1. The consequences of decisions in the long term


2. The interests of their employees
3. The need to develop good relationships with customers and suppliers
4. The impact of the company on the local community and the environment
5. The desirability of maintaining high standards of business conduct and good
reputation
6. The need to act fairly between all members of the company

1The Cadbury Report, titled Financial Aspects of Corporate Governance, is a report of a committee
chaired by Adrian Cadbury that sets out recommendations on the arrangement of company boards
and accounting systems to mitigate corporate governance risks and failures.
___________________________________________________________________________

7
Directors’ Responsibility for the Financial Statements

The directors are responsible for preparing the annual financial statements in
accordance with applicable law and regulations. Company law requires the directors
to prepare financial statements for each financial year and such financial statements
must give a true and fair view. Hence, the directors are required to:
 select suitable accounting policies and then apply them consistently;
 make judgments and estimates that are reasonable and prudent; and
 state whether they have been prepared in accordance with IFRSs.

Directors are responsible for the internal controls necessary to enable the
preparation of financial statements that are free from material misstatement, whether
due to error or fraud. They are also responsible for the prevention and detection of
fraud.

Financial statements of companies are usually audited. An audit is an independent


examination of the accounts to ensure that they comply with legal requirements and
accounting standards. The findings of the audit are reported to the shareholders.

Lecture Example 3

A company's ____________________ is (are) potentially the most effective


instrument of good corporate governance.

A. Shareholders
B. Board of directors
C. Top executive officers

Lecture Example 4

The primary responsibility for the preparation of financial statements in accordance


with IFRSs rests with the ______________________________.

A. Shareholders
B. Board of directors
C. Top executive officers

___________________________________________________________________________

8
__________________________

KEY POINTS
_________________

___________________________________________________________________________

9
1. What is financial reporting?
Financial reporting is a way or recording, analyzing and summarizing financial
data.

2. Define: -
a. Sole traders – Sole traders are people who work for themselves. They
have unlimited liability.

b. Partnerships – Partnerships occur when two or more people decide to


run a business together. In general, the partners have unlimited
liability.

c. Limited liability companies - Limited liability companies are


incorporated to take advantage of ‘limited liability’ for their owners. The
maximum amount that an owner stands to lose in the event that the
company becomes insolvent and cannot pay off his debts, is his share
of the capital in the business.

3. Legal differences between a sole trader, partnership and a company

In law, sole traders and partnerships are not separate entities from their owners.
On the other hand, a limited liability company has a separate legal identity from
its shareholders.

4. A limited liability company has many advantages compared to a sole trader and
a partnership. It has limited liability and finds it easier to raise finance. It has a
separate legal identity from its shareholders. Hence, a company continues to
exist regardless of the identity of its owners.

5. Financial Accounting vs. Management Accounting: -


Financial accounting is a method of reporting the results and financial position
of a business. It provides historical (past) information
Management accounting analyses data to provide information to management
as a basis for managerial action.

6. Information needs of the different users: -

The objective of financial statements is to provide information about the financial


position, performance and changes in financial position of an entity that is useful
to a wide range of users in making economic decisions.

There are various groups of people who need information about the activities of
a business. These include investors, suppliers and customers, providers of
finance, the employees of the company, government and the general public

___________________________________________________________________________

10
7. Corporate Governance

Corporate governance is fundamentally linked to directing the organisation in


order to achieve objectives.

Board of Directors: -

The directors are responsible for preparing the annual financial statements in
accordance with applicable law and regulations. Company law requires the
directors to prepare financial statements for each financial year and such
financial statements must give a true and fair view.

___________________________________________________________________________

11
_______________________________

QUESTION BANK
____________________

___________________________________________________________________________

12
1. Financial reporting is the name given to the actual transactions carried out by a
business.

A. True
B. False

2. Which of the following information is particularly useful to shareholders?

A. Bank statements
B. Financial statements for the past five years
C. Tax records for the past five years
D. Budgets for the coming financial year

3. Which of the following information is particularly useful to managers?

A. Bank statements
B. Financial statements for the past five years
C. Tax records for the past five years
D. Budgets for the coming financial year

4. The main objective of accounting is to:

A. provide useful information to users


B. record, categorise and summarise financial transactions
C. calculate the taxation due to the government
D. calculate the amount of dividend to pay to shareholders

5. The IASB Framework identifies user groups. Which of the following is not an
information need for the ‘Investor’ group?

A. assessment of repayment ability of an entity


B. measuring performance, risk and return
C. taking decisions regarding holding investments
D. taking buy/sell decisions

___________________________________________________________________________

13
__________________________

ANSWER BANK
_________________

___________________________________________________________________________

14
1. B – Financial reporting records, analyses and summarizes financial data.

2. B

3. D

4. A

5. A – This is an information need for the bank and suppliers.

___________________________________________________________________________

15
Chapter 2
The Regulatory Framework

2.1 ACCA SYLLABUS GUIDE OUTCOMES 1 and 2:


Understand the role of the regulatory system including the roles of the IFRS
Foundation (IFRSF), the International Accounting Standards Board (IASB),
the IFRS Advisory Council (IFRS AC) and the IFRS Interpretations Committee
(IFRS IC)
Understand the role of International Financial Reporting Standards

2.1.1 Introduction

Limited liability companies are required by law to prepare and publish financial
statements annually. The form and content of these accounts are primarily regulated
by national legislation. They must also comply with International Accounting
Standards (IASs) and International Financial Reporting Standards (IFRSs).

2.1.2 Accounting Standards

International Accounting Standards were issued by the IASC from 1973 to 2000.
They provide guidance as to how items should be shown in a set of financial
statements both in terms of their monetary value and any other disclosures. They are
a single set of high quality, understandable and enforceable global standards.

The IASB replaced the IASC in 2001. Since then, the IASB has amended some IASs
and has proposed to amend others, has replaced some IASs with new International
Financial Reporting Standards, and has adopted or proposed certain new IFRSs on
topics for which there was no previous IAS.

Accounting standards were developed for two main reasons: -


a. To reduce subjectivity
b. To achieve comparability between different organisations

Financial statements may not be described as complying with IFRSs unless they
comply with all of the requirements of each applicable standard and each applicable
interpretation.

2.1.3 The IFRS Foundation (IFRSF)

___________________________________________________________________________

16
The IFRS Foundation is an independent organisation having two main bodies, the
Trustees and the International Accounting Standards Board (IASB), as well as the
IFRS Advisory Council (IFRS AC) and the IFRS Interpretations Committee (IFRS
IC).

The IFRSF is governed by a board of 22 trustees. These trustees appoint the


members of the IASB, IFRS IC and the IFRS AC. They also review annually the
strategy of the IFRSF and the IASB and its effectiveness, including consideration,
but not determination, of the IASB's agenda. These trustees also raise the funds
necessary to support the IFRSF.

2.1.4 The International Accounting Standards Board (IASB)

The International Accounting Standards Board (IASB) is an independent, privately-


funded accounting standard-setter based in London, UK. There are 14 Board
members, each with one vote.

The IASB is committed to developing, in the public interest, a single set of high
quality, understandable and enforceable global accounting standards that require
transparent and comparable information in general purpose financial statements. In
addition, the IASB co-operates with national accounting standard-setters to achieve
convergence in accounting standards around the world.

How are standards developed?

International Financial Reporting Standards (IFRSs) are developed through an


international consultation process, the "due process” that involves interested
individuals and organisations from around the world.
The due process comprises six stages:

1. Setting the agenda;


2. Planning the project, including forming a 'working group' to advise the IASB and
its staff on the project;
3. Developing and publishing the discussion paper for public comment;
4. Developing and publishing the exposure draft for public comment. This exposure
draft must be approved by vote of at least nine IASB members;
5. Developing and publishing the standard. The standard must be approved by
vote of at least nine IASB members and finally
6. The standard is issued

Lecture Example 1

What are the objectives of the IASB?

A To enforce IFRSs
___________________________________________________________________________

17
B To issue IFRSs

2.1.5 The IFRS Advisory Council (IFRS AC)

The IFRS Advisory Council (IFRS AC) gives advice to the IASB on a range of issues
which includes, but is not limited to, the following:

1. input on the IASB’s agenda;


2. input on the IASB’s project timetable (work programme) including project
priorities, and consultation on any changes in agenda and priorities; and
3. advise on projects, with particular emphasis on practical application and
implementation issues, including matters relating to existing standards that
may warrant consideration by the IFRS Interpretations Committee (IFRS IC)

The IFRS AC also supports the IASB in the promotion and adoption of IFRSs
throughout the world. This may include the publishing of articles supportive of IFRSs
and addressing public meetings on the same subject.

2.1.6 The IFRS Interpretations Committee (IFRS IC)

The IASB’s interpretative body, was reconstituted in December 2001 as the


International Financial Reporting Interpretations Committee (IFRIC).

The IFRIC reviews, on a timely basis within the context of current International
Financial Reporting Standards (IFRSs) and the IASB Framework, accounting issues
that are likely to receive divergent or unacceptable treatment in the absence of
authoritative guidance, with a view to reaching consensus on the appropriate
accounting treatment.

In developing interpretations, the IFRS IC works closely with similar national


committees.

The IFRS IC addresses issues of reasonably widespread importance, not issues that
are of concern to only a small minority of entities. The interpretations cover both:

 newly identified financial reporting issues not specifically dealt with in IFRSs;
or
 issues where unsatisfactory or conflicting interpretations have developed, or
seem likely to develop in the absence of authoritative guidance, with a view to
reaching a consensus on the appropriate treatment.

___________________________________________________________________________

18
Lecture Example 2

Which of the following provides advice to the International Accounting


Standards Board (IASB) as well as informing the IASB of the implications of
proposed standards for users and preparers of financial statements?

A The IFRS Advisory Council

B The IFRS Interpretations Committee

Lecture Example 3

What is the role of the IFRS Interpretations Committee?

A To create a set of global accounting standards

B To issue guidance on the application of International Financial


Reporting Standards

___________________________________________________________________________

19
__________________________

KEY NOTES
_________________

___________________________________________________________________________

20
1. The Regulatory System

Accounting standards provide guidance as to how items should be shown in a set


of financial statements both in terms of their monetary value and any other
disclosures.

They were developed for two main reasons: -


a. To reduce subjectivity
b. To achieve comparability between different organisations

2. The Structure

Extracted from: http://www.ifrs.org/The-organisation/Pages/How-we-are-


structured.aspx

3. The IFRS Foundation (IFRSF)

Its roles: -
___________________________________________________________________________

21
1. The trustees appoint the members of the IASB, IFRSIC and the IFRS AC.
2. They also review annually the strategy of the IFRSF and the IASB and its
effectiveness, including consideration, but not determination, of the IASB's
agenda.
3. These trustees also raise the funds necessary to support the IFRSF

4. The International Accounting Standards Board (IASB)

Its roles: -

1. The IASB aims to develop a single set of high quality, understandable and
enforceable global accounting standards that require transparent and
comparable information in general purpose financial statements.
2. It co-operates with national accounting standard-setters to achieve
convergence in accounting standards around the world.

5. The IFRS Advisory Council (IFRS AC)

Its roles: -

1. input on the IASB’s agenda;


2. input on the IASB’s project timetable (work programme); and
3. advise on projects, including matters relating to existing standards that may
warrant consideration by IFRIC

6. The IFRS Interpretations Committee (IFRS IC)

Its roles: -

1. provides guidance on both how to apply existing IFRSs and how to account
for new financial reporting issues where no IFRS exists.

___________________________________________________________________________

22
__________________

QUESTION BANK
___________________

___________________________________________________________________________

23
1. International Financial Reporting Standards are set by which body?

A. IASB
B. IASCF

2. The IASB has the power to enforce compliance with IASs/ IFRSs. Is this
statement?

A. True
B. False

3. Who issues International Accounting Standards?

A. The auditing practices board


B. The stock exchange
C. The IASB
D. The government

4. The role of the IASB is to?

A. Oversee the standard setting and regulatory process


B. Formulate accounting standards
C. Review defective accounts
D. Control the accountancy profession

___________________________________________________________________________

24
______________________

ANSWER BANK
______________

___________________________________________________________________________

25
1. A

2. B

3. C

4. B

___________________________________________________________________________

26
Chapter 3
The Qualitative Characteristics of Financial
Information

3.1 ACCA SYLLABUS GUIDE OUTCOME 1:


Define, understand and apply qualitative characteristics:
i. Relevance
ii. Faithful representation
iii. Comparability
iv. Verificability
v. Timeliness
vi. Understandability

The IASB’s Conceptual Framework for Financial Reporting describes the basic
concepts by which financial statements are prepared. The main purpose of the
Framework is to:

i. assist in the development of future IFRS and the review of existing standards
by setting out the underlying concepts

ii. promote harmonisation of accounting regulation and standards by reducing


the number of permitted alternative accounting treatments

iii. assist the preparers of financial statements in the application of IFRS, which
would include dealing with accounting transactions for which there is not (yet)
an accounting standard.

3.1.1 Qualitative Characteristics of Financial Information

The revised Framework distinguishes between two types of qualitative


characteristics that are necessary to provide useful financial information: -

1. Fundamental qualitative characteristics (relevance and faithful representation)


and
2. enhancing qualitative characteristics (comparability (including consistency),
timeliness, verifiability and understandability).

___________________________________________________________________________

27
3.1.1.1 Fundamental Qualitative Characteristics

For information to be useful, it must be both relevant and faithfully represented.

1. Relevance

Influences economic decisions of user

Relevant financial information is capable of making a difference in the decisions


made by users.

Has predictive value and/or confirmatory value or both

Relevant information assists in the predictive ability of financial statements. That is


not to say the financial statements should be predictive in the sense of forecasts, but
that (past) information should be presented in a manner that assists users to assess
an entity’s ability to take advantage of opportunities and react to adverse situations.

Materiality

Materiality is a threshold or cut-off point for information whose omission or


misstatement could influence the economic decisions of users taken on the basis of
the financial statements. This depends on the size of the item or error judged in the
particular circumstances of its omission or misstatement. Hence, materiality is not a
matter to be considered by standard-setters but by preparers and their auditors.

2. Faithful Representation

General purpose financial reports represent economic phenomena in words and


numbers. To be useful, financial information must not only be relevant, it must also
represent faithfully the phenomena it purports to represent. Financial statements will
generally show a fair presentation when:

 They conform with accounting standards


 They conform with the any relevant legal requirements
 They have applied the qualitative characteristics from the Framework.

Financial information that faithfully represents economic phenomena has three


characteristics: -

 it is complete
 it is neutral
 it is free from error
.

___________________________________________________________________________

28
True and fair override

IAS 1 states that an entity whose financial statements comply with IFRSs should
disclose that fact.

However in extremely rare circumstances management may conclude that


compliance with an IFRS or interpretation would be misleading.

In this case an entity should depart from the requirement of the standard provided
the relevant regulatory framework permits such departure.

3.1.1.2 Enhancing Qualitative Characteristics

Comparability, verifiability, timeliness and understandability are directed to enhance


both relevant and faithfully represented financial information. Those characteristics
should be maximised both individually and in combination.

1. Comparability

Users can identify similarities and differences

Comparability is fundamental to assessing the performance of an entity by using its


financial statements. Assessing the performance of an entity over time (trend
analysis) requires that the financial statements used have been prepared on a
comparable (consistent) basis.

Consistent application of methods

Comparability is enhanced by the use and disclosure of consistent accounting


policies. Users can confirm that comparative information for calculating trends is
comparable. The disclosure of accounting policies at least informs users if different
entities use different policies.

Comparability should be distinguished from consistency (the consistent use of


accounting methods). It is recognised that there are situations where it is necessary
to adopt new accounting policies (usually through new Standards) if they enhance
relevance and reliability. Consistency and comparability require the existence and
disclosure of accounting policies.

2. Verifiability

Financial information is verifiable when it enables knowledgeable and independent


observers to reach a consensus on whether a particular depiction of an event or
transaction is a faithful representation.

3. Timeliness
___________________________________________________________________________

29
Timeliness means that information is available to decision-makers in time to be
capable of influencing their decisions.

4. Understandability

Understandability is enhanced when the information is:

1. classified
2. characterised
3. presented clearly and concisely

However, relevant information should not be excluded solely because it may be too
complex and cannot be made easy to understand. To exclude such information
would make financial reports incomplete and potentially misleading. Financial reports
are prepared for users who have a reasonable knowledge of business and economic
activities and who review and analyse the information with diligence.

The Cost Constraint on Useful Financial Reporting

Cost is a pervasive constraint to financial reporting. Reporting such information


imposes costs and those costs should be justified by the benefits of reporting that
information. The IASB assesses costs and benefits in relation to financial reporting
generally, and not solely in relation to individual reporting entities. The IASB will
consider whether different sizes of entities and other factors justify different reporting
requirements in certain situations.

___________________________________________________________________________

30
3.2 ACCA SYLLABUS GUIDE OUTCOME 2:
Define, understand and apply accounting concepts:
i. Materiality (in Section 3.1)
ii. Substance over form
iii. Going concern
iv. Business entity concept
v. Accruals
vi. Fair presentation
vii. Consistency (in Section 3.1)

3.2.1 Underlying Assumptions

The Framework sets out two concepts which can be presumed when reading
financial statements:

• Accrual Basis

The effects of transactions and other events are recognised when they occur,
rather than when cash or its equivalent is received or paid, and they are
reported in the financial statements of the periods to which they relate.

• Going Concern

The financial statements presume that an enterprise will continue in operation


in the foreseeable future or, if that presumption is not valid, disclosure and a
different basis of reporting are required.

3.2.2 Other Accounting Concepts

1. The business entity concept (separate entity)

In accounting, a business should always be treated separately from its owner(s).

2. Fair presentation

The financial statements must "present fairly" the financial position, financial
performance and cash flows of an entity.

Fair presentation requires the faithful representation of the effects of transactions,


other events, and conditions in accordance with the definitions and recognition
criteria for assets, liabilities, income and expenses set out in the Framework.

IAS 1, “Presentation of Financial Statements” states that: -

___________________________________________________________________________

31
1. compliance with IFRSs should be disclosed
2. All relevant IFRS must be followed if compliance with IFRSs is disclosed

In some rare circumstances, management may decide that compliance with a


requirement of an IFRS would be misleading. Departure from the IFRS is therefore
required to achieve a fair presentation.

IAS 1 states that for a fair presentation, the following is required: -

1. selection and application of accounting policies


2. presentation of information in a manner which provides relevant, reliable,
comparable and understandable information
3. additional disclosures when required

3. Substance over form

Transactions need to be accounted for and presented in accordance with their


substance and economic reality even if their legal form is different.

Lecture Example 1

Sales revenue should be recognised when goods and services have been supplied;
costs are incurred when goods and services have been received.
The accounting concept which governs the above is the

A. business entity concept


B. materiality concept
C. accruals concept

Lecture Example 2

The IASB’s Conceptual Framework for Financial Reporting identifies characteristics


which make financial information faithfully represent what it purports to represent.

Which of the following are examples of those characteristics?

(1) Accruals
(2) Completeness
(3) Going concern
(4) Neutrality

A. 1 and 2
B. 2 and 4
C. 2 and 3
D. 1 and 4
___________________________________________________________________________

32
3.2.3 Historical Cost

Historical cost has been defined as the amount paid or fair value of the consideration
given.

Advantages of Historical Cost Accounting

1. The cost is known and can be proved (e.g. against an invoice). It is therefore
objective
2. It enhances comparability
3. It leads to stable pricing – using current market values would lead to volatility
in asset values

Disadvantages of Historical Cost Accounting

1. Non-current asset values are unrealistic

2. Since non-current asset values are low, depreciation is low and does not fully
reflect the value of the asset consumed during the accounting year

3. Lower costs, e.g. depreciation expense, would lead to higher profits. There is
a possibility that this may lead to higher taxation, wage demands and dividend
expectation (based on overstated earnings per share). The combination of
these effects is that a company may overspend or over distribute its profits
and not maintain its capital base.

4. Comparisons over time are unrealistic

5. Understatement of asset values tends to overstate gearing, and leads to a low


asset per share value and can make the company vulnerable to a take over

6. Where assets, particularly land and buildings, are being used as security to
raise finance, it is current value that lenders are interested in, not historical
values

These disadvantages usually arise in times of rising prices. In fact, in times of


rising prices, historical cost accounting tends to understate asset values and
overstate profits.

___________________________________________________________________________

33
__________________________

KEY NOTES
_________________

___________________________________________________________________________

34
The IASB’s Conceptual Framework for Financial Reporting describes the basic
concepts by which financial statements are prepared.

Qualitative Characteristics of Financial Information

 Fundamental qualitative characteristics (relevance and faithful representation)


and
 enhancing qualitative characteristics (comparability (including consistency),
timeliness, verifiability and understandability).

Fundamental Qualitative Characteristics

For information to be useful, it must be both relevant and faithfully represented.

1. Relevance

 Influences economic decisions of user

 Has predictive value and/or confirmatory value or both

 Materiality

2. Faithful Representation

To be useful, financial information must not only be relevant, it must also represent
faithfully the phenomena it purports to represent.

Financial information that faithfully represents economic phenomena has three


characteristics: -

 it is complete
 it is neutral
 it is free from error

___________________________________________________________________________

35
Enhancing Qualitative Characteristics

1. Comparability

 Users can identify similarities and differences


 Consistent application of methods

Comparability should be distinguished from consistency (the consistent use of


accounting methods).

2. Verifiability

Financial information is verifiable when it enables knowledgeable and independent


observers to reach a consensus on whether a particular depiction of an event or
transaction is a faithful representation.

3. Timeliness

Timeliness means that information is available to decision-makers in time to be


capable of influencing their decisions.

4. Understandability

Understandability is enhanced when the information is:

 classified
 characterised
 presented clearly and concisely

Underlying Assumptions

• Accrual Basis

• Going Concern

___________________________________________________________________________

36
Other Accounting Concepts

1. The business entity concept (separate entity)

In accounting, a business should always be treated separately from its owner(s).

2. Fair presentation

The financial statements must "present fairly" the financial position, financial
performance and cash flows of an entity.

3. Substance over form

Transactions need to be accounted for and presented in accordance with their


substance and economic reality even if their legal form is different.

4. Historical cost

In times of rising prices, historical cost accounting tends to understate asset values
and overstate profits.

___________________________________________________________________________

37
_________________

QUESTION BANK
_________________

___________________________________________________________________________

38
1. Why is a conceptual framework necessary?
A. to provide a theoretical basis for preparing financial statements
B. to provide concepts on which to build a framework

2. The accounting concept which requires assets to be valued at their net book
value, rather than their 'breakup' value is the
A. materiality concept
B. going concern concept
C. prudence concept
D. business entity convention

3. Are these statements true or false?

A. To comply with the law, the legal form of a transaction must always be
reflected in financial statements.

True False

B. Standards provide guidance on accounting for all types of transaction.

True False

C. The framework assists in harmonising accounting practice.

True False

D. Materiality means that only items having a physical existence may be


recognised as assets.

True False

E. The substance over form convention means that the legal form of a
transaction must always be shown in financial statements even if this differs
from the commercial effect.

True False

___________________________________________________________________________

39
_________________

ANSWER BANK
_________________

___________________________________________________________________________

40
1 A

2 B

A. FALSE

B. FALSE

C. TRUE

D. FALSE

E. FALSE

___________________________________________________________________________

41
Chapter 4
The Main Elements of Financial Reports

4.1 ACCA SYLLABUS GUIDE OUTCOMES 1 and 2:


Understand and identify the purpose of each of the main financial statements.
Define and identify assets, liabilities, equity, revenue and expenses

The principle financial statements of a sole trader are the statement of financial
position and the statement of profit or loss.

Statement of Financial Position

The statement of financial position is a list of all the assets owned and the liabilities
owed by a business as at a particular date. It is a snapshot of the financial position
of the business at a particular moment.

4.1.1 Assets

An asset is a resource controlled by the entity as a result of past events and from
which future economic benefits are expected to flow to the entity.

Some assets are held and used in operations for a long time. These are known as
non-current assets.

Other assets are held for only a short time. They are likely to be realized within the
normal operating cycle or 12 months after the end of the reporting period. These are
classified as current assets.

Lecture Example 1

List some examples of assets found in a business and state whether they are
classified as non-current or current assets: -

a. _________________________________________________

b. _________________________________________________

c. _________________________________________________

d. _________________________________________________

___________________________________________________________________________

42
4.1.2 Liabilities

A liability is a present obligation of the entity arising from past events, the settlement
of which is expected to result in an outflow from the entity of resources embodying
economic benefits.

Some liabilities are due to be settled within the normal operating cycle or 12 months
after the end of the reporting period. These are classified as current liabilities.
Other liabilities may take some years to repay – non-current liabilities.

Lecture Example 2

List some examples of liabilities found in a business and state whether they are
classified as non-current or current liabilities: -

a. _________________________________________________

b. _________________________________________________

c. _________________________________________________

d. _________________________________________________

4.1.3 Capital / Equity

Capital is the amount invested in a business by the owner. This is the amount the
business owes to the owner. In the case of a sole trader,

CAPITAL = ASSETS – LIABILITIES

CAPITAL = NET ASSETS

In the case of a limited liability company, capital usually takes the form of shares.
Share capital is known as equity. The Framework defines equity as “the residual
interest in the assets of the entity after deducting all its liabilities.”

___________________________________________________________________________

43
A TRADER
PROFORMA STATEMENT OF FINANCIAL POSITION AS AT 30 APRIL 20X8
$ $

Assets

Non-Current Assets
Land and buildings 100,000
office equipment 80,000
Motor vehicles 30,000
Furniture and fixtures 10,000
220,000

Current Assets
Inventories 30,000
Trade Receivables 27,000
less: Allowance for receivables (2,000)
25,000
Prepayments 15,000
Cash in hand and at bank 10,000
80,000
Total assets 300,000

Capital and Liabilities


Capital
Capital 150,000
Profit 50,000
less: Drawings (20,000)
180,000
Non-Current Liabilities
Bank loans 60,000

Current Liabilities
Bank overdraft 20,000
Trade payables 30,000
Accruals 10,000
60,000
Total capital and liabilities 300,000

___________________________________________________________________________

44
Statement of profit or loss

A statement of profit or loss is a record of revenue generated and expenditure


incurred over a given period. The statement shows whether the business has had
more revenue than expenditure (a profit) or vice-versa (a loss)

4.2.1 Revenue

Revenue is the income for a period. It is the gross inflow of economic benefits (cash,
receivables, other assets) arising from the ordinary operating activities of an
enterprise (such as sales of goods, sales of services, interest, royalties, and
dividends).

4.2.2 Expenses

Expenses arise in the course of the ordinary activities of the enterprise. They
include, for example, cost of sales, wages and depreciation.

A TRADER
PROFORMA STATEMENT OF PROFIT OR LOSS FOR THE YEAR ENDED 30 APRIL
20X8

___________________________________________________________________________

45
$ $

Sales 300,000
Less: Cost of Sales
Opening Inventories 50,000
Purchases 200,000
Carriage Inwards 30,000
280,000
Closing Inventories (70,000)
210,000
Gross Profit 90,000
Sundry Income 15,000
Discounts Receivable 5,000
110,000
Less: Expenses
Telephone expenses 2,000
Office stationery 8,000
Wages and salaries 12,000
Depreciation expense 7,000
Bad and doubtful debts 4,000
Discounts allowed 2,000
Carriage out 1,000
Electricity expense 6,000
42,000
Profit for the year 68,000

Notes: -

1 The top part of the statement of profit or loss, i.e. Sales – Cost of Sales = Gross
Profit, is called the Trading Account. It records the trading activities of the
business.
2 Sundry income includes bank interest, rent receivable, income from investments.
3 Carriage inwards is the cost of transport of goods into the firm and is therefore
added to the purchases figure.
4 Carriage outwards is the cost of transport of goods out of the firm to its
customers, it is not part of the firm's expenses in buying the goods and is always
entered as an expense.

___________________________________________________________________________

46
__________________________

KEY NOTES
_________________

___________________________________________________________________________

47
1 Statement of Financial Position

The statement of financial position (SOFP) is a list of all the assets owned and the
liabilities owed by a business as at a particular date.

Statement of Financial Position

Assets Liabilities Capital

Current Non- Current Non-


Current Current

An asset is a resource controlled by the entity as a result of past events and from
which future economic benefits are expected to flow to the entity.

Non-current assets are assets that are held and used in operations for a long time.

Current assets are assets held for only a short time.

A liability is a present obligation of the entity arising from past events, the settlement
of which is expected to result in an outflow from the entity of resources embodying
economic benefits.

Non-current liabilities may take some years to repay.

Current liabilities are due to be settled within the normal operating cycle or 12
months after the end of the reporting period.

Capital is the amount invested in a business by the owner. This is the amount the
business owes to the owner.

___________________________________________________________________________

48
2 Statement of profit or loss

Statement of profit or loss

Revenue Expenses

Revenue is the gross inflow of economic benefits (cash, receivables, other assets)
arising from the ordinary operating activities of an enterprise (such as sales of
goods, sales of services, interest, royalties, and dividends).

Expenses arise in the course of the ordinary activities of the enterprise. They
include, for example, cost of sales, wages and depreciation.

___________________________________________________________________________

49
_________________

QUESTION BANK
_________________

___________________________________________________________________________

50
1 Which of the following is an example of a liability?

A Inventory
B Receivables
C Plant and machinery
D Loan

2 Which of the following is an example of a non-current asset?

A Inventory
B Receivables
C Plant and machinery
D Petty cash

___________________________________________________________________________

51
_________________

ANSWER BANK
_________________

___________________________________________________________________________

52
1. D - the others are all assets.

2. C - the others are all current assets

___________________________________________________________________________

53
Chapter 5
The Main Data Sources in an
Accounting System

5.1 ACCA SYLLABUS GUIDE OUTCOME 1:


Identify and explain the function of the main data sources in an accounting
system

A business will enter many transactions during the year. All of these need to be
recorded and summarized to produce the entity’s financial statements.

These business transactions are recorded on source documents. These


documents are the source of all the information recorded by a business. Examples
include sales and purchase orders, invoices and credit notes.

5.2 ACCA SYLLABUS GUIDE OUTCOME 2:


Outline the contents and purpose of different types of business
documentation, including: quotation, sales order, purchase order, goods
received note, goods despatched note, invoice, statement, credit note, debit
note, remittance advice, receipt

Documents used to record business transactions include: -

1. Quotation: - a business makes a written offer to a customer to produce or


deliver goods or services for a certain amount of money

2. Sales Order: - a customer writes out or signs an order for goods or


services he requires

3. Purchase Order: - a business orders from another business goods or


services

4. Goods received note: - a list of goods that a business has received from a
supplier

5. Goods despatched note: - a list of goods that a business has sent out to a
customer

___________________________________________________________________________

54
6. Invoice: - An invoice relates to a sales order or a purchase order. When a
business sells goods or services on credit to a customer, it sends out an
invoice. When a business buys goods or services on credit, it receives an
invoice from the supplier.

7. Statement: - A document sent by a supplier to a customer listing all


invoices, credit notes and payments done by the customer

8. Credit note: - a document sent by a supplier to a customer in respect of


goods returned or overpayments made by the customer

9. Debit note: - a document sent by a customer to a supplier in respect of


goods returned or an overpayment made. It is a formal request for the
supplier to issue a credit note

10. Remittance advice: - a document sent with a payment, detailing which


invoice are being paid and which credit notes offset

11. Receipt: - a written confirmation that money has been paid.

5.3 ACCA SYLLABUS GUIDE OUTCOME 3:


Identify the main books of prime entry, and understand their nature and
function
Record sale and purchase transactions in day books
Understand and record sales and purchase returns
Understand and illustrate the uses of journals
Understand the need for a record of petty cash transactions
Describe the features and operation of a petty cash imprest system
Account for petty cash using imprest and non-imprest methods
Understand the importance of, and identify controls and security over the
petty cash system.

Main Books of Prime Entry

Cash Sales Day Purchase Petty Cash Journal


Book Book Day Book Book Book

Sales Purchase
___________________________________________________________________________
Returns Returns
Day Book Day Book
55
5.3.1 Cash Book

The cash book records receipts and payments into and out of the business bank
account. These would include receipts and payments made by bank transfer,
standing order, direct debit and bank interest and charges, directly by the bank.

Cash Book (Receipts)

Date Narrative Total Accounts Cash


Receivable Sales Other
20x8 $ $ $ $
1 Oct Balance b/d 1,000
4 Oct Cash sale 500 500
12 Accounts receivable – Star 750 750
Oct & Moon Co
15 Sale of non-current asset 350 350
Oct
2,600 750 500 350

Cash Book (Payments)

Date Narrative Total Accounts Petty


Payable Cash Other
20x8 $ $ $ $
5 Oct Accounts payable – Jupiter 250 250
& Co
7 Oct Telephone 500 500
9 Oct Petty cash 155 155
12 Oct Purchase of machinery 1,250 1250
15 Oct Balance c/d 445
2,600 250 155 1,750

5.3.2 Sales Day Book

The sales day book lists all sales made on credit. It is used to keep a list of all
invoices sent out to customers each day.

___________________________________________________________________________

56
Sales Day Book

Date Invoice Customer Total amount


invoiced
20x8 $
Oct 4 145 Sunshine Co 452.60
146 Clouds Co 254.20
147 Star & Moon Co 845.90
1,552.70

5.3.3 Sales Returns Day Book

When customers return goods for some reason, a credit note is raised. All credit
notes are recorded in the sales returns day book.

5.3.4 Purchase Day Book

The purchase day book lists all purchases made on credit, i.e. a list of all invoices it
receives.

Purchase Day Book

Date Supplier Total amount


invoiced
20x8 $
Oct 1 Jupiter & Co 556.10
Mars Co 189.60
Venus & Co 245.50
991.20

5.3.5 Purchase Returns Day Book

The purchase returns day book records credit notes received in respect of goods
which the business sends back to its suppliers.

5.3.6 Petty Cash Book

Most businesses keep a small amount of cash on the premises to make occasional
small payments in cash, e.g. staff refreshments, postage stamps, to pay the office
cleaner, taxi fares, etc. This is often called the cash float or petty cash account.
Therefore, the petty cash book is a cash book for small payments.

___________________________________________________________________________

57
Very often these businesses use the imprest system. Under the imprest system, the
petty cash is kept at an agreed sum, so that each topping up is equal to the amount
paid out in the period.

Example: -

The amount of money in petty cash is kept at an agreed sum of $250. Expense
items are recorded on vouchers as they occur and the total voucher payments for
the period were $55. Therefore:
$
Cash still held in petty cash (250 - 55) 195
Plus voucher payments (25+5+10+15) 55
Must equal the agreed sum or float 250

The cash payment required from the bank account into petty cash is equal to $55,
i.e. total of the voucher payments since the previous top-up.

Receipts Payments
Date Narrative Total Date Narrative Total Postage Stationery
$ $ $ $
20x8
1 Oct Cheque 55 2 Oct Post 10 10
cashed Office

3 Oct XY 15 15
Bureau

25 10 15

Keeping cash (even in small amounts) on the premises is a security risk. Therefore a
petty cash system is usually subject to strict controls.

1. Payment is only made in respect of authorised claims.


2. All claims are supported by evidence.

Lecture Example 1: -

Petty cash is controlled under an imprest system. The imprest amount is $100.
During a period, payments totaling $53 have been made. How much needs to be
reimbursed at the end of the period to restore petty cash to the imprest account?

___________________________________________________________________________

58
5.3.7 Journal Book

The journal keeps a record of unusual movement between accounts. It is used to


record any double entries made which do not arise from the other books of prime
entry. For example, journal entries are made when errors are discovered and need
to be corrected and for period end adjustments (depreciation, bad and doubtful
debts, accruals and prepayments).

The format of a journal entry is: -

Debit Credit
$ $
Account to be debited
Account to be credited
Narrative to explain the transaction

Lecture Example 2

State which books of prime entry the following transactions would be entered
into:
i. Your business pays X Blackman (a supplier) $600
ii. You send Y Young (a customer) an invoice for $850.
iii. The marketing manager asks you for $100 urgently in order to buy some
postage stamps.
iv. You receive an invoice from X Blackman for $500.
v. You pay Y Young $500.
vi. F Clark (a customer) returns goods to the value of $650.
vii. You return goods to X Price to the value of $890.

5.4 Memorandum Ledgers

The main purpose of memorandum ledgers is to know how much is owed by each
particular customer or to a specific supplier at a point in time.

There are two main types of memorandum ledgers: -

1. Receivables Ledger
2. Payables Ledger

5.4.1 Receivables Ledger

This ledger shows how much is owed to the business by each individual customer

___________________________________________________________________________

59
Sunshine Co
Date Narrative Sales Cash Total
$ $ $
Oct 4 Invoice 145 452.60 452.60

Clouds Co
Date Narrative Sales Cash Total
$ $ $
Oct 4 Invoice 146 254.20 254.20

Star & Moon Co


Date Narrative Sales Cash Total
$ $ $
Oct 4 Invoice 147 845.90 845.90
Oct 12 750.00 95.90

5.4.2 Payables Ledger

This ledger shows how much is owed by the business to each individual supplier.

Jupiter Co
Date Narrative Cash Purchases Total
$ $ $
Oct 1 Invoice J851 556.10 556.10
Oct 5 Cash book 250.00 306.10

Mars Co
Date Narrative Cash Purchases Total
$ $ $
Oct 1 Invoice M048 189.60 189.60

Venus & Co
Date Narrative Cash Purchases Total
$ $ $
Oct 1 Invoice 0124 245.50 245.50

___________________________________________________________________________

60
__________________________

KEY NOTES
_________________

___________________________________________________________________________

61
1. The function of the main data sources in an accounting system

A business will enter many transactions during the year. All of these need to be
recorded and summarized to produce the entity’s financial statements.

2. Documents used to record business transactions

___________________________________________________________________________

62
3. The Main Books of Prime Entry

Main Books of Prime Entry

Cash Sales Day Purchase Petty Cash Journal


Book Book Day Book Book Book

Sales Purchase
Returns Returns
Day Book Day Book
Receipts
and Errors and
Small cash
payments Credit Credit Adjustments
transactions
Sales Purchases

Goods Goods
returned returned to
by supplier
customer
s

___________________________________________________________________________

63
4. Memorandum Ledgers

Memorandum Ledgers

Receivables Payables
Ledger Ledger

Amounts owed to
Amounts owed by each individual
each individual supplier
customer

___________________________________________________________________________

64
_________________

QUESTION BANK
_________________

___________________________________________________________________________

65
1. Which of the following is not a book of prime entry?

A. Sales invoice
B. Purchase day book
C. Sales day book
D. Journal

2. Which of the following is a source document for petty cash?

A. Purchase invoice
B. Quotation
C. Sales invoice
D. Receipt and claim form

3. What is the purchase returns day book used to record?

A. Supplier's invoices
B. Customer's invoices
C. Details of goods returned to suppliers
D. Details of goods returned by customers

4. All petty cash claims are automatically paid from petty cash.
Is this statement:

A. True
B. False

___________________________________________________________________________

66
_________________

ANSWER BANK
_________________

___________________________________________________________________________

67
1. A – a sales invoice is a source document

2. D

3. C

4. B – only authorized and evidenced claims are paid out of petty cash

___________________________________________________________________________

68
Chapter 6
Double-Entry Book-Keeping Principles

6.1 ACCA SYLLABUS GUIDE OUTCOME 1:


Understand and apply the concept of double-entry accounting and the duality
concept

6.1.1 Books of Prime Entry

As we have seen in the previous chapter, repetitive transactions may initially be


captured in day books (also known as books of prime entry) e.g., all the sales
invoices may be listed in the sales day book. These day books are not part of the
double-entry system but enable the number of double-entries to be reduced by
ascertaining an aggregate.

6.1.2 Nominal Ledger

The total of the day book, or the single transaction, is recorded in the double-entry
system by being posted to the nominal accounts in the general/nominal ledger. Each
nominal account (or T account) has two sides, the left hand side of which is called
the debit side (DR) and the right hand side of which is called the credit side (CR).

Example: -

Cash A/C

Debit side Credit side

Cash coming in Cash going out

Nominal accounts are normally opened for each asset and liability (or class thereof),
and one for each type of expense and income. In addition a sole trader will also have
an account for capital. Capital represents the proprietary interest in the net assets of
the business. It is created when the owner introduces resources into the business
entity and increases when the business generates a profit.

As already mentioned, only transactions capable of being measured objectively in


monetary terms can be recorded (this is known as the money measurement
concept).
___________________________________________________________________________

69
6.1.3 Double-entry rules

Rule 1: - The duality rule

Every transaction has two effects, one of which will be recorded as a debit in one
account and the other which will be recorded as a credit in another account. If this
rule is broken, the trial balance will not agree and a suspense account is opened.
This will be discussed later in “Correction of Errors”.

TOTAL DEBITS = TOTAL CREDITS

Rule 2: - The when to DR and CR rule

The rules as to when to debit a T account and when to credit a T account can be
summarized in the following table.

The Debit/Credit Table: -

Increase Decrease
Asset
Expense Debit Credit
Purchases
Drawings
Liability
Income Credit Debit
Sales
Capital

Rule 3: - Debit is on the left and credit is on the right

___________________________________________________________________________

70
6.2 ACCA SYLLABUS GUIDE OUTCOME 2:
Identify the main types of business transactions e.g. sales, purchases,
payments, receipts

Lecture Example 1: -

Apply the 3 double-entry rules to the following transactions. State the double-entry
of each of these transactions and open the necessary nominal accounts.

1. Land is bought for cash, $80,000


2. An item of plant and machinery is bought on credit, $65,000
3. A sale of goods on credit is made for $33,000
4. Goods are purchased on credit for $25,000
5. Paid water and electricity during the year for $5,000
6. Received rental income of $2,500
Lecture Example 2: -

In the books of P. Powell

20X7
Jan 1 Paid $50,000 into a business bank account
Jan 3 Bought shop premises $20,000, paid by cheque
Jan 5 Bought shop fittings, paid by cheque $2,500
Jan 9 Bought goods for resale $14,000, paid by cheque
Jan 10 Withdrew $100 cash from the bank for his own personal use
Jan 11 Cash sales $5,000
Jan 12 Bought stationery, paid in cash $20
Jan 16 Bought a van, paid by cheque $2,500
Jan 19 Sold goods for $8,500, received payment by cheque
Jan 23 Paid petrol bill for van, $15 by cheque
Jan 26 Sold goods on credit to J. Parker $3,500
Jan 28 Paid for shop cleaning $20 cash
Jan 29 Bought goods for resale $3,000 on credit from A. Young
Jan 31 Received cheque from J. Parker for $2,000 in part payment of his
account

Inventory in hand, at cost price, was valued at $4,000 on 31 January 20X7

Required: -

Post all transactions to relevant ledger accounts

___________________________________________________________________________

71
6.3 ACCA SYLLABUS GUIDE OUTCOME 3:
Illustrate how to balance and close a ledger account

The totals from the books of prime entry are posted to the nominal accounts in the
nominal ledger through double-entry. A business will want to know the balance on
each account. This is done by 'balancing off' each account.

6.3.1 Steps to balance off a ledger account: -

1. Add the debit and credit sides separately.


2. Fill in the higher of the two totals on both sides.
3. 'Balance' the account (make the two sides equal) – balance c/d
4. Complete the 'double entry' – balance b/d on the opposite side.

Lecture Example 3: -

Refer to Lecture Example 2 P. Powell.

Required: -

Balance off the ledger accounts in the books of P. Powell.

___________________________________________________________________________

72
__________________________

KEY NOTES
_________________

___________________________________________________________________________

73
1. Books of Prime Entry and the Nominal Ledger

Repetitive transactions are initially captured in books of prime entry. The total of
each book of prime entry, or the single transaction, is recorded in the double-entry
system by being posted to the nominal accounts in the general/nominal ledger.

The principles of double entry work on the basis that for each debit entry there must
be a credit entry (total debits must equal total credits). This is also known as the dual
effect.

2. The Flow of Information

Assorted business
transactions

Categorized in books of
prime entry

Total transferred to nominal ledger


using double entry

Preparation and presentation of financial


statements: - trial balance, statement of profit or
loss and statement of financial position

___________________________________________________________________________

74
3. The Double-Entry Rules

Increase Decrease
Asset
Expense Debit Credit
Purchases
Drawings
Liability
Income Credit Debit
Sales
Capital

4. Balancing-off the ledger accounts

At the end of each period the nominal ledger accounts are 'balanced off' to
determine the closing balance on each nominal account.

___________________________________________________________________________

75
__________________________

QUESTION BANK
_________________

___________________________________________________________________________

76
1. In double-entry book-keeping, which of the following statements is correct?

A. Credit entries decrease liabilities and increase income


B. Debit entries decrease income and increase assets
C. Credit entries decrease expenses and increase assets

2. State the double-entry of each of these transactions: -

a. Bought a computer on credit from Star Ltd, cost $10,000.


b. Bought goods on credit from Club Co, cost $4,500.
c. Sold goods on credit to Boomerang Co, value $3,200.
d. Paid Demo Co (a credit supplier) $500.
e. Received $280 by cheque from Acorn Ltd, a credit customer.
f. Paid wages $4,000 in cash.
g. Paid annual rent for warehouse of $2,700 in cash.
h. Paid insurance premium $100 in cash.
i. Received a credit note for $500 from Price Co, a supplier.
j. Sent out a credit note for $200 to Boomerang Co.

3. A debit balance of $3,000 brought down on A Co’s account in B Co’s books


means that: -

A. A Co is owed $3,000 by B Co
B. B Co has sold $3,000 of goods to A Co
C. B Co is owed $3,000 by A Co
D. A Co has sold $3,000 of goods to B Co

4. Which of the following is the best description of the function of the books of prime
entry in a standard double entry bookkeeping system?

A. Books of prime entry record amounts owed to/from individual suppliers


and customers
B. Books of prime entry are used to list similar transactions with the totals
being posted to the nominal ledger
C. Books of prime entry are used to record cash transactions
D. Books of prime entry are used to summarise credit transactions

5. Which of the following statements is true?

A. A debit records an increase in liabilities


B. A debit records a decrease in assets
C. A credit records an increase in liabilities

___________________________________________________________________________

77
6. How is the total of the purchases daybook posted to the nominal ledger?

A. Debit purchases, credit cash


B. Debit payables control, credit purchases
C. Debit cash, credit purchases
D. Debit purchases, credit payables control

7. You have bought stationery with a company cheque from the stationery shop
next door. How should this transaction be recorded?

A. DR Stationery CR Payable
B. DR Cash CR Payable
C. DR Stationery CR Cash
D. DR Payable CR Cash

___________________________________________________________________________

78
__________________________

ANSWER BANK
_________________

___________________________________________________________________________

79
1. B

2. (a) Debit Computer equipment $10,000


Credit Payables $10,000

(b) Debit Purchases $4,500


Credit Payables $4,500

(c) Debit Receivables $3,200


Credit Sales $3,200

(d) Debit Payables $500


Credit Cash $500

(e) Debit Bank $280


Credit Receivables $280

(f) Debit Wages $4,000


Credit cash $4,000

(g) Debit Rent $2,700


Credit Cash $2,700

(h) Debit Insurance $100


Credit Cash $100

(i) Debit Payables $500


Credit Returns out $500

(j) Debit Returns in $200


Credit Receivables $200

3. C

B is wrong as the sale has to be on credit.

4. B

5. C is correct

A – A debit records a DECREASE in liabilities.

B – A debit records an INCREASE in assets.

6. D – only credit purchases are listed in the purchases day book

___________________________________________________________________________

80
7. C - this is a cash payment, so the stationery shop is not a supplier with
whom you have an account

___________________________________________________________________________

81
Chapter 7
PREPARING FINANCIAL STATEMENTS

In Chapter 6, we have seen the posting of transactions into ledger accounts using
double-entry (duality concept). The entries in each ledger account are then totaled
and a balance is found. Balances are usually collected in a trial balance which is
then used as a basis for preparing a statement of profit or loss and a statement of
financial position.

7.1 ACCA SYLLABUS GUIDE OUTCOME 1:


Identify the purpose of a trial balance.

A trial balance is a list of ledger balances shown in debit and credit columns. It lists
the balances on ledger accounts and totals them. Total debits should equal total
credits. Therefore, it is a method used to test the accuracy of the double-entry
bookkeeping, i.e. the accuracy of the accounting records.

7.2 ACCA SYLLABUS GUIDE OUTCOMES 2 and 3:


Extract ledger balances into a trial balance
Prepare extracts of an opening trial balance

Lecture Example 1

As at 31.12.X7, a business had the following balances on its ledger accounts.

$
Bank overdraft 2,000
Cash 11,700
Capital 13,000
Rent expense 1,880
Purchases 12,400
Sales 24,600
Trade payables 12,820
Trade receivables 12,000
Other expenses 12,420
Equipment 2,020

Required: -

Prepare the trial balance as at 31.12.X7

___________________________________________________________________________

82
Lecture Example 2

Refer to Chapter 6 Lecture Example 2

Prepare the trial balance for P Powell as at 31 January 20X7

7.3 ACCA SYLLABUS GUIDE OUTCOME 4:


Identify and understand the limitations of a trial balance

We have seen that the trial balance is a method used to test the accuracy of the
accounting records. Therefore, if the two columns of the list are not equal, there must
be an error in recording the transactions in the accounts. However, the trial balance
will not disclose the following types of errors.
1. The complete omission of a transaction, because neither a debit nor a credit
is made.
2. The posting of a debit or credit to the correct side of the ledger, but to a wrong
account.
3. Compensating errors (e.g. an error of $500 is exactly cancelled by another
$500 error elsewhere).
4. Errors of principle, e.g. cash from receivables being debited to receivables
account and credited to cash at bank instead of the other way round.

These errors will be discussed again in the chapter “Correction of Errors”.

Closing Inventories

A business will purchase goods to sell during the year. It is unlikely that all of these
goods will have been sold by the year end. The goods still held at the year end are
known as closing inventories. These are an asset of the business and so should be
included in the statement of financial position. Also, these inventories will be included
in the cost of sales calculation. When a business determines its profit for the year it
should match the sales revenue earned to the cost of goods it sold.

The double-entry for closing inventories is: -

Dr Inventories (SOFP)
Cr Closing Inventories (COS)

Closing inventories will be discussed in further detail in the chapter “Inventories”.

___________________________________________________________________________

83
7.4 ACCA SYLLABUS GUIDE OUTCOME 5:
Prepare extracts of a statement of profit or loss from given information
Calculate revenue, cost of sales, gross profit, and profit for the year from
given information
Disclose items of income and expenditure in the statement of profit or loss.

The first step in the process of preparing the financial statements is to open up
another ledger account, called the statement of profit or loss. The balances on all the
income and expenditure T-accounts are transferred to the statement of profit or loss
and the closing inventory adjustment is made. The statement of profit or loss is part
of the double entry system, so the basic rule of double entry still applies: every debit
must have an equal and opposite credit entry.

Lecture Example 3

Refer to Chapter 6 Lecture Example 2

Prepare the statement of profit or loss for P Powell both as a ledger account and in
vertical form.

7.5 ACCA SYLLABUS GUIDE OUTCOME 6:


Prepare extracts of a statement of financial position from given information.

The balances on all remaining ledger accounts (including the profit or loss in the
statement of profit or loss) can be listed and rearranged to form the statement of
financial position. A credit balance brought down denotes a liability. An asset would
be represented by a debit balance brought down. The statement of financial position
is not part of the double-entry system so the balances are not transferred out.

Lecture Example 4

Refer to Chapter 6 Lecture Example 2

Prepare the statement of financial position for P Powell as at 31 January 20X7.

7.6 ACCA SYLLABUS GUIDE OUTCOME 7:


Recognise how the accounting equation and business entity convention
underlie the statement of financial position
Understand and apply the accounting equation

The accounting equation expresses the statement of financial position as an


equation. It emphasises the equality between assets and liabilities (including capital
___________________________________________________________________________

84
as a liability). In accounting, capital is an investment of money (funds) with the
intention of earning a return. A business proprietor invests capital with the intention
of earning profit. As long as that money is invested, accountants will treat the capital
as money owed to the proprietor by the business. Also, the business entity concept
states that, regardless of how a business is legally set up, in accounting a business
is always treated separately from its owners(s).

The Accounting Equation: -

E.g. Statement of Financial Position – M. Stark

Assets $ Liabilities $

Motor Vehicles 10,000 Trade payables 3,000


Inventory 4,000 Proprietor’s interest:
Receivables 2,000 Capital 13,000
Cash 3,000 Profit 4,000
Drawings (1,000)
19,000 19,000

Assets = Liabilities

Assets = (Capital + Profit – Drawings) + Payables


Assets – Payables = Capital +Profit – Drawings

Net Assets = Proprietor’s Interest

Lecture Example 5

Refer to Chapter 6 Lecture Example 2

Prepare the accounting equation for P Powell

Lecture Example 6

At 1.1.X3 Henry has net assets of $120,000. During the year he puts in capital of
$50,000 and draws out $90,000. His net assets at 31.12.X3 are $25,000.

Required:

What is his profit or loss for the year?

___________________________________________________________________________

85
Lecture Example 7

Nancy's business has net assets of $13,200 at the beginning of the year. During the
following month she purchases new equipment for $1,200, makes sales on credit of
$7,500, receives payments from customers of $3,750 and receives bills from
suppliers of $2,250. These are not payable until next month.

What are the net assets at the end of the month?

A. $15,750
B. $18,450
C. $18,150
D. $20,700

___________________________________________________________________________

86
__________________________

KEY NOTES
_________________

___________________________________________________________________________

87
1. Trial Balance

A trial balance is a list of ledger balances shown in debit and credit columns.
Total debits should equal total credits. Therefore, it is a method used to test
the accuracy of the double-entry bookkeeping.

2. Limitations of a Trial Balance

The trial balance will not disclose the following types of error:
a) The complete omission of a transaction
b) The posting of a debit or credit to the correct side of the ledger,
but to a wrong account
c) Compensating errors
d) Errors of principle

3. Closing Inventory Adjustment

Closing inventories are items which are still held at year-end. They are
included as assets in the statement of financial position. They are also
included in the calculation of cost of sales in the trading account (Opening
inventories + Purchases – Closing inventories)

The double-entry for closing inventories is: -

Dr Inventories (SOFP)
Cr Closing Inventories (COS)

4. The Statement of profit or loss

The balances on all the income and expenditure T-accounts are transferred to
the statement of profit or loss. The statement of profit or loss is part of the
double entry system.

5. The Statement of Financial Position

The balances on all remaining ledger accounts (including the balance of


profit/loss on the statement of profit or loss) can be listed and rearranged to
form the statement of financial position. The statement of financial position is
not part of the double-entry system so the balances are not transferred out

___________________________________________________________________________

88
6. The Accounting Equation: -

Assets Liabilities
=

Capital Profit less Payables


Drawings

Net Assets = Proprietor’s Interest

___________________________________________________________________________

89
__________________________

QUESTION BANK
_________________

___________________________________________________________________________

90
1. Fred Jones set up in business on 1 June 20X6 by putting $25,000 into a
business bank account.

Given below are his transactions for the month of June:

a) purchased a delivery van with a cheque for $8,000


b) purchased goods for resale with a cheque for $6,000
c) paid 3 months rent by cheque totalling $2,400
d) sold goods on credit for $2,700
e) purchased office stationery for cash of $100
f) purchased goods on credit for $4,500
g) sold goods on credit for $5,200
h) sold goods for cash for $1,200
i) paid telephone bill by cheque totalling $140
j) sold goods on credit for $3,000
k) paid creditors $2,500 by cheque
l) withdrew $1,200 from the bank account for personal use
m) received $3,600 from debtors by cheque

Write up these transactions in Fred’s ledger accounts.


Balance off each of the accounts in Fred’s ledger showing clearly the
balance carried down and brought down and prepare a trial balance.

2. William Young

William Young has the following transactions to record for the month of May:

1 May Paid $10,000 capital into his business


2 May Bought a motor car for $2,000 cash
3 May Bought goods to the value of $4,000, and paid for them
immediately
4 May Sold goods for $3,000 cash
5 May Paid rent of $500 cash (representing half of the monthly rent)
6 May Bought goods costing $2,000 on credit from Higgins
7 May Bought a computer costing $2,500 from Snow, paying a deposit
of 20% cash
8 May Sold goods for $1,600 on credit to Mrs. Grey
9 May Paid motor running expenses of $50 cash
10 May Paid $1,000 to Higgins in cash
11 May Bought 100 shares in Eliza Co. for $600 cash
12 May Mrs Grey paid in full the amount owing in cash
13 May Bought stationery for $150 on credit from Mrs Pearce
___________________________________________________________________________

91
14 May Paid the balance owing to Higgins in cash
15 May Sold goods to Klent for $1,700 on credit
16 May Sold goods to Frog for $750 on credit
17 May Received a dividend from Eliza Co. of $25 cash
18 May Received an invoice for the remainder of this months rent
19 May Bought a van for $5,000, taking out a two-year term loan to do
so
20 May Bought trade goods from Corvax to the value of $6,000 on credit
21 May Sold goods to Mrs Grey for $3,200 on credit
22 May Paid insurance of $200 cash
23 May Drew cash for personal expenses of $300
24 May Paid bank charges of $60
25 May Paid Mrs Pearce in full in cash
26 May Received full payment from Klent in cash
27 May Paid employee's salary of $400 in cash
28 May Sold goods for $5,000 cash
29 May Paid Corvax in full for trade goods in cash
30 May Received $400 from Frog in cash
31 May Received $1,600 from Mrs Grey in cash

Required

A. Show how these transactions would be recorded in William Young's books


of account.
You are required to write out the double entry and then post the transactions
into the relevant ‘T’ accounts.
B. Balance off the accounts.
C. Produce a trial balance
D. Produce a statement of profit or loss and a statement of financial position

3. The accounting equation can be rewritten as

A. assets plus profit less drawings less liabilities equals closing capital
B. assets less liabilities less drawings equals opening capital plus profit
C. assets less liabilities less opening capital plus drawings equals profit

4. Which of the following is the correct format for the accounting equation?

A. Assets + Liabilities = Capital


B. Assets + Capital = Liabilities
C. Assets – Liabilities = Capital

___________________________________________________________________________

92
5. Which one of the following would not cause a trial balance imbalance?

A. An error of single entry


B. A transposition error
C. An error of principle
D. An omitted account

6. These are the year end balances for Josie's business.

$
Sales 54,000
Purchases 21,000
Inventory 9,500
Cash 27,250
Receivables ?
Motor vehicle 7,500
Payables 5,500
Capital 18,500

If the trial balance balances, what is the missing figure for receivables?

__________________

7. Which of the following is the correct posting from the sales day book to the
nominal ledger?

A. DR Sales; CR Receivables control account


B. DR Cash; CR Receivables control account
C. DR Receivables control account; CR Sales
D. DR Receivables control account; CR Cash

8. Bill, a sole trader, set up business on 1 October 20X0 with $30,000 of his own
money. During the year to 30 September 20X1 he won $50,000 on the lottery and
paid $30,000 of this into his business. He took cash drawings of $5,000 during
the year and at 30 September 20X1 the net assets of the business totalled
$59,000.

What was the profit or loss of the business for the year ended 30 September
20X1?

A. $4,000 profit
B. $6,000 profit
C. $16,000 loss
D. $6,000 loss
___________________________________________________________________________

93
9. Harry has been unable to calculate his business' profit or loss for the year ended
31 December 20X8 as fire destroyed most of his accounting records. He has,
however, been able to provide the following information.

1. Net assets at 31 December 20X7 were $23,000 and $32,500 at 31


December 20X8
2. He introduced capital during the year of $4,000 cash
3. He took cash drawings of $2,500 and goods with a selling price of $800, the
cost of the goods was $750.

What was Harry's profit or loss for the year ended 31 December 20X8?

A. $8,750 profit
B. $1,750 loss
C. $9,800 profit
D. $2,750 loss

___________________________________________________________________________

94
__________________________

ANSWER BANK
_________________

___________________________________________________________________________

95
1. Bank account
$ $
Capital 25,000 a) Delivery Van 8,000
h) Sales 1,200 b) Purchases 6,000
m) Receivables Control 3,600 c) Rent 2,400
e) Stationery 100
i) Telephone 140
k) Payables control 2,500
l) Drawings 1,200
Balance c/d 9,460
29,800 29,800
Balance b/d 9,460

Capital account
$ $
Bank 25,000

Delivery Van account


$ $
a) Bank 8,000

Purchases account
$ $
b) Bank 6,000
f) Payables Control 4,500 Balance c/d 10,500
10,500 10,500
Balance b/d 10,500

Rent account
$ $
c) Bank 2,400

___________________________________________________________________________

96
Receivables ledger control account
$ $
d) Sales 2,700 m) Bank 3,600
g) Sales 5,200
j) Sales 3,000 Balance c/d 7,300
10,900 10,900
Balance b/d 7,300

Sales account
$ $
d) Receivables ledger control 2,700
g) Receivables ledger control 5,200
h) Bank 1,200
Balance c/d 12,100 j) Receivables ledger control 3,000
12,100 12,100
Balance b/d 12,100

Stationery account
$ $
e) Bank 100

Payables ledger control account


$ $
k) Bank 2,500 f) Purchases 4,500
Balance c/d 2,000
4,500 4,500
Balance b/d 2,000

Telephone account
$ $
i) Bank 140

Drawings account
$ $
i) Bank 1,200

___________________________________________________________________________

97
Trial balance
Debits Credits
$ $
Bank 9,460
Capital 25,000
Delivery van 8,000
Purchases 10,500
Rent 2,400
Receivables ledger control account 7,300
Sales 12,100
Stationery 100
Payables ledger control account 2,000
Telephone 140
Drawings 1,200
39,100 39,100

2. William Young

Trial Balance as at 31 May


DR CR
$ $
Cash at bank 6,565
Capital 10,000
Motor Car 2,000
Purchases 12,000
Sales 15,250
Rent 1,000
Computer 2,500
Payables – Snow 2,000
Receivables – Mrs Grey 1,600
Motor Running Expenses 50
Investment – Eliza plc 600
Stationery 150
Receivables – Frog 350
Dividends Received 25
Accruals – rent 500
Van 5,000
Loan 5,000
Insurance 200
Drawings 300
Bank Charges 60
Wages 400
32,775 32,775

___________________________________________________________________________

98
Statement of profit or loss for the month ended 31 May

$ $
Sales 15,250
less cost of sales 12,000
Gross Profit 3,250

Dividends Received 25
3,275

Less expenses
Rent 1,000
Motor 50
Stationery 150
Insurance 200
Bank Charges 60
Wages 400 1,860
Net Profit 1,415

Statement of financial position as at 31 May

Non-current Assets $ $
Investment 600
Motor Car 2,000
Computer 2,500
Van 5,000
10,100

Current Assets
Receivables (1,600+350) 1,950
Cash and cash equivalents 6,565
8,515
18,615

Financed by:

Capital at 1 May 10,000


add Profit for the month 1,415
11,415

___________________________________________________________________________

99
less Drawings 300
11,115

Non-Current Liabilities
Loan 5,000

Current Liabilities
Payables 2,000
Accruals – rent 500
2,500
18,615

3. C

4. C

5. C – There will still be a debit and a credit entry.

6. $12,750

Dr Cr
Sales 54,000
Purchases 21,000
Inventory 9,500
Cash 27,250
Motor vehicle 7,500
Payables 5,500
Capital 18,500
i.e. Receivables 12,750 _____
78,000 78,000

7. C

8. A

$
Net assets 1.10.X0 30,000
Capital introduced 30,000
Drawings (5,000)
Profit (missing figure) 4,000
Net assets 30.09.x1 59,000

___________________________________________________________________________

100
9. A

Using the accounting equation

$
Net assets at 31.12.X8 32,500

Opening capital 23,000


Capital introduced 4,000
Profit (balancing figure) 8,750
Drawings (2,500 + 750) (3,250)
32,500

Goods drawn by proprietor are recorded at cost

___________________________________________________________________________

101
Chapter 8
Accounting for Sales Tax

8.1 ACCA SYLLABUS GUIDE OUTCOME 1:


Understand the general principles of the operation of a sales tax

8.1.1 Introduction

Sales tax is an indirect tax on the supply of goods and services which is eventually
borne by the final customer, but it is collected at each stage of the production and
distribution chain.

Lecture Example 1

A digital radio manufacturer purchases materials and components at $100, net of


sales tax. He sells the finished radio to a wholesaler at $250, excluding sales tax.
The wholesaler sells the radio to the retailer at $350, net of sales tax. The retailer
sells the digital radio to his customers at $470, inclusive of sales tax. It is assumed
that the rate of sales tax is 17.5%.

Price net of Sales Tax Price gross of


Sales Tax at 17.5% Sales Tax
Manufacturer
Output tax
Input tax
Wholesaler
Output tax
Input tax
Retailer
Output tax
Input tax
Total amount
paid to tax
authorities

___________________________________________________________________________

102
8.1.2 Input and Output Tax

Sales tax charged on goods and services sold by a business is referred to as output
tax. Sales tax paid on goods and services ‘bought in’ by a business is referred to as
input tax.

If output sales tax exceeds input sales tax, the business pays the difference in tax to
the authorities. If output sales tax is less than input sales tax in a period, the tax
authorities will refund the difference to the business.

8.2 ACCA SYLLABUS GUIDE OUTCOME 2:


Calculate sales tax on transactions and record the consequent accounting
entries.

8.2.1 Accounting Treatment

Registered businesses charge output sales tax on sales and suffer input sales tax on
purchases. Sales tax does not affect the statement of profit or loss, but is simply
being collected on behalf of the tax authorities to whom a quarterly payment is made.

Therefore, if a business sells goods for $1,000 + 17.5% sales tax, the accounting
entries to record the sale would be: -

Dr Cash/trade receivables(Gross) $1,175


Cr Sales (Net) $1,000
Cr Sales tax control account $ 175

If input sales tax is recoverable, the cost of purchases should exclude the sales tax
and be recorded net of tax. Therefore, if a business purchases goods on credit for
$500 + 17.5% sales tax, the accounting entries would be: -

Dr Purchases $500.00
Dr Sales tax control account $ 87.50
Cr Cash/trade payables $587.50

Lecture Example 2
All of Jimmy's sales are subject to sales tax at 17.5%. He invoices a customer for
$1,700 plus sales tax and receives a cheque in full payment the following month.
How is the cheque posted to the nominal ledger?

A. DR Cash $1,997.50; CR Sales $1,700; CR Sales Tax $297.50


B. DR Receivables ledger control account $1,700; DR Sales Tax $297.50;
CR Cash $1,997.50
C. DR Cash $1,700; DR Sales Tax $297.50; CR Sales $1,997.50
D. DR Cash $1,997.50; CR Receivables ledger control account $1,997.50
___________________________________________________________________________

103
8.2.2 Irrecoverable Sales Tax

There are some circumstances in which traders are not allowed to reclaim sales tax
paid on their inputs. For e.g. sales tax charged on motor cars, other than for resale,
and on certain business entertaining expenses is irrecoverable.

In these cases, sales tax must be regarded as part of the cost of the items
purchased and included in the statement of profit or loss charge or in the statement
of financial position as appropriate.

Therefore, the double entry for buying a motor vehicle, where sales tax is
irrecoverable, is: -

Dr Motor Vehicles A/c (cost + sales tax)


Cr Cash A/c (cost + sales tax)

Lecture Example 3

The following information relates to Eva Co's sales tax for the month of March 20X3:
$
Sales (including sales tax) 109,250
Purchases (net of sales tax) 64,000

Sales tax is charged at a flat rate of 15%. Eva Co's sales tax account showed an
opening credit balance of $4,540 at the beginning of the month and a closing debit
balance of $2,720 at the end of the month.

What was the total sales tax paid to regulatory authorities during the month of March
20X3?

A. $6,470.00
B. $11,910.00
C. $14,047.50
D. $13,162.10

___________________________________________________________________________

104
__________________________

KEY NOTES
_________________

___________________________________________________________________________

105
1. Sales Tax – General Principles

Sales Tax

Indirect Tax

Amount
Output Tax - Input Tax = payable to
tax
authorities

Difference transferred to tax authorities

Sales tax charged on goods and services sold by a business is referred to as output
tax. Sales tax paid on goods and services ‘bought in’ by a business is referred to as
input tax.

2. Accounting Treatment

The accounting entries to record the sale would be: -

Dr Cash/trade receivables (GROSS)


Cr Sales (NET)
Cr Sales tax control account (SALES TAX ELEMENT)

The accounting entries to record a purchase would be: -

Dr Purchases (NET)
Dr Sales tax control account (SALES TAX ELEMENT)
Cr Cash/trade payables (GROSS)

___________________________________________________________________________

106
3. Irrecoverable Sales Tax

Where sales tax cannot be recovered, it must be regarded as part of the cost of the
items purchased and included in the statement of profit or loss charge or in the
statement of financial position as appropriate

___________________

QUESTION BANK
___________________

___________________________________________________________________________

107
1. W is registered for sales tax. The managing director has asked four staff in the
accounts department why the output tax for the last quarter does not equal 17.5%
of sales (17.5% is the rate of tax). Which one of the following four replies she
received was not correct?

A. The company had some exports that were not liable to sales tax
B. The company made some sales of zero-rated products
C. The company made some sales of exempt products
D. The company sold some products to businesses not registered for
sales tax

2. Anwar makes sales in the quarter of $34,075 including sales tax at 17.5%. His
total purchases net of sales tax are $11,010, of which $2,500 is for zero rated
goods.

How much should he pay to the government?

A. $4,473.88
B. $3,435.21
C. $3,585.75
D. $3,807.55

3. Sales tax is due on all sales. Is this statement correct?

A. Yes
B. No

4. A business in its first period of trading charges $4,000 of sales tax on its sales
and suffers $3,500 of sales tax on its purchases which include $250 sales tax on
business entertaining. What is the amount owed to tax authorities?

_____________

___________________________________________________________________________

108
5. A sales tax registered trader has recorded the following transactions during the
accounting period.
$
Standard rated sales 200,000
Purchases 150,000

Included in purchases are the purchases of a motor car for $20,000, a


photocopier for $8,000 and entertaining expenses of $5,000. Sales tax is not
recoverable on the motor car or entertainment expenses.

All figures are given inclusive of sales tax at 17.5%.

How much input tax can be reclaimed by the trader?

_______________

6. With respect to sales tax which of the following statements is correct?

A. Sales tax is charged on purchases and sales after trade discounts and
before settlement discounts.
B. Exempt and zero rated supplies have the same tax effect.
C. Sales tax is not reclaimable on capital expenditure.
D. Sales tax is charged on purchases and sales after trade discounts and
after settlement discounts

___________________________________________________________________________

109
______________________

ANSWER BANK
______________

___________________________________________________________________________

110
1. D

2. C

Output tax (34,075 x 17.5/117.5) 5,075.00


Input tax ((11,010 – 2,500) x 17.5%) (1,489.25)
Due to government 3,585.75

3. B – sales tax is only due on taxable outputs

4. $750

Output tax 4,000


Input tax (3,500 – 250) 3,250
Amount owed to tax authorities 750

N.B. Tax on business entertaining is irrecoverable.

5. $18,617

$
Input tax
Purchases 150,000
Less: motor car (20,000)
entertaining (5,000)
125,000

Tax included @ 17.5% = $125,000 × 17.5


117.5 = $18,617

6. D

___________________________________________________________________________

111
Chapter 9
Inventories

9.1 ACCA SYLLABUS GUIDE OUTCOME 1:


Recognize the need for adjustments for inventory in preparing financial
statements

Inventories are assets:


1. held for sale in the ordinary course of business;
2. in the process of production for such sale; or
3. in the form of materials or supplies to be consumed in the production process
or in the rendering of services.

Inventory can be a significant figure for some businesses, e.g. manufacturing


companies.

It affects the financial statement in two ways:

1. Statement of financial position: it is included as a current asset


2. Statement of profit or loss: opening and closing inventory have a direct impact
on cost of sales and therefore profits. (The cost of goods sold is calculated as:
Opening inventory + Purchases – Closing inventory).

All businesses must therefore ensure that their financial statements account for
inventory accurately in terms of:
1. the accounting adjustment
2. its valuation

9.2 ACCA SYLLABUS GUIDE OUTCOME 2:


Record opening and closing inventory

Inventory is generally accounted for as a year end adjustment via a journal entry.

9.2.1 Opening Inventories

These are the goods held by the business at the beginning of the year. However,
such goods will normally have been sold during the year. They are no longer an
asset of the entity but will form part of the costs that should be matched against
sales revenue when determining profit.

Therefore, opening inventories brought forward in the inventory account are


transferred to the trading account.

___________________________________________________________________________

112
The accounting entry is:

Dr Cost of sales (I/S)


Cr Inventories (SOFP)

9.2.2 Closing Inventories

Goods might be unsold at the end of an accounting period and so still be held in
inventory.

The value of closing inventories is accounted for in the nominal ledger by debiting an
inventory account and crediting the trading account at the end of an accounting
period. Inventory will therefore have a debit balance at the end of a period, and this
balance will be shown in the statement of financial position as a current asset.

The accounting entry is:

Dr Inventories (SOFP)
Cr Cost of sales (I/S)

Lecture Example 1

A business had an opening inventory of $180,000 and a closing inventory of


$220,000 in its financial statements for the year ended 31 December 2005.

Which of the following entries for these opening and closing inventory figures are
made when completing the financial records of the business?

Debit Credit
$ $
A. Inventory account 180,000
Statement of profit or loss 180,000
Statement of profit or loss 220,000
Inventory account 220,000

B. Statement of profit or loss 180,000


Inventory account 180,000
Inventory account 220,000
Statement of profit or loss 220,000

C. Inventory account 40,000


Purchases account 40,000

D. Purchases account 40,000


Inventory account 40,000
___________________________________________________________________________

113
9.3 ACCA SYLLABUS GUIDE OUTCOME 3:
Identify the alternative methods of valuing inventory

The inventories figure is made up of two elements: -

1 Quantity: - The quantity of inventories held at the year end is established by


means of a physical count of inventory in an annual counting exercise, or by a
'continuous' inventory count.

2 Valuation: - The basic rule as per IAS 2 “Inventories” states that:

Inventories should be measured at the lower of cost and net realisable value

The value of inventories is calculated at the lower of cost and net realisable value for
each separate item or group of items.

Here, the prudence concept is being applied in presenting financial information.

There are other methods which, in theory, might be used for the valuation of
inventory: -

(i) Inventories might be valued at their expected selling price.

(ii) Inventories might be valued at their expected selling price, less


any costs still to be incurred in getting them ready for sale and
then selling them. This amount is referred to as the net
realisable value (NRV) of the inventories.

(iii) Inventories might be valued at their historical cost (ie the cost at
which they were originally bought).

(iv) Inventories might be valued at the amount it would cost to


replace them. This amount is referred to as the current
replacement cost of inventories.

9.4 ACCA SYLLABUS GUIDE OUTCOME 4:


Understand and apply the IASB requirements for valuing inventories

IAS 2 lays out the required accounting treatment for inventories under the historical
cost system. The major area of contention is the cost value of inventory to be
recorded. This is recognised as an asset of the enterprise until the related revenues
are recognised (i.e. the item is sold) at which point the inventory is recognised as an
expense (i.e. cost of sales). Part or all of the cost of inventories may also be
expensed if a write-down to net realisable value is necessary.

___________________________________________________________________________

114
9.5 ACCA SYLLABUS GUIDE OUTCOME 5
Recognize which costs should be included in valuing inventories.

9.5.1 Cost

The cost of inventories will consist of all the following costs: -

1. Purchase
2. Costs of conversion
3. Other costs incurred in bringing the inventories to their present location and
condition, e.g. carriage inwards

9.5.1.1 Costs of purchase

IAS 2 lists the following as comprising the costs of purchase of inventories.

(a) Purchase price; plus


(b) Import duties and other taxes; plus
(c) Transport, handling and any other cost directly attributable to the
acquisition of finished goods, services and materials; less
(d) Trade discounts, rebates and other similar amounts.

9.5.1.2 Costs of conversion

Costs of conversion of inventories consist of two main parts: -

(i) Costs directly related to the units of production, e.g. direct materials, direct
labour
(ii) Fixed and variable production overheads that are incurred in converting
materials into finished goods, allocated on a systematic basis.

Fixed production overheads are those indirect costs of production that remain
relatively constant regardless of the volume of production, e.g. the cost of factory
management and administration. Variable production overheads are those indirect
costs of production that vary directly, or nearly directly, with the volume of
production, e.g. indirect materials and labour. (IAS 2)

9.5.2 Net Realisable Value

The net realisable value of an item is essentially its net selling proceeds after all
costs have been deducted.

___________________________________________________________________________

115
It is calculated as:

$
Estimated selling price X
Less: estimated costs of completion (X)
Less: estimated selling and distribution costs (X)
X

As a general rule, assets should not be carried at amounts greater than those
expected to be realised from their sale or use. In the case of inventories this amount
could fall below cost when items are damaged or become obsolete, or where the
costs to completion have increased in order to make the sale.

Lecture Example 2

The closing inventory at cost of a company at 31 January 20X3 amounted to


$284,700.
The following items were included at cost in the total:

1. 400 coats, which had cost $80 each and normally sold for $150 each. Owing
to a defect in manufacture, they were all sold after the reporting date at 50%
of their normal price. Selling expenses amounted to 5% of the proceeds.

2. 800 skirts, which had cost $20 each. These too were found to be defective.
Remedial work in February 20X3 cost $5 per skirt, and selling expenses for
the batch totalled $800. They were sold for $28 each.

What should the inventory value be according to IAS 2 “Inventories” after


considering the above items?

A. $281,200
B. $282,800
C. $329,200
D. None of these

9.6 ACCA SYLLABUS GUIDE OUTCOME 6:


Understand the use of continuous and period end inventory records.

The quantity of inventories held at the year end is established by means of a


physical count of inventory in an annual counting exercise, or by a 'continuous'
inventory count.

___________________________________________________________________________

116
In simple cases, when a business holds easily counted and relatively small amounts
of inventory, quantities of inventories on hand at the reporting date can be
determined by physically counting them in an inventory count.
In more complicated cases, where a business holds considerable quantities of varied
inventory, an alternative approach to establishing quantities is to maintain
continuous inventory records. This means that a card is kept for every item of
inventory, showing receipts and issues from the stores, and a running total. A few
inventory items are counted each day to make sure their record cards are correct –
this is called a 'continuous' count because it is spread out over the year rather than
completed in one count at a designated time.

9.7 ACCA SYLLABUS GUIDE OUTCOME 7:


Calculate the value of closing inventory using FIFO (first in, first out) and
AVCO (Average Cost) – both periodic weighted average and continuous
weighted average

9.7.1 FIFO (first in, first out)

FIFO assumes that materials are issued out of inventory in the order in which they
were delivered into inventory, i.e. issues are priced at the cost of the earliest delivery
remaining in inventory

9.7.2 AVCO (average cost)

AVCO calculates a weighted average price for all units in inventory. Issues are
priced at this average cost, and the balance of inventory remaining would have the
same unit valuation.

A new weighted average price is calculated whenever a new delivery of materials


into store is received.

LIFO is no longer permitted under IAS 2.

Lecture Example 3

Boomerang Co had 200 units in inventory at 30 November 20X1 valued at $800.


During December it made the following purchases and sales.

2/12 Purchased 1,000 @ $5 each


5/12 Sold 700 @ $7.50 each
12/12 Purchased 800 @ $6.20 each
15/12 Purchased 300 @ $6.60 each
21/12 Sold 400 @ $8.00 each
28/12 Sold 500 @ $8.20 each

___________________________________________________________________________

117
Calculate the value of closing inventory using FIFO and AVCO.

How do we calculate the PERIODIC weighted average cost?

This method is only used if specifically mentioned in the exam question. Otherwise,
the cumulative weighted average method should be used.

Periodic weighted average = cost of opening inventory + total costs of receipts


units of opening inventory + total units received

Lecture Example 4

Using the information in lecture example 3, calculate the value of closing inventory at
the end of December using the periodic weighted average.

9.8 ACCA SYLLABUS GUIDE OUTCOME 8:


Understand the impact of accounting concepts on the valuation of inventory

The fundamental accounting assumption of accrual requires costs to be matched


with associated revenues. In order to achieve this, costs incurred for goods which
remain unsold at the year end must be carried forward in the statement of financial
position and matched against future revenues.

In valuing inventory, we also follow the prudence concept which states that a profit
cannot be anticipated before it is realised.

A. If inventory is expected to be sold at a profit:


(i) value at cost
(ii) do not anticipate profit.

B. If inventory is expected to be sold at a loss:


(i) value at net realisable value
(ii) do provide for the future loss.

___________________________________________________________________________

118
9.9 ACCA SYLLABUS GUIDE OUTCOME 9:
Identify the impact of inventory valuation methods on profit and on assets

Each method of valuation produces different costs both of closing inventories and
also of material issues. Since raw material costs affect the cost of production, and
the cost of production works through eventually into the cost of sales, it follows that
different methods of inventory valuation will provide different profit figures.

In times of rising prices, using FIFO method will mean the financial statements show
higher inventory values and higher profit.

9.10 ACCA SYLLABUS GUIDE OUTCOME 10:


Draft the disclosure note for inventory

The financial statements should disclose the following: -

 accounting policy for inventories


 carrying amount, generally classified as merchandise, supplies, materials,
work in progress, and finished goods. The classifications depend on what is
appropriate for the entity
 carrying amount of any inventories carried at fair value less costs to sell
 amount of any write-down of inventories recognised as an expense in the
period

Example of disclosure note

Note 1: Accounting Policies

Inventories

Inventories are valued at the lower of cost and net realizable value. Cost is
determined using first in, first out method. Net realizable value is the
estimated selling price in the ordinary course of business, less the costs
estimated to make the sale.

Note 10: Inventories

2011 2010

Raw Materials 15,000 17,000


Work in progress 10,000 13,000
Finished goods 25,000 15,000
50,000 45,000

___________________________________________________________________________

119
Lecture Example 5

Which of the following statements about the treatment of inventory and work
in progress in financial statements are correct?
1. Inventory should be valued at the lowest of cost, net realisable value and
replacement cost.
2. In valuing work in progress, materials costs, labour costs and variable and
fixed production overheads must be included.
3. Inventory items can be valued using either first in, first out (FIFO) or weighted
average cost.
4. A company’s financial statements must disclose the accounting policies used
in measuring inventories.

A. All four statements are correct


B. 1, 2 and 3 only
C. 2, 3 and 4 only
D. 1 and 4 only

Lecture Example 6

A company with an accounting date of 31 October carried out a physical check of


inventory on 4 November 20X3, leading to an inventory value at cost at this date of
$483,700.

Between 1 November 20X3 and 4 November 20X3 the following transactions took
place:
1. Goods costing $38,400 were received from suppliers.
2. Goods that had cost $14,800 were sold for $20,000.
3. A customer returned, in good condition, some goods which had been sold to
him in October for $600 and which had cost $400.
4. The company returned goods that had cost $1,800 in October to the supplier,
and received a credit note for them.

What figure should appear in the company’s financial statements at 31 October 20X3
for closing inventory, based on this information?

A. $458,700
B. $505,900
C. $508,700
D. $461,500

___________________________________________________________________________

120
__________________________

KEY NOTES
_________________

___________________________________________________________________________

121
1. Inventories are assets:

i. held for sale in the ordinary course of business;


ii. in the process of production for such sale; or
iii. in the form of materials or supplies to be consumed in the production process
or in the rendering of services.

2. It affects the financial statement in two ways:

i. Statement of financial position: it is included as a current asset


ii. Statement of profit or loss: opening and closing inventory have a direct impact
on cost of sales and therefore profits.

3. Accounting treatment: -

i. Opening Inventories

Dr Cost of sales (I/S)


Cr Inventories (SOFP)

ii. Closing Inventories

Dr Inventories (SOFP)
Cr Cost of sales (I/S)

4. Inventories should be measured at the lower of cost and net realisable value
for each separate item or group of items.

5. Cost: -

The cost of inventories will consist of all the following costs: -

1. Purchase
2. Costs of conversion
3. Other costs incurred in bringing the inventories to their present
location and condition, e.g. carriage inwards

___________________________________________________________________________

122
6. Net Realisable Value: -

It is calculated as:
$
Estimated selling price X
Less: estimated costs of completion (X)
Less: estimated selling and distribution costs (X)
X

7. FIFO: -

FIFO assumes that materials are issued out of inventory in the order in which they
were delivered into inventory, i.e. issues are priced at the cost of the earliest delivery
remaining in inventory

8. AVCO: -

AVCO calculates a weighted average price for all units in inventory. Issues are
priced at this average cost, and the balance of inventory remaining would have the
same unit valuation. A new weighted average price is calculated whenever a new
delivery of materials into store is received.

9. Profits in periods of rising prices: -

In times of rising prices, using FIFO method will mean the financial statements show
higher inventory values and higher profit.

___________________________________________________________________________

123
__________________________

QUESTION BANK
_________________

___________________________________________________________________________

124
1. According to IAS 2, inventory is valued under which basis?

A. Historic cost
B. Lower of cost and net realisable value

2. Which of the following are correct?

1. The value of inventory in the statement of financial position should be


as close as possible to net realisable value.
2. The valuation of finished goods inventory must include production
overheads.
3. Production overheads included in valuing inventory should be
calculated by reference to the company’s normal level of production
during the period.
4. In assessing net realisable value, inventory items must be considered
separately, or in groups of similar items, not by taking the inventory
value as a whole.

A. 1 and 2 only
B. 2 and 4 only
C. 1 and 3 only
D. 2, 3 and 4

3. The entries required to correctly reflect inventory and cost of sales in the
financial accounts for the first year of trading are:

A. Debit inventory - closing inventory Credit trading a/c - closing inventory


Debit trading a/c - opening inventory Credit inventory - opening
inventory
B. Debit trading a/c - closing inventory Credit inventory - closing inventory
Debit inventory - opening inventory Credit trading a/c - opening
inventory
C. Debit inventory - closing inventory Credit payables - closing inventory
D. Debit inventory - closing inventory Credit trading a/c - closing inventory

___________________________________________________________________________

125
4. The following information relates to Carnberwell’s year-end inventory of
finished goods: -

Direct costs of Production Expected selling and Expected


materials and labour overheads incurred distribution overheads Selling price
$ $ $ $
Inventory Category 1 2,470 2,100 480 5,800
Inventory Category 2 9,360 2,730 150 12,040
Inventory Category 3 1,450 850 190 2,560
13,280 5,680 820 20,400

What amount should finished goods inventory be stated in the company’s


statement of financial position?

5. In a period of rapid inflation, which method of valuing inventory issues will


give the lower gross profit figure?

A. FIFO
B. AVCO

6. Which of the following is not an acceptable basis for inventory valuation under
IAS 2?

A. Last in first out


B. Standard cost
C. Average cost
D. First in first out

7. For Morgan the direct cost of production of each unit of inventory is $46
(including carriage inwards of $11 and import duties of $1 on the raw
materials element). Production overheads amount to $15 per unit. Currently
the goods can only be sold if they are modified at a cost of $17 per unit. The
selling price of each modified unit is $80 and selling costs are estimated at
10% of selling price. At what value should each unmodified unit of inventory
be included in the statement of financial position?

8. The inventory counters of Crocodile Co inform you that there are 6,000 items
of product A, and 2,000 of product B, these cost $10 and $5 respectively.
They also tell you the following information:

Product A – 500 of these were found to be defective and would be sold at a


cut price of $8.
Product B – 100 of these were also to be sold for $4.50 with selling expenses
of $1.50 each.

___________________________________________________________________________

126
What figure should appear in Crocodile's statement of financial position for
inventory?
9. A company values its inventory using the first in, first out (FIFO) method. At 1
May 20X2 the company had 700 engines in inventory, valued at $190 each.
During the year ended 30 April 20X3 the following transactions took place:

20X2
1 July Purchased 500 engines at $220 each
1 November Sold 400 engines for $160,000

20X3
1 February Purchased 300 engines at $230 each
15 April Sold 250 engines for $125,000

What is the value of the company’s closing inventory of engines at 30 April


20X3?

A. $188,500
B. $195,500
C. $166,000
D. None of these figures

___________________________________________________________________________

127
__________________________

ANSWER BANK
_________________

___________________________________________________________________________

128
1. B

2. D

3. D

4. $18,760

Cost NRV Lower of


Cost & NRV

Inventory 1 2,470 + 2,100 5,800 – 480


= 4,570 = 5,320 4,570
Inventory 2 9,360 + 2,730 12,040 – 150
= 12,090 = 11,890 11,890
Inventory 3 1,450 + 850 2,560 – 190
= 2,300 = 2,370 2,300
18,760

5. B

6. A

7. $ 55

Cost NRV Lower of


Cost & NRV

46 + 15 80 – 17 – 8
= 61 = 55 55

Cost = Total Production Costs

NRV = SP – Modification Costs – Selling Costs

8. $68,800

500 x 8 = 4,000
5,500 x 10 = 55,000
6,000 59,000

100 x 3 = 300
___________________________________________________________________________

129
1,900 x 5 = 9,500
2,000 9,800
68,800
9. A

50 @ $190 9,500
500 @ $220 110,000
300 @ $230 69,000
188,500

___________________________________________________________________________

130
Chapter 10
Tangible Non-Current Assets
and Depreciation

10.1 ACCA SYLLABUS GUIDE OUTCOME 1:


Define non-current assets

Non-current assets - all assets other than current assets shall be classified as non-
current assets. They include both tangible and intangible assets.

The accounting treatment of tangible non-current assets is covered by IAS 16:


Property, Plant and Equipment.

10.2 ACCA SYLLABUS GUIDE OUTCOME 2:


Recognise the difference between current and non-current assets

Current assets are assets: -


1. realized (sold/consumed) in entities’ normal operating cycle
2. which are held for trading
3. which include cash and cash equivalent
4. are expected to realize within 12 months after the end of the reporting period

Difference between a current asset and a non-current asset: -

Current Assets Non-Current Assets

1. realized within normal operating 1. not realized in normal operating


cycle of entity cycle
2. intended for sale or consumption 2. intended for use over a long period
of time
3. used for trading purposes 3. used for investment and productive
purposes
4. realized within 12 months 4. held for more than 12 months
5. e.g. inventory, cash and bank 5. e.g. plant and machinery,
balance, raw material, receivables. equipment, land and buildings, office
furniture.

___________________________________________________________________________

131
10.3 ACCA SYLLABUS GUIDE OUTCOMES 3 AND 4:
Explain the difference between capital and revenue items
Classify expenditure as capital or revenue expenditure

Transactions

Capital item Revenue item


(SOFP) (I/S)

Capital Capital Revenue Revenue


expenditure receipt expenditure receipt

Nature of Meaning
Transaction

Capital expenditure i. it increases the value of non-current assets


ii. it improves the earning capacity of an asset
iii. e.g. purchase of computers, vehicles, building,
land, plant and machinery;
stamp duty, registration fees, solicitor’s fees,
architect’s fees, installation charges;
fitting of air conditioner in vehicles
Revenue i. it is incurred to maintain existing capacity of
expenditure asset
ii. regular expenditure
iii. e.g. repairs and maintenance to machinery,
electricity cost for machinery, spare parts for
machinery
Capital receipt i. it is income which is not earned out of the
regular operations of an entity, i.e. not realized
by the sale of the merchandise of the entity
ii. it is a receipt earned when an item of capital

___________________________________________________________________________

132
expenditure is sold
iii. it decreases the value of non-current assets

Revenue receipt i. it is a regular receipt/income


ii. it decreases current assets
iii. it is a result of the sale of the entity’s
merchandise and other revenue items such as
rent received or commission received

Capital expenditure results in the appearance of a non-current asset in the


statement of financial position of the business.

Revenue expenditure results in an expense in the statement of profit or loss.

Lecture Example 1: -

Arthur sets up his demolition business from scratch on 1 January 20X1. During the
year he:

(a) Buys a warehouse


(b) Pays legal expenses on the purchase
(c) Buys three wrecking machines
(d) Rents office premises
(e) Builds an extension to the warehouse
(f) Pays wages
(g) Repairs the warehouse roof
(h) Writes off a damaged machine

Which items represent capital expenditure?

A (a), (b), (c) and (e)


B (a), (b), (c), (e), (g) and (h)
C (a) and (c)
D All of them

10.4 ACCA SYLLABUS GUIDE OUTCOME 5:


Prepare ledger entries to record the acquisition of non-current assets

When a non-current asset is acquired, the double-entry is: -

Dr Non-Current Asset
Cr Cash/Payables

___________________________________________________________________________

133
Tangible non-current assets should initially be recorded at cost.

The cost of an asset includes: -


i. Purchase price – after deducting trade discounts and rebates and adding
duties and non-refundable taxes
ii. Cost directly attributable to bring the asset to its location and to make it
available for its intended use.

These include:
a. Initial delivery and handling costs
b. Installation and assembly costs
c. Costs of testing whether the asset is working properly
d. Professional fees

The following costs may not be included:


a) The cost of maintenance contracts
b) Administration and general overhead costs
c) Staff training costs

iii. Dismantling cost – cost of removing old asset from its place in order to put in
the new one

Lecture Example 2: -

On 8th June 20X8, an entity bought a machine.


The invoice showed that:
$
Cost of machine 50,000
Delivery costs 600
Installation costs 2,500
One-year maintenance contract 5,000

At what amount should the machine be capitalized in the entity’s records?

______________

10.5 ACCA SYLLABUS GUIDE OUTCOME 6:


Understand and explain the purpose of depreciation

Where assets held by an enterprise have a limited useful life, it is necessary to


apportion the value of an asset used in a period against the revenue it has helped to

___________________________________________________________________________

134
create. Therefore, with the exception of land held on freehold or very long leasehold,
every non-current asset has to be depreciated.

A charge is made in the statement of profit or loss to reflect the use that is made of
the asset by the business. This charge is called depreciation. The need to depreciate
non-current assets arises from the accrual assumption. If money is spent on an
asset, then the amount must be charged against profits.

Some key terms are: -

1. Depreciation: - the allocation of the depreciable amount of an asset over its


estimated useful life.

2. Useful life: - the period over which a depreciable asset is expected to be


used by the enterprise; or the number of production or similar units expected
to be obtained from the asset by the enterprise.

3. Depreciable amount: - cost/revalued amount – residual value

4. Residual value: - the amount the asset is expected to be sold for at the end
of its useful life. It is also known as scrap value

10.6 ACCA SYLLABUS GUIDE OUTCOMES 7 AND 8:


Calculate the charge for depreciation using straight line and reducing balance
methods.
Identify the circumstances where different methods of depreciation would be
appropriate.

There are two main methods for calculating depreciation:


(a) Straight line method
(b) Reducing balance method

10.6.1 Straight line method

The depreciation charge is the same every year.

Formula: - Cost of asset – residual value


Expected useful life of asset

OR

(Cost – Residual value) × %

This method is suitable for assets which are used up evenly over their useful life, e.g.
fixtures and fittings in the accounts department.
___________________________________________________________________________

135
Lecture Example 3: -

A non-current asset costing $60,000 has an estimated life of 5 years and a residual
value of $7,000.

Required: -

(a) Calculate the annual depreciation charge.


(b) Calculate the cost, accumulated depreciation and net book value (NBV) for
each year of the asset’s life.

(a)

(b)
Year Cost Accumulated NBV
Depn
$ $ $
1
2
3
4
5

10.6.2 Reducing balance method

This method is suitable for those assets which generate more revenue in
earlier years than in later years; for example machinery in a factory where
productivity falls as the machine gets older.

Under this method the depreciation charge will be higher in the earlier years
and reduce over time.

Formula: - Depreciation rate (% ) × Net Book Value (NBV)

Net book value (NBV) / Carrying value = cost – accumulated depreciation to


date

___________________________________________________________________________

136
This method ignores residual value, since the NBV under this method will
never reach zero.

Lecture Example 4: -

A business buys a lorry costing $17,000. After 5 years, it is expected to be sold for
scrap for $2,000. The depreciation rate is 35% on a reducing balance basis.

Required
Calculate depreciation expense, accumulated depreciation and net book value
of the machine for these five years using the reducing balance basis.

Year Cost / Depreciation Depreciation Accumulated NBV c/d


NBVb/d Rate Expense Depreciation
$ % $ $ $
1
2
3
4
5

10.6.3 Change in method of depreciation

It is up to the business to decide which method of depreciation to apply to its non-


current assets. The chosen method of depreciation should be applied consistently
from year to year. This is an instance of the fundamental accounting assumption of
consistency.

The depreciation method has to be reviewed. If there are any changes in the
expected pattern of use of the asset, then the method used should be changed. In
such cases, the remaining net book value is depreciated under the new method, i.e.
only current and future periods are affected. The change is prospective.

Lecture Example 5: -

Clog Co purchased an asset for $100,000 on 1 st January 20x1. It had an estimated


useful life of 5 years and it was depreciated using the reducing balance method at a
rate of 40%. On 1st January 20x3, it was decided to change the method to straight
line.

___________________________________________________________________________

137
Required: -
Show the depreciation charge for each year of the asset’s life.

Year Depreciation Accumulated Net Book Value


charge depreciation
$ $ $
20x1
20x2
20x3
20x4
20x5

10.7 ACCA SYLLABUS GUIDE OUTCOMES 9 AND 10:


Illustrate how depreciation expense and accumulated depreciation are
recorded in ledger accounts
Record depreciation in the statement of profit or loss and statement of
financial position

Depreciation has a dual effect which needs to be accounted for:


(a) It reduces the value of the asset in the statement of financial position.
(b) It is an expense in the statement of profit or loss.

The double-entry for depreciation is:

Dr Depreciation expense (I/S)


Cr Accumulated Depreciation (SOFP)

with the depreciation charge for the period.

Lecture Example 6: -

Required

Using the information in Lecture example 4, show:


(a) The journal entry which would have been written at the end of the first
year.
(b) The treatment of depreciation for all years in the relevant ledger
accounts.

___________________________________________________________________________

138
(c) The relevant statement of profit or loss and statement of financial
position extracts for each year.

10.8 ACCA SYLLABUS GUIDE OUTCOME 11:


Prepare ledger entries to record the disposal of non-current assets

When a non-current asset is sold, there is likely to be a profit or loss on disposal.


This is the difference between the net sale price of the asset and its net book value
at the time of disposal.
If:

Sales proceeds > NBV → profit on disposal


Sales proceeds < NBV → loss on disposal

Accounting Treatment: -

(1) Remove the cost of the asset:


Dr Disposal account
Cr Non-current asset

(2) Remove the accumulated depreciation charged to date:


Dr Accumulated depreciation
Cr Disposal account

(3) Account for the sales proceeds:


Dr Cash
Cr Disposal account

(4) Balance off disposal account to find the profit or loss on disposal.

A profit on disposal is shown in the statement of profit or loss as sundry income, a


loss as an expense in the statement of profit or loss.

Lecture Example 7: -

A company buys a machine on 31 August 20X0 for $22,000. It has an expected life
of seven years and an estimated residual value of $1,000. On 30 June 20X4 the
machine is disposed of for $9,000. The company's year end is 31 December. Its
accounting policy is to charge depreciation using the straight line method with a
proportionate charge in the years of acquisition and disposal.

Required:

___________________________________________________________________________

139
(a) Calculate the profit or loss on disposal of the machine which will appear
in the statement of profit or loss for the year ended 31 December 20X4
(b) Prepare the ledger accounts to show how the disposal would be
accounted for.

Lecture Example 8: -

The plant and equipment account in the records of a company for the year ended 31
December 20X6 is shown below:

Plant and Equipment - cost


20X6 $ 20X6 $

1 Jan Balance 960,000 30 Sept Transfer to disposal a/c 84,000


1 July Cash 48,000 31 Dec Balance 924,000
1,008,000 1,008,000

The company’s policy is to charge depreciation on the straight line basis at 20% per
year, with proportionate depreciation in the years of purchase and sale.
What should be the charge for depreciation in the company’s statement of profit or
loss for the year ended 31 December 20X6?

A $184,800
B $192,600
C $191,400
D $184,200

10.9 ACCA SYLLABUS GUIDE OUTCOME 12:


Calculate and record profits or losses on disposal of non-current assets in the
statement of profit or loss including part exchange transactions

Instead of receiving sales proceeds as cash, a part exchange allowance could be


offered against the cost of a replacement asset:

Dr New asset cost


Cr Disposal account

IAS 16 states that the cost of an item obtained through part exchange is the fair
value of the asset received.

The part exchange allowance takes the place of proceeds in the disposals account.

___________________________________________________________________________

140
Lecture Example 9:

Assume in Lecture example 7 that instead of cash proceeds of $9,000, there is a


part exchange allowance of $3,000 on a replacement machine costing $20,000.

Required
(a) Calculate the profit or loss on disposal of the machine.
(b) Calculate the amount of cash paid for the new machine.
(c) Complete the ledger accounts to show both the disposal and the
acquisition.

10.10 ACCA SYLLABUS GUIDE OUTCOME 13:


Record the revaluation of a non-current asset in ledger accounts
Calculate the profit or loss on disposal of a revalued asset
Calculate depreciation on a revalued non-current asset including the transfer
of excess depreciation between the revaluation surplus and retained earnings.

IAS 16 allows entities the choice of two valuation models for its non-current assets –
the cost model or the revaluation model.

Each model needs to be applied consistently to all non-current assets of the same
‘class’. A class of assets is a grouping of assets that have a similar nature or function
within the business. For example, properties would typically be one class of assets,
and plant and equipment another. Additionally, if the revaluation model is chosen,
the revaluations need to be kept up to date, although IAS 16 is not specific as to how
often assets need to be revalued.

When the revaluation model is used, assets are carried at their fair value, defined as
‘the amount for which an asset could be exchanged between knowledgeable, willing
parties in an arm’s length transaction’.

When a revalued asset is disposed of, any revaluation surplus may be transferred
directly to retained earnings, or it may be left in equity under the heading revaluation
surplus. The transfer to retained earnings should not be made through the statement
of profit or loss

IAS 16 allows (but does not require) entities to make a transfer of the ‘excess depreciation’
(the extra depreciation which results due to the increased value of the asset) from the
revaluation surplus directly to retained earnings.

___________________________________________________________________________

141
Accounting treatment:

(1) Adjust cost account to revalued amount.


(2) Remove accumulated depreciation charged on the asset to date.
(3) Put the balance to the revaluation surplus.

The required double-entry is:

Dr Non-current asset cost


Dr Accumulated Depreciation
Cr Revaluation surplus

Lecture Example 10: -

A property was purchased on 1 January 2000 for $2m (estimated depreciable


amount $1m – useful economic life 50 years). On 1 January 2005 the carrying value
of the property was $1.9m.

The property was revalued to $2.8m on 1 January 2005 (estimated depreciable


amount $1.35m – the estimated useful economic life was unchanged).

Required:

Show the treatment of the revaluation surplus and compute the revised annual
depreciation charge.

10.11 ACCA SYLLABUS GUIDE OUTCOME 14:

Change in the useful life of an asset

The useful life of an item of property, plant and equipment should be reviewed at
least every financial year-end and, if expectations are significantly different from
previous estimates, the depreciation charge for current and future periods should be
revised.

___________________________________________________________________________

142
This is achieved by writing the net book value off over the asset's revised remaining
useful life.

NBV – residual value


Revised useful life

Lecture Example 11:

Clog Co purchased an asset for $100,000 on 1 st January 20X1. It had an estimated


useful life of 5 years and it was depreciated using the straight line method. On 1 st
January 20X3, it was decided to revise the total useful life to 4 years.

Required: -
Show the depreciation charge for each year of the asset’s life.

Year Depreciation Accumulated Net Book Value


charge depreciation
$ $ $
20X1
20X2
20X3
20X4
Lecture Example 12: -

On 1 January 20X0 Goblin bought a machine for $63,000. It was estimated that the
machine's useful life would be 7 years and its residual value $7,000. Two years later
the useful life was revised to four remaining years with a residual value of $7,000. At
31 December 20X4 the machine was sold for $30,000. No depreciation was
provided in 20X4.

What is the profit or loss on disposal?

A $3,000 loss
B $7,000 profit
C $3,000 profit
D $7,000 loss

10.12 ACCA SYLLABUS GUIDE OUTCOME 15:


Draft the disclosure note for tangible non-current assets

For each class of property, plant, and equipment, disclose:

 basis for measuring carrying amount


 depreciation method(s) used
___________________________________________________________________________

143
 useful lives or depreciation rates
 gross carrying amount and accumulated depreciation and impairment losses
 reconciliation of the carrying amount at the beginning and the end of the
period, showing:

o additions
o disposals
o acquisitions through business combinations
o revaluation increases or decreases
o impairment losses
o reversals of impairment losses
o depreciation
o net foreign exchange differences on translation
o other movements

If property, plant, and equipment is stated at revalued amounts, certain additional


disclosures are required:

 the effective date of the revaluation


 whether an independent valuer was involved
 the methods and significant assumptions used in estimating fair values
 for each revalued class of property, the carrying amount that would have been
recognised had the assets been carried under the cost model
 the revaluation surplus, including changes during the period and any
restrictions on the distribution of the balance to shareholders

Land Machinery Office Total


and Equipment
Buildings
$ $ $ $
Cost or valuation
At 1 January 2010 50,000 10,000 8,000 68,000
Revaluation surplus 12,000 2,000 2,000 16,000
Additions in year 4,000 4,000 - 8,000
Disposals in year (1,000) (1,000) - (2,000)
At 31 December 2010 65,000 15,000 10,000 90,000

Depreciation
At 1 January 2010 16,000 6,000 4,000 26,000
Charge for year 4,000 3,000 2,000 9,000
Eliminated on disposals (500) (500) - (1,000)
At 31 December 2010 19,500 8,500 6,000 34,000

___________________________________________________________________________

144
Carrying Amount
At 31 December 2010 45,500 6,500 4,000 56,000
At 1 January 2010 34,000 4,000 4,000 42,000

10.12.1 The Asset Register

An asset register is used to record all non-current assets and is an internal check on
the accuracy of the nominal ledger. For example, an asset may have been scrapped
and the asset register updated, but the asset has not yet been written off in the
accounting records.

In an asset register, the following details about each non-current asset are found: -
 Purchase date
 Cost depreciation method
 Estimated useful life
 Carrying amount
 Description of asset
 Location of asset
 Internal reference number
 Manufacturer’s seriel number

Lecture Example 13

An organization’s asset register shows a carrying value of $135,600. The non-


current asset account in the nominal ledger shows a carrying value of $125,600.

Which of the following disposals, if not deducted from the asset register could
account for the difference?

A. Asset A with disposal proceeds of $15,000 and a profit on disposal of


$5,000
B. Asset B with disposal proceeds of $15,000 and a carrying value of
$5,000
C. Asset C with disposal proceeds of $15,000 and a loss on disposal of
$5,000
D. Asset D with disposal proceeds of $5,000 and a carrying value of
$5,000

___________________________________________________________________________

145
__________________________

KEY NOTES
_________________

___________________________________________________________________________

146
1. Non-current assets

All assets other than current assets shall be classified as non-current assets.
They include both tangible and intangible assets.

The accounting treatment of tangible non-current assets is covered by IAS 16:


Property, Plant and Equipment.

2. Current assets

Current assets are assets: -


1. realized (sold/consumed) in entities’ normal operating cycle
2. which are held for trading
3. which include cash and cash equivalent
4. are expected to realize within 12 months after the end of the reporting
period

3. Capital vs. revenue expenditure

Capital i. it increases the value of non-current assets


expenditure ii. it improves the earning capacity of an asset
iii. e.g. purchase of computers, vehicles, building,
land, plant and machinery;
stamp duty, registration fees, solicitor’s fees,
architect’s fees, installation charges;
fitting of air conditioner in vehicles

Revenue 1. it is incurred to maintain existing capacity of


expenditure asset
2. regular expenditure
3. e.g. repairs and maintenance to machinery,
electricity cost for machinery, spare parts for
machinery

Capital expenditure results in the appearance of a non-current asset in the


statement of financial position of the business. Revenue expenditure results
in an expense in the statement of profit or loss.

4. Cost of a non-current asset

___________________________________________________________________________

147
Tangible non-current assets should initially be recorded at cost.

The cost of an asset includes: -


i. Purchase price
ii. Cost directly attributable to bring the asset to its location and to
make it available for its intended use.
iii. Dismantling cost
5. Depreciation

Depreciation is the allocation of the depreciable amount of an asset over its


estimated useful life. Where assets held by an enterprise have a limited
useful life, it is necessary to apportion the value of an asset used in a period
against the revenue it has helped to create. Therefore, with the exception of
land held on freehold or very long leasehold, every non-current asset has to
be depreciated.

A charge is made in the statement of profit or loss to reflect the use of the
asset by the business

There are two main methods for calculating depreciation:


a. Straight line method
b. Reducing balance method

5.1 Straight line method

The depreciation charge is the same every year.

Formula: - Cost of asset – residual value


Expected useful life of asset

OR

(Cost – Residual value) × %

5.2 Reducing balance method

Under this method the depreciation charge will be higher in the earlier years
and reduce over time.

Formula: - Depreciation rate (%) × Net Book Value (NBV)

Net book value (NBV) = cost – accumulated depreciation to date

This method ignores residual value.

6. Change in the method of depreciation


___________________________________________________________________________

148
If there are any changes in the expected pattern of use of the asset, then the
method of depreciation used should be changed. In such cases, the
remaining net book value is depreciated under the new method, i.e. only
current and future periods are affected.

7. Accounting for depreciation

The double-entry for depreciation is:

Dr Depreciation expense (I/S)


Cr Accumulated Depreciation (SOFP)

with the depreciation charge for the period.

8. Disposal of non-current assets

Accounting Treatment: -

(1) Remove the cost of the asset:


Dr Disposal account
Cr Non-current asset

(2) Remove the accumulated depreciation charged to date:


Dr Accumulated depreciation
Cr Disposal account

(3) Account for the sales proceeds:


Dr Cash
Cr Disposal account

(4) Balance off disposal account to find the profit or loss on disposal.

A profit on disposal is shown in the statement of profit or loss as sundry


income, a loss as an expense in the statement of profit or loss.

9. Part-exchange of non-current assets

Instead of receiving sales proceeds as cash, a part exchange allowance could


be offered against the cost of a replacement asset:

Dr New asset cost


Cr Disposal account

___________________________________________________________________________

149
The part exchange allowance takes the place of proceeds in the disposals
account.

10. Revaluation of non-current assets

Accounting treatment:

a. Adjust cost account to revalued amount.


b. Remove accumulated depreciation charged on the asset to date.
c. Put the balance to the revaluation surplus.

The required double-entry is:

Dr Non-current asset cost


Dr Accumulated Depreciation
Cr Revaluation surplus

11. Change in the useful life of an asset

The useful life of an item of property, plant and equipment should be reviewed
at least every financial year-end and, if expectations are significantly different
from previous estimates, the depreciation charge for current and future
periods should be revised.

This is achieved by writing the net book value off over the asset's revised
remaining useful life.

NBV – residual value


Revised useful life

12. Asset Register

An asset register is used to record all non-current assets and is an internal


check on the accuracy of the nominal ledger.

___________________________________________________________________________

150
___________________

QUESTION BANK
___________________

___________________________________________________________________________

151
1. A business sells a non-current asset for $55,000. The asset originally cost
$100,000 and accumulated depreciation is $45,000. What is the profit or loss
on disposal?

A. $10,000 profit
B. No gain or loss
C. $10,000 loss

2. Octopus bought a car on 1 January 20X0 for $20,000 and decided to


depreciate it at 30% per annum on a reducing balance basis. It was disposed
of during the year ended 31 December 20X2 for $12,000. Octopus does not
charge depreciation in the year of disposal.

What is the net effect on the statement of profit or loss for the year ended 31
December 20X2?

A. Increase of $2,200
B. Decease of $2,200
C. Increase of $12,000
D. Decrease of $12,000

3. Demolition Co purchases a machine for $15,000. After incurring transportation


costs of $1,300 and spending $2,500 on installing the machine the company
are disappointed when it breaks down and costs $600 to repair. Depreciation
is charged at 10% per annum with a full year's charge in the year of
acquisition.

What is the net book value of the machine that will be shown in Demolition's
statement of financial position at the year end? $___________

4. A company buys a machine on 31 August 20X0 for $22,000. It has an


expected life of seven years and an estimated residual value of $1,000. On 30
June 20X4 the machine is disposed of for $9,000. The company's year end is
31 December. Its accounting policy is to charge depreciation using the
straight line method with a proportionate charge in the years of acquisition
and disposal.

What is the profit or loss on disposal of the machine which will appear in the
statement of profit or loss for the year ended 31 December 20X4?
$_________

___________________________________________________________________________

152
5. Vernon Vinyl purchased some new equipment on 1 April 20X1 for his mobile
disco for $6,000. The estimated scrap value of the new equipment in 5 years'
time is estimated to be $400. Vernon charges depreciation on the straight line
basis, with a proportionate charge in the period of acquisition.

What should the depreciation charge for the plant be in Vernon's accounting
period of twelve months to 30 September 20X1? $ _________

6. The NBV of B's property, plant and equipment is $51,000 at 1 January 20X5
and $100,000 at 31 December 20X5. A motor vehicle costing $20,000 was
purchased during the year and land was revalued by $43,000.

What is the depreciation expense for the year? $_____________

7. The plant and equipment account in the records of a company for the year
ended 31 December 20X6 is shown below:

Plant and Equipment - cost


20X6 $ 20X6 $

1 Jan Balance 960,000 30 Sept Transfer to disposal a/c 84,000


1 July Cash 48,000 31 Dec Balance 924,000
1,008,000 1,008,000

The company’s policy is to charge depreciation on the straight line basis at


20% per year, with proportionate depreciation in the years of purchase and
sale.

What should be the charge for depreciation in the company’s statement of


profit or loss for the year ended 31 December 20X6?

A. $184,800
B. $192,600
C. $191,400
D. $184,200

___________________________________________________________________________

153
_________________

ANSWER BANK
_________________

___________________________________________________________________________

154
1. B - Net book value is $55,000 ($100,000 – $45,000). So the proceeds are the
same as NBV and so there is no gain or loss.

2. A
$
31.12.X0 - NBV 14,000
31.12.X1 - NBV 9,800

Proceeds 12,000
NBV (9,800)
Profit 2,200

3. $16,920 $
Cost (15,000 + 1,300 + 2,500) 18,800
Depreciation (10% x 18,800) (1,880)
NBV 16,920

4. $1,500 loss on disposal


The depreciable amount is $(22,000 – 1,000) = $21,000. This is to be written
off over seven years (or 84 months). The monthly depreciation charge is
therefore $250.
$
Cost of asset 22,000
Accumulated depreciation (46 months × $250) 11,500
Net book value at date of disposal 10,500
Proceeds on disposal 9,000
Loss on disposal 1,500

5. $560 Cost of equipment


6,000 − 400 = $5,600
Depreciation charge for year $1,120
This year 6/12 = $560

6. $14,000

___________________________________________________________________________

155
PPE - NBV
Balance b/d 51,000 Depreciation expense 14,000
Bank 20,000
Revaluation Surplus - Land 43,000
Balance c/d 100,000

114,000 114,000

7. B

$960,000 - $84,000 = $876,000 x 20% = 175,200


$48,000 x 20% x 6/12 = 4,800
$84,000 x 20% x 9/12 = 12,600
192,600

___________________________________________________________________________

156
Chapter 11
IAS 38:- Intangible Assets

11.1 ACCA SYLLABUS GUIDE OUTCOME 1:


Recognise the difference between tangible and intangible non-current assets

As stated in Chapter 10, tangible non-current assets are defined as those which:
a. are held for use in the production or supply of goods or services for
administrative purposes; and
b. are expected to be used during more than one period.

An intangible non-current asset is an identifiable, non-monetary asset without


physical substance.

11.2 ACCA SYLLABUS GUIDE OUTCOME 2:


Identify types of intangible assets

Examples of intangible assets are: -


 Development expenditure
 Goodwill
 Concessions, patents, licences, trade marks, copy rights
 Computer software

Paper F3 only requires the accounting treatment of research and development


expenditure

11.3 ACCA SYLLABUS GUIDE OUTCOME 3:


Identify the definition and treatment of “research costs” and “development
costs” in accordance with International Financial Reporting Standards

___________________________________________________________________________

157
11.3.1 Research and Development Expenditure

Many businesses in the commercial world spend vast amounts of money, on an


annual basis, on the research and development of products and services. These
entities, including pharmaceutical and motor companies, do this with the intention of
developing a product or service that will, in future periods, provide significant
amounts of income for years to come.

11.3.2 Research and Development – Definitions

Research is original and planned investigation undertaken with the prospect of


gaining new scientific or technical knowledge and understanding.

An example of research could be a company in the pharmaceuticals industry


undertaking activities or tests aimed at obtaining new knowledge to develop a new
vaccine. The company is researching the unknown, and therefore, at this early
stage, no future economic benefit can be expected to flow to the entity.

Development is the application of research findings or other knowledge to a plan or


design for the production of new or substantially improved materials, devices,
products, processes, systems, or services, before the start of commercial production
or use.

An example of development is a car manufacturer undertaking the design,


construction, and testing of a pre-production model.

11.3.3 Accounting Treatment of Research and Development

IAS 38, Intangible Assets, separates a research and development project into a
research phase and a development phase.

Research phase

It is impossible to demonstrate whether or not a product or service at the research


stage will generate any probable future economic benefit. As a result, IAS 38 states
that all expenditure incurred at the research stage should be written off to the
statement of profit or loss as an expense when incurred, and will never be
capitalised as an intangible asset.

Development phase

Under IAS 38, an intangible asset must demonstrate all of the following criteria:
 Probable future economic benefits
 Intention to complete and use or sell the asset
 Resources (technical, financial and other resources) are adequate and
available to complete and use the asset
___________________________________________________________________________

158
 Ability to use or sell the asset
 Technical feasibility of completing the intangible asset (so that it will be
available for use or sale)
 Expenditure can be measured reliably

If any of the recognition criteria are not met then the expenditure must be charged to
the statement of profit or loss as incurred.

Note that if all the recognition criteria have been met, capitalisation must take place:

Dr Intangible non-current assets (SOFP)


Cr Bank/Payables

11.4 ACCA SYLLABUS GUIDE OUTCOME 4:


Calculate amounts to be capitalised as development expenditure or to be
expensed from given information.

Lecture Example 1

A company, Clarke Ltd, incurs research costs, during one year, amounting to
$125,000, and development costs of $490,000. The accountant informs you that the
recognition criteria (as prescribed by IAS 38) have been met. How should these
costs be accounted for in the financial statements?

Lecture Example 2

Medica is a company producing medicinal drugs. During the year ended 31


December 20X7, it incurred the following costs:

a) $15,000 on salaries for market research staff


b) $250,000 to purchase a machine to manufacture a new drug. It has an
estimated useful life of 10 years.
c) $40,000 on salaries relating to the design and manufacture of this new drug.

Required:
How should each of the above items be shown in the financial statements of
Medica for the year ended 31 December 20X7?

11.5 ACCA SYLLABUS GUIDE OUTCOME 5:


Explain the purpose of amortisation

Treatment of Capitalised Development Costs

___________________________________________________________________________

159
Once development costs have been capitalised, the asset should be amortised in
accordance with the accruals concept over its finite life.

What is amortization?

A tangible non-current asset, e.g. machinery, is capitalised and then depreciated


over its useful life. Similarly, the cost of the development expenditure should be
amortised over the useful life. Therefore, the cost of the development expenditure is
matched against the revenue it produces.

Amortisation must only begin when the asset is available for use (hence matching
the income and expenditure to the period in which it relates). It is an expense in the
statement of profit or loss: -

Dr Amortisation expense (I/S)


Cr Accumulated amortization (SOFP)

Each development project must be reviewed at the end of each accounting period to
ensure that the recognition criteria are still met. If the criteria are no longer met, then
the previously capitalised costs must be written off to the statement of profit or loss
immediately.

If the intangible asset is considered to have an indefinite useful life, it should not be
amortised but should be subjected to an annual impairment review, i.e. check wehter
there has been a fall in the value of the intangible asset.

11.6 ACCA SYLLABUS GUIDE OUTCOME 6:


Calculate and account for the charge for amortisation

Lecture Example 3

A company manufacturing aircraft engages in a number of research and


development projects.

At 1 January 20X6 the company’s records showed total capitalised development


costs of $18,000,000 made up as follows:
$
Project A17 14,000,000
This project was completed in 20X5 at a total cost of
$16,000,000, and is being amortised over 8 years on
the straight line basis, beginning on 1 January 20X5.

___________________________________________________________________________

160
Project J9 4,000,000
This project began in 20X4 and the $4m balance
represents expenditure qualifying for capitalisation
to 31 December 20X5
Project J9 is due to be completed in 20X9
––––––––
18,000,000
––––––––

During the year ended 31 December 20X6 the following further expenditure was
incurred:

Project J9
Further expenditure qualifying for capitalisation $1,500,000

Project A20
Investigation into new materials for aircraft construction $3,000,000

Required:
Calculate the amounts for research and development to be included in the
company’s statement of profit or loss and statement of financial position for
the year ended 31 December 20X6.

11.7 ACCA SYLLABUS GUIDE OUTCOME 7:


Draft the disclosure note for intangible non-current assets

For each class of intangible asset, disclose

 useful life or amortisation rate


 amortisation method
 gross carrying amount
 accumulated amortisation and impairment losses
 line items in the statement of profit or loss in which amortisation is included
 reconciliation of the carrying amount at the beginning and the end of the
period showing:
o additions (business combinations separately)
o assets held for sale
o retirements and other disposals
o revaluations
o impairments
o reversals of impairments
o amortisation
o foreign exchange differences
o other changes

___________________________________________________________________________

161
 basis for determining that an intangible has an indefinite life
 description and carrying amount of individually material intangible assets
 certain special disclosures about intangible assets acquired by way of
government grants
 information about intangible assets whose title is restricted
 contractual commitments to acquire intangible assets

Additional disclosures are required about:

 intangible assets carried at revalued amounts


 the amount of research and development expenditure recognised as an
expense in the current period

Development expenditure
$
Net book value at 1 April 20X0 X
Additions X
Amortisation charge (X)
Disposals (X)
Net book value at 31 March 20X1 X

At 31 March 20X0
Cost X
Accumulated amortization (X)
Net book value X

At 31 March 20X1
Cost X
Accumulated amortisation (X)
Net book value X

Lecture Example 4

Which TWO of the following items must be disclosed in the note to the
financial statements for intangible assets?

(1) The useful lives of intangible assets capitalised in the financial statements
(2) A description of the development projects that have been undertaken during the
period
(3) A list of all intangible assets purchased or developed in the period
(4) Impairment losses written off intangible assets during the period

___________________________________________________________________________

162
A. 1 and 4
B. 2 and 3
C. 3 and 4
D. 1 and 2

__________________________

KEY NOTES
________________

___________________________________________________________________________

163
Intangible Non-Current Assets

___________________________________________________________________________

164
non-monetary asset without physical substance

Research Development

Application of
the entity acquires research findings
new scientific or
technical knowledge

 Must capitalize if all the


recognition criteria (PIRATE)
are met:
an expense in the
statement of o Probable future economic
profit or loss benefits
o Intention to complete and use/
sell the asset
o Resources are adequate and
available to complete and use
the asset
o Ability to use or sell the asset
o Technical feasibility of
completing the intangible
asset
o Expenditure can be measured
reliably

 Start amortization when


commercial production begins
 Review annually to ensure
criteria are still met – if not,
an expense in the I/S.


 expense

___________________________________________________________________________

165
_____________________________

QUESTION BANK
___________________

___________________________________________________________________________

166
1. Which of the following statements about research and development
expenditure are correct?

1) Research expenditure, other than capital expenditure on research


facilities, should be recognised as an expense as incurred.
2) In deciding whether development expenditure qualifies to be recognised
as an asset, it is necessary to consider whether there will be adequate
finance available to complete the project.
3) Development expenditure recognised as an asset must be amortised over
a period not exceeding five years.

A. (1), (2) and (3)


B. (1) and (2) only
C. (1) and (3) only
D. (2) and (3) only

2. Which of the following statements about research and development


expenditure are correct according to IAS38 Intangible Assets?

1) If certain conditions are met, an enterprise may decide to capitalise


development expenditure.
2) Research expenditure, other than capital expenditure on research
facilities, must be written off as incurred.
3) Capitalised development expenditure must be amortised over a period not
exceeding 5 years.
4) Capitalised development expenditure must be disclosed in the statement
of financial position under intangible non-current assets.

A. 1, 2 and 4 only
B. 1 and 3 only
C. 2 and 4 only
D. 3 and 4 only.

3. Which of the following statements concerning the accounting treatment of


research and development expenditure are true, according to IAS 38
Intangible Assets?

1) Development costs recognised as an asset must be amortised over a


period not exceeding five years.
2) Research expenditure, other than capital expenditure on research
facilities, should be recognised as an expense as incurred.
3) In deciding whether development expenditure qualifies to be recognised
as an asset, it is necessary to consider whether there will be adequate
finance available to complete the project.

___________________________________________________________________________

167
4) Development projects must be reviewed at each statement of financial
position date, and expenditure on any project no longer qualifying for
capitalisation must be amortised through the statement of profit or loss
over a period not exceeding five years.

A. 1 and 4
B. 2 and 4
C. 2 and 3
D. 1 and 3

4. IAS 38 Intangible Assets governs the accounting treatment of expenditure on


research and development.

The following statements about the provisions of IAS 38 may or may not be
correct.
1) Capitalised development expenditure must be amortised over a period not
exceeding five years.
2) If all the conditions specified in IAS 38 are met, development expenditure
may be capitalised if the directors decide to do so.
3) Capitalised development costs are shown in the statement of financial
position under the heading of Non-Current Assets.
4) Amortisation of capitalised development expenditure will appear as an
item in a company’s statement of changes in equity.

Which of these four statements are in fact correct?

A. 3 only
B. 2 and 3
C. 1 and 4
D. 1 and 3

5. A newly set up dot-com entity has engaged you as its financial advisor. The
entity has recently completed one of its highly publicized research and
development projects and seeks your advice on the accuracy of the following
statements made by one of its stakeholders.
___________________________________________________________________________

168
a) Costs incurred during the research phase can be capitalized
b) Costs incurred during the development phase can be capitalized if criteria
such as technical feasibility of the project being established are met
c) Training costs of technicians used in research can be capitalised
d) Designing of jigs and tools qualify as research activities

Which of these four statements are in fact correct?


A. a and b
B. b and c
C. b only
D. b and d

___________________________________________________________________________

169
_________________

ANSWER BANK
_________________

1. B – There is no time limit to amortise development costs.

___________________________________________________________________________

170
2. C – If the six criteria are met, an entity MUST capitalise development
expenditure.

3. C

4. A – Amortisation is an expense and is not included separately in the SOCIE.

5. C – Research costs are always an expense.

___________________________________________________________________________

171
Chapter 12
Accruals and Prepayments

12.1 ACCA SYLLABUS GUIDE OUTCOME 1:


Understand how the matching concept applies to accruals and prepayments

We have mentioned that one of the underlying assumptions in the


“Framework for the Preparation and Presentation of Financial Statements” is
the accruals concept. It is also known as the matching concept because of the
way it strives to match costs against the revenues generated by incurring
those costs. Its basic tenet is that revenues should be recognised (i.e.
included in the statement of profit or loss) in the period in which they are
earned, not necessarily when they are received in cash. Thus, for example, a
sale made to a customer on credit just before the year-end would be included
in that year's statement of profit or loss, even though the cash may not be
received until the following year.
In the same way, expenses are recognised according to the period to which
they relate, and not when they are paid. For example, an electricity bill not
paid by the year-end would still be charged in that year's statement of profit or
loss whereas rates paid in advance would be held back and not charged until
the next year.

12.2 ACCA SYLLABUS GUIDE OUTCOMES 2 - 5:


Identify and calculate the adjustments needed for accruals and prepayments
in preparing financial statements
Illustrate the process of adjusting for accruals and prepayments in preparing
financial statements
Prepare the journal entries and ledger entries for the creation of an accrual or
prepayment
Understand and identify the impact on profit and net assets of accruals and
prepayments.

12.2.1 Accrued expenses (accruals) are expenses which relate to an accounting


period but have not been paid for. They are expenses which are charged
against the profit for a particular period, even though they have not yet been
paid for.

Accruals are included in payables as current liabilities as they represent


liabilities which have been incurred but for which no invoice has yet been
received.

___________________________________________________________________________

172
Accounting Treatment: Accruals

Dr Expense (I/S)
Cr Accruals (SOFP)

Lecture Example 1

Light Stores receives electricity bills quarterly. It paid the following electricity
bills during its accounting year ended 28 Feb 20X7:

Date paid
$
4.6.X6 (covering quarter ended 31.5.20X6) 70.50
5.9.X6 (covering quarter ended 31.8.20X6) 81.80
2.12.X6 (covering quarter ended 30.11.20X6) 100.20
10.2.X7 (covering the two months to 28.1.20X7) 108.00

On 3.6.20X7 an electricity bill was received for $105 covering the quarter
ended 30.4.20X7.

On the basis of the above data, you are required to


a. calculate the electricity expense to be charged to the statement
of profit or loss for the year ended 28 February 20X7
b. calculate the amount of any accrual/prepayment at the end of
the year
c. state the journal entry for the accrual/prepayment

Lecture Example 2

A company pays rent quarterly in arrears on 1 January, 1 April, 1 July and 1


October each year. The rent was increased from $90,000 per year to
$120,000 per year as from 1 October 20X2.

What rent expense and accrual should be included in the company’s financial
statements for the year ended 31 January 20X3?

12.2.2 Prepaid expenses (prepayments) are expenses which have already been
paid but relate to a future accounting period. Therefore, these are payments
which have been made in one accounting period, but should not be charged
against profit until a later period, because they relate to that later period.

___________________________________________________________________________

173
Prepayments are included in receivables in current assets in the statement of
financial position. They are assets as they represent money that has been
paid out in advance of the expense being incurred.

Accounting Treatment: Prepayments

Dr Prepayments (SOFP)
Cr Expense (I/S)

Lecture Example 3

A business opens a shop on 1 January 20X7. The rent is $20,000 per annum
and is payable quarterly in advance. Payments were made as follows: -
$
1 January 20X7 5,000
20 March 20X7 5,000
25 June 20X7 5,000
29 September 20X7 5,000
24 December 20X7 5,000

On the basis of the above data, you are required to


a. calculate the rent expense to be charged to the statement of
profit or loss for the year ended 31 December 20X7
b. calculate the amount of any accrual/prepayment at the end of
the year
c. state the journal entry for the accrual/prepayment

Lecture Example 4

Michelle rents premises at an annual rental of $1,000. The rates payable for
the accounting year 1 July 20X6 – 30 June 20X7, his first year of business,
were $360. Cheques for rent and rates were paid as follows: -

20X6 $
July 28 Rates for 9 months to 31 March 20X7 220
Sept 28 Rent for 3 months to 30 September 20X6 250

20X7
Jan 3 Rent for 3 months to 31 December 20X6 250
Mar 28 Rent for 3 months to 31 March 20X7 250
Apr 30 Rates for 6 months to 30 September 20X7 280

What rent and rates expense should be included in the company’s statement
of profit or loss for the year ended 30 June 20X7

___________________________________________________________________________

174
12.2.3 Reversal of Accruals and Prepayments

Accruals and prepayments brought forward at the beginning of the year must be
reversed.

Five steps are involved: -


a. At the beginning of the year, reverse opening accrual or prepayment
b. Double-entry: -

1. Reversal of an accrual

Dr Accruals (SOFP)
Cr Expense (I/S)

2. Reversal of a prepayment

Dr Expense (I/S)
Cr Prepayment (SOFP)

c. Post the cash paid during the year


d. Post any closing accrual or prepayment
e. Balance off the expense and accruals/prepayments accounts

Lecture Example 5

At 1 July 20X4 RCA Malta had prepaid insurance of $8,200. On 1 January 20X5 the
company paid $38,000 for insurance for the year to 30 September 20X5.

What figures should appear for insurance in the company’s financial statements for
the year ended 30 June 20X5?

Statement of profit or loss Statement of Financial Position


A. $27,200 Prepayment $19,000
B. $39,300 Prepayment $9,500
C. $36,700 Prepayment $9,500
D. $55,700 Prepayment $9,500

___________________________________________________________________________

175
Lecture Example 6

Details of a company’s insurance policy are shown below:


Premium for year ended 31 March 20X6 paid April 20X5 $10,800
Premium for year ending 31 March 20X7 paid April 20X6 $12,000

What figures should be included in the company’s financial statements for the year
ended 30 June 20X6?
Statement of profit or loss Statement of Financial Position
$ $
A 11,100 9,000 prepayment (Dr)
B 11,700 9,000 prepayment (Dr)
C 11,100 9,000 accrual (Cr)
D 11,700 9,000 accrual (Cr)

12.2.4 Accrued and Deferred Income

An entity will accrue income when it has earned the income during the period but it
has not yet been invoiced or received. This will increase income in the statement of
profit or loss and be shown as a receivable in the statement of financial position at
year end.

Accounting Treatment: Accrued Income

Dr Accrued income (SOFP)


Cr Income Account (I/S)

When an entity has received income in advance of it being earned, it should be


deferred to the following period. This will reduce income in the statement of profit or
loss and be shown as a payable in the statement of financial position at the year
end.

Accounting Treatment: Deferred Income

Dr Income Account (I/S)


Cr Deferred Income (SOFP)

Lecture Example 7

A company sublets part of its office accommodation. In the year ended 30 June
20X5 cash received from tenants was $83,700.

Details of rent in arrears and in advance at the beginning and end of the year were:

___________________________________________________________________________

176
In arrears In advance
$ $
30 June 20X4 3,800 2,400
30 June 20X5 4,700 3,000

All arrears of rent were subsequently received.

What figure for rental income should be included in the company’s statement of profit
or loss for the year ended 30 June 20X5?

A. $84,000
B. $83,400
C. $80,600
D. $85,800

___________________________________________________________________________

177
__________________________

KEY NOTES
_________________

___________________________________________________________________________

178
1. The accruals concept states that revenues should be recognised (i.e. included
in the statement of profit or loss) in the period in which they are earned, not
necessarily when they are received in cash. In the same way, expenses are
recognised according to the period to which they relate, and not when they are
paid.

2. Accrued expenses (accruals) are expenses which relate to an accounting


period but have not been paid for. Accruals are included in payables in current
liabilities as they represent liabilities which have been incurred but for which no
invoice has yet been received.

Accounting Treatment: Accruals

Dr Expense (I/S)
Cr Accruals (SOFP)

3. Prepaid expenses (prepayments) are expenses which have already been paid
but relate to a future accounting period. Prepayments are included in
receivables as current assets in the statement of financial position.

Accounting Treatment: Prepayments

Dr Prepayments (SOFP)
Cr Expense (I/S)

___________________________________________________________________________

179
4. Accruals and prepayments brought forward at the beginning of the year must
be reversed.

Four steps are involved: -


a. At the beginning of the year, reverse opening accrual or prepayment
b. Double-entry: -

1. Reversal of an accrual

Dr Accruals (SOFP)
Cr Expense (I/S)

2. Reversal of a prepayment

Dr Expense (I/S)
Cr Prepayment (SOFP)

c. Post the cash paid during the year


d. Post any closing accrual or prepayment
e. Balance off the expense and accruals/prepayments accounts

5. An entity will accrue income when it has earned the income during the period
but it has not yet been invoiced or received. This will increase income in the
statement of profit or loss and be shown as a receivable in the statement of
financial position at year end.

Accounting Treatment: Accrued Income

Dr Accrued income (SOFP)


Cr Income Account (I/S)

When an entity has received income in advance of it being earned, it should be


deferred to the following period. This will reduce income in the statement of
profit or loss and be shown as a payable in the statement of financial position at
the year end.

Accounting Treatment: Deferred Income

Dr Income Account (I/S)


Cr Deferred Income (SOFP)

___________________________________________________________________________

180
6. Effect on profit and assets/liabilities

Effect on Effect on profit Effect on


income/expenses assets/liabilities
Accruals Increases Reduces profit Increases liabilities
expenses
Prepayments Reduces expenses Increases profit Increases assets
Prepayments of Reduces income Reduces profit Increases liabilities
income
Income accrued Increases income Increases profit Increases assetss

___________________________________________________________________________

181
__________________________

QUESTION BANK
_________________

___________________________________________________________________________

182
1. B, a limited liability company, receives rent for subletting part of its office
premises to a number of tenants.

In the year ended 31 December 20X4 B received cash of $318,600 from its
tenants.

Details of rent in advance and in arrears at the beginning and end of 20X4 are as
follows:
31 December 20X4 31 December 20X3
$ $
Rent received in advance 28,400 24,600
Rent owing by tenants 18,300 16,900

All rent owing was subsequently received

What figure for rental income should be included in the statement of profit or loss
of B for 20X4?

A. $341,000
B. $336,400
C. $300,800
D. $316,200

2. During 20X4, B, a limited liability company, paid a total of $60,000 for rent,
covering the period from 1 October 20X3 to 31 March 20X5.

What figures should appear in the company’s financial statements for the year
ended 31 December 20X4?

Statement of profit or loss Statement of Financial Position


A. $40,000 Prepayment $10,000
B. $40,000 Prepayment $15,000
C. $50,000 Accrual $10,000
D. $50,000 Accrual $15,000

3. Beth’s draft accounts for the year to 31 October 20X5 report a loss of $1,486.
When she prepared the accounts, Beth did not include an accrual of $1,625 and
a prepayment of $834.

What is Beth’s profit or loss for the year to 31 October 20X5 following the
inclusion of the accrual and prepayment?

A. a loss of $695
B. a loss of $2,277
C. a loss of $3,945
D. a profit of $1,807

___________________________________________________________________________

183
4. Brighton has received telephone bills as follows:

Date received Amount of bill Date paid


$
Quarter to 30 November 20X0 December 20X0 739.20 Jan 20X1
Quarter to 28 February 20X1 March 20X1 798.00 April 20X1
Quarter to 31 May 20X1 June 20X1 898.80 June 20X1
Quarter to 31 August 20X1 September 20X1 814.80 Oct 20X1
Quarter to 30 November 20X1 December 20X1 840.00 Jan 20X2
Quarter to 28 February 20X2 March 20X2 866.00 March 20X2

What is the charge for telephone in the statement of profit or loss for the year
ended 31 December 20X1?

What is the amount of accrual or prepayment at 31 December 20X1?

5. On 1 April 20X0 a sole trader paid $3,080 in rent for the year ending 31 March
20X1. This was an increase of 10% on the charge for the previous year.

What is the correct charge for rent in her statement of profit or loss for the year
ended 31 December 20X0?

6. A business sublets part of its office accommodation.

The rent is received quarterly in advance on 1 January, 1 April, 1 July and 1


October. The annual rent has been $24,000 for some years, but it was increased
to $30,000 from 1 July 20X5.

What amounts for this rent should appear in the company’s financial statements
for the year ended 31 January 20X6?

Statement of profit or loss Statement of Financial Position

A $27,500 $5,000 in sundry receivables


B $27,000 $2,500 in sundry receivables
C $27,000 $2,500 in sundry payables
D $27,500 $5,000 in sundry payables

___________________________________________________________________________

184
__________________________

ANSWER BANK
_________________

___________________________________________________________________________

185
1. D

Rent receivable A/c


$ $
Rent in arrears - reversal 16,900 Rent in advance - reversal 24,600
I/S 316,200 Cash 318,600
Rent in advance 28,400 Rent in arrears 18,300
361,500 361,500

2. A

60,000

----------- --------

1/10 1/1/X4 31/12/X4 31/3/X5

Expense = 60,000 x 12 months = 40,000


18 months

Prepayment = 60,000 x 3 = 10,000


18

3. B

$
Original loss (1,486)
Accrual (1,625)
Prepayment 834
Revised loss (2,277)

4. $3,374.27

(2/3 × 798.00) + 898.80 + 814.80 + 840.00 + (1/3 × 866.00)

5. $3,010

$
$3,080 × 9/12 2,310
$3,080 × 100/110 × 3/12 700

___________________________________________________________________________

186
3,010

6.

Rent receivable A/c


$ $
1.1 Statement of profit or loss 27,500 1.2 Rent in advance - reversal 4,000
1.4 Cash 6,000
1.7 Cash 7,500
1.10 Cash 7,500
1.1 Cash 7,500
31.1 Rent in advance 5,000
32,500 32,500

___________________________________________________________________________

187
Chapter 13
Irrecoverable Debts and
Allowance for Receivables

13.1 ACCA SYLLABUS GUIDE OUTCOME 1:


Identify the benefits and costs of offering credit facilities to customers

Today, very few businesses expect to be paid immediately in cash. Most businesses
buy and sell to one another on credit terms. A business will allow credit terms to
customers and receive credit terms from its suppliers. This provides the benefit of
allowing businesses to keep trading without having to provide cash 'up front'.

However, providing credit facilities to customers can lead to problems. Customers


might fail to pay, either out of dishonesty or because they have gone bankrupt.
Therefore, the costs of offering credit facilities to customers can include:

1. Interest costs of an overdraft, if customers do not pay promptly.


2. Costs of trying to obtain payment, e.g. chasing customers by phone
3. Court costs, e.g. the costs of legal letters

13.2 ACCA SYLLABUS GUIDE OUTCOME 2:


Understand the purpose of an aged receivables analysis

A tool to control these problems of providing credit facilities is the aged receivables
analysis. This shows how long invoices have been outstanding, current, 30 days, 60
days, 90 and 90+ days, and may also indicate that a customer is unable to pay. Most
credit controllers will have a system of chasing up payment for long outstanding
invoices.

13.3 ACCA SYLLABUS GUIDE OUTCOME 3:


Understand the purpose of credit limits

Another tool in credit control is the credit limit. A customer will be given a credit limit,
which cannot be exceeded. This is a threshold that a company will allow its
customers to owe at any one time without having to go back and review their credit
file. Credit limit is the maximum amount that a firm is willing to risk in an account.

___________________________________________________________________________

188
13.4 ACCA SYLLABUS GUIDE OUTCOME 4:
Prepare the book-keeping entries to write off an irrecoverable debt

Irrecoverable debts (bad debts) are specific debts owed to a business which it
decides are never going to be paid. If a debt is definitely irrecoverable, the prudence
concept dictates it should be written off to the statement of profit or loss as a bad
debt. The value of outstanding receivables must be reduced by the amount written
off. This is because the customers are no longer expected to pay, and it would be
misleading to show them in the statement of financial position as current assets of
the business for which cash payment is expected within one year.

Accounting treatment

Dr Bad debts expense (I/S)


Cr Trade Receivables (SOFP)

Lecture Example 1

Dora Co has trade receivables at 31 December 20X7 of $100,000. A customer,


Swiper Inc, who owes Dora Co $5,000, at the end of 20X7, has been declared
bankrupt and the whole amount is considered irrecoverable.

Required: -
a) Calculate the new balance on the trade receivables account
b) Calculate the bad debts expense which is transferred to the statement of profit
or loss

13.5 ACCA SYLLABUS GUIDE OUTCOME 5:


Record a bad (irrecoverable) debt recovered

An irrecoverable debt which has been written off might occasionally be unexpectedly
paid. If it is paid in the same accounting period, the write-off journal can simply be
reversed. The only accounting problem to consider is when a debt written off as
irrecoverable in one accounting period is subsequently paid in a later accounting
period. In this case, the amount paid should be recorded as additional income in the
statement of profit or loss of the period in which the payment is received.

Accounting Treatment

Dr Cash
Cr Trade Receivables

Dr Trade Receivables
Cr Bad debts recovered (I/S)

___________________________________________________________________________

189
Lecture Example 2

Following from Lecture Example 1, Dora Co subsequently received a cheque from


Swiper Inc of $5,000.

Required: -

Show the treatment of this transaction in the relevant ledger accounts

13.6 ACCA SYLLABUS GUIDE OUTCOME 6:


Prepare the book-keeping entries to create and adjust an allowance for
receivables

13.6.1 Doubtful Debts:

If a debt is possibly irrecoverable, an allowance for the potential irrecoverability of


that debt should be made.

Accounting Treatment

Dr Doubtful debt expense (I/S)


Cr Allowance for receivables (SOFP)

This allowance is offset against trade receivables in the statement of financial


position.

Lecture Example 3

Following from Lecture Example 1, a further review of Dora’s customer files indicates
that there is some uncertainty whether Benny Co will pay its amount due of $2,500.

Required: -

Show the necessary entries in the relevant ledger accounts

13.6.2 Types of allowances:

There are two types of allowance for receivables.


1. Specific allowance – an allowance against a particular receivable (e.g.
Lecture example 3 – an allowance provided against Benny Co)
2. General allowance – a percentage allowance based on past experience of
irrecoverable debts (e.g. 2% of all outstanding receivables)

___________________________________________________________________________

190
Therefore, an allowance for receivables provides for future irrecoverable debts, as a
prudent precaution by the business. For both types of allowance for receivables, the
double-entry still remains: -

Dr Doubtful debt expense (I/S)


Cr Allowance for receivables (SOFP)

13.6.3 Specific allowance:

There are two situations in which a specific allowance previously done is no longer
required: -
1. customer pays outstanding amount
2. customer goes bankrupt

13.6.3.1 Customer pays outstanding amount

Accounting treatment

Dr Cash (SOFP)
Cr Trade Receivables (SOFP)

Dr Allowance for receivables (SOFP)


Cr Doubtful debts expense (I/S)

Therefore, this will be credited to income in the statement of profit or loss or it will
reduce the total expense for bad and doubtful debts.

Lecture Example 4

Following from Lecture Examples1 and 3, Benny Co has paid his amount due of
$2,500 in cash.

Required: -

Show the accounting treatment in the books of Dora Co.

13.6.3.2 Customer goes bankrupt

Accounting treatment

Dr Allowance for receivables (SOFP)


Cr Trade Receivables (SOFP)

___________________________________________________________________________

191
Therefore, no entry is posted in the bad and doubtful debts account as this would
have already been debited with the expense in the first year when we have taken the
specific allowance.

Lecture Example 5

Following from Lecture Examples 1 and 3, Benny Co has been declared bankrupt
and his amount due of $2,500 is now considered irrecoverable.

Required: -
Show the accounting treatment in the books of Dora Co.

13.7 ACCA SYLLABUS GUIDE OUTCOME 7


Illustrate how to include movements in the allowance for receivables in the
statement of profit or loss and how the closing balance of the allowance
should appear in the statement of financial position

13.7.1 How do we calculate the general allowance?

There are a number of steps which must be followed.

1. Take the balance on the trade receivables account after posting credit sales
and cash received from credit customers
2. Deduct bad debts from this balance of trade receivables
3. Deduct also any specific allowances from trade receivables
4. Calculate the general allowance by applying the percentage given to the
remaining balance

Example: - General allowance


$
Trade receivables (net of bad debts written off) 10,000
Less: specific allowance ( 2,000)
8,000
General allowance @ 2% (2% x 8,000) 160

13.7.2 General allowance – subsequent years

In subsequent years, adjustments may be needed to the amount of the allowance.


The procedure to be followed then is: -
1. Calculate the new allowance required.
2. Compare it with the existing balance on the allowance account (i.e. the
balance b/f from the previous accounting period).
3. Calculate increase or decrease required.
___________________________________________________________________________

192
(i) If a higher allowance is required now:
Dr Irrecoverable debts expense
Cr Allowance for receivables
with the amount of the increase.
(ii) If a lower allowance is needed now than before:
Dr Allowance for receivables
Cr Irrecoverable debts expense
with the amount of the decrease.

Lecture Example 6

A. Boots has total receivables outstanding at 31 December 20X2 of $28,000. He


believes that about 1% of these balances will not be collected and wishes to make
an appropriate allowance. Before this date, he has not made any allowance for
receivables at all.
On 31 December 20X3 his trade receivables amount to $40,000. His experience
during the year has convinced him that an allowance of 5% should be made.

What accounting entries should A. Boots make on 31 December 20X2 and 31


December 20X3, and what figures for trade receivables will appear in his statement
of financial position at those dates?

Lecture Example 7

At 31 December 20X4 a company’s trade receivables totalled $864,000 and the


allowance for receivables was $48,000.

It was decided that debts totalling $13,000 were to be written off, and the allowance
for receivables adjusted to five per cent of the receivables.

What figures should appear in the statement of financial position for trade
receivables (after deducting the allowance) and in the statement of comprehensive
income for the total of irrecoverable debts and movement on receivables allowance?

Irrecoverable debts
and receivables allowance Net trade receivables
$ $
A. 8,200 807,800
B. 7,550 808,450
C. 18,450 808,450
D. 55,550 808,450

___________________________________________________________________________

193
13.8 ACCA SYLLABUS GUIDE OUTCOME 8
Prepare, reconcile and understand the purpose of supplier statements.

Lecture Example 8

Alpha buys goods from Beta. At 30 June 20X5 Beta's account in Alpha's records
showed $5,700 owing to Beta. Beta submitted a statement to Alpha as at the same
date showing a balance due of $5,200.

Which of the following could account fully for the difference?


A. Alpha has sent a cheque to Beta for $500 which has not yet been
received by Beta.
B. The credit side of Beta's account in Alpha's records has been
undercast by $500.
C. An invoice for $250 from Beta has been treated in Alpha's records as if
it had been a credit note.
D. Beta has issued a credit note for $500 to Alpha which Alpha has not
yet received.

___________________________________________________________________________

194
__________________________

KEY NOTES
_________________

___________________________________________________________________________

195
1. Credit facilities provide the benefit of allowing businesses to keep trading
without having to provide cash 'up front'. However, providing credit facilities to
customers can lead to problems. Customers might fail to pay, either out of
dishonesty or because they have gone bankrupt.

2. An aged receivables analysis shows how long invoices have been


outstanding and may also indicate that a customer is unable to pay.

3. A credit limit is a threshold that a company will allow its customers to owe at
any one time without having to go back and review their credit file.

4. Irrecoverable debts (bad debts) are specific debts owed to a business which it
decides are never going to be paid.

Accounting treatment

Dr Bad debts expense (I/S)


Cr Trade Receivables (SOFP)

5. An irrecoverable debt which has been written off might occasionally be


unexpectedly paid.

Accounting Treatment

Dr Cash
Cr Trade Receivables

Dr Trade Receivables
Cr Bad debts recovered (I/S)

6. If a debt is possibly irrecoverable, an allowance for the potential


irrecoverability of that debt should be made.

Accounting Treatment

Dr Doubtful debt expense (I/S)


Cr Allowance for receivables (SOFP)

7. There are two types of allowance for receivables.


a. Specific allowance – an allowance against a particular receivable
b. General allowance – a percentage allowance based on past
experience of irrecoverable debts (e.g. 2% of all outstanding
receivables)

___________________________________________________________________________

196
8. There are two situations in which a specific allowance previously done is no
longer required: -

a. customer pays outstanding amount

Accounting treatment

Dr Cash (SOFP)
Cr Trade Receivables (SOFP)

Dr Allowance for receivables (SOFP)


Cr Doubtful debts expense (I/S)

b. customer goes bankrupt

Accounting treatment

Dr Allowance for receivables (SOFP)


Cr Trade Receivables (SOFP)

9. General allowance – calculation

a. Take the balance on the trade receivables account after posting credit
sales and cash received from credit customers
b. Deduct bad debts from this balance of trade receivables
c. Deduct also any specific allowances from trade receivables
d. Calculate the general allowance by applying the percentage given to
the remaining balance

10. In subsequent years, adjustments may be needed to the amount of the


allowance. The procedure to be followed then is: -
(i) Calculate the new allowance required.
(ii) Compare it with the existing balance on the allowance
(iii) Calculate increase or decrease required.

___________________________________________________________________________

197
__________________________

QUESTION BANK
_________________

___________________________________________________________________________

198
1. A company has been notified that a receivable has been declared bankrupt.
The company had previously made a specific allowance for this debt. Which
of the following is the correct double entry?

DR CR
A. Irrecoverable debts account Account receivable
B. Account receivable Irrecoverable debts account
C. Allowance for receivables Account receivable
D. Account receivable Allow for receivables

2. At 30 June 20X4 a company's allowance for receivables was $39,000. At 30


June 20X5 trade receivables totalled $517,000. It was decided to write off
debts totalling $37,000 and to adjust the allowance for receivables to the
equivalent of 5 per cent of the trade receivables based on past events.

What figure should appear in the statement of comprehensive income for


these items?

A. $61,000
B. $22,000
C. $24,000
D. $23,850

3. At 1 January 20X5, a company had an allowance for receivables of $18,000.


At 31 December 20X5 the company’s trade receivables were $458,000.

It was decided:
a) To write off debts totalling $28,000 as irrecoverable;
b) To adjust the allowance for receivables to the equivalent of 5% of the
remaining receivables based on past experience.

What figure should appear in the company’s statement of comprehensive


income for the total of debts written off as irrecoverable and the movement in
the allowance for receivables for the year ended 31 December 20X5?

A. $49,500
B. $31,500
C. $32,900
D. $50,900

4. At 1 July 20X5 a company’s allowance for receivables was $48,000.


At 30 June 20X6, trade receivables amounted to $838,000. It was decided to
write off $72,000 of these debts and adjust the allowance for receivables to
$60,000.

___________________________________________________________________________

199
What are the final amounts for inclusion in the company’s statement of
financial position at 30 June 20X6?
Trade Allowance for Net
Receivables Receivables Balance
$ $ $
A. 838,000 60,000 778,000
B. 766,000 60,000 706,000
C. 766,000 108,000 658,000
D. 838,000 108,000 730,000

5. At 1 January 20X6, a company had an allowance for receivables of $49,000.


At 31 December 20X6, the company’s trade receivables were $863,000 and it
was decided to write off balances totalling $23,000 and to adjust the
allowance for receivables to the equivalent of 5% of the remaining receivables
based on past experience.

What total figure should appear in the company’s statement of comprehensive


income for irrecoverable debts and allowance for receivables?

A. $16,000
B. $65,000
C. $30,000
D. $16,150

___________________________________________________________________________

200
__________________________

ANSWER BANK
_________________

___________________________________________________________________________

201
1. C

2. B

Allowance for receivables ((517,000 – 37,000) × 5%) 24,000


Previous allowance (39,000)
Reduction (15,000)
Debts written off 37,000
Charge to statement of profit or loss 22,000

3. B

$
Closing receivables 458,000
Irrecoverable debts w/off (28,000)
430,000
Allowance required (5% × 430,000) 21,500
Existing allowance (18,000)
Increase required 3,500
Charge to statement of profit or loss (28,000 + 3,500) 31,500

4. B

Trade receivables = 838,000 – 72,000


= 766,000
Allowance for receivables = 60,000
Net balance = 766,000 – 60,000
= 706,000

5. A

Trade receivables 863,000


Irrecoverable debts w/off (23,000)
840,000

Closing allowance for receivables (5% × 840,000) 42,000


Opening allowance 49,000
Reduction (7,000)
Charge = 23,000 – 7,000 = 16,000

___________________________________________________________________________

202
Chapter 14
Control Accounts

14.1 ACCA SYLLABUS GUIDE OUTCOME 1:


Understand the purpose of control accounts for accounts receivable and
accounts payable

A control account is a total account in the nominal ledger. Its balance represents an
asset or a liability which is the grand total of many individual assets or liabilities. The
control accounts provide a convenient total which can be used immediately in
extracting a trial balance or preparing accounts.

Most businesses operate control accounts for trade receivables and payables, but
such accounts may be useful in other areas too, e.g. sales tax control account.

The accounts of individual trade receivables and payables are found in the
Receivables Ledger (RL) and Payables Ledger (PL) respectively. These are
maintained for memorandum purposes only. Therefore, entering a sales invoice in
the account of an individual customer is not part of the double entry process. These
individual accounts are necessary for administrative convenience. For example, a
customer may wish to query the balance he owes to the business.

Reconciliation between the control account total and the receivables ledger will help
to detect errors, thus providing an important control.

14.2 ACCA SYLLABUS GUIDE OUTCOME 2:


Understand how control accounts relate to the double-entry system

In Chapter 5, we discussed the books of prime entry. We have also looked at the
flow of information where we have seen that the totals from the books of prime entry
are posted in the nominal accounts using double-entry.

___________________________________________________________________________

203
B = $600 B = $450

Source Cheques
Invoices Received
Documents A = $500 A = $520

Books of Memorandum Ledger


Original Receivables
Entry Ledger
Sales Day Cash Book
(personal accounts)
Book
$ A B $
A 500 A 520
500 520 600 450
B 600 B 450
Balance Balance
Total 1100 970
=$20CR =$150DR
Overall Balance
= $130DR

Receivables account
Ledger Sales account (control a/c) Cash account
Accounts CR Total DR Total
(Nominal Sales $1100 DR Total $ Sales $970
/ General Sales 1100
Ledger) CR Total
Cash 970

Balance (DR) 130

The trade receivables figure shows the total amount owed by all customers at a
particular point in time. It is also called the receivables ledger control account
(RLCA).

The trade payables figure shows the total amount owed to all suppliers at a particular
point in time. It is also called the payables ledger control account (PLCA).
___________________________________________________________________________

204
14.3 ACCA SYLLABUS GUIDE OUTCOME 3:
Prepare ledger control accounts from given information

14.3.1 Main entries in control accounts

The two main entries in the RLCA are credit sales and cash received from credit
customers.

The double-entry for credit sales is: -

Dr RLCA
Cr Sales

The double-entry for cash received from customers is: -

Dr Bank/Cash
Cr RLCA

The two main entries in the PLCA are credit purchases and cash paid to credit
suppliers.

The double-entry for credit purchases is: -

Dr Purchases
Cr PLCA

The double-entry for cash paid to suppliers is: -

Dr PLCA
Cr Bank/Cash

There are other entries which will be included in the control accounts. It is important
to note that any transaction recorded in the RLCA or the PLCA is also reflected in
the memorandum ledgers.

14.3.2 Other entries in control accounts

14.3.2.1 Contras

This is where an amount of money is owed to a supplier, who is also a customer who
owes money, i.e., a payable who is also a receivable. Instead of paying the full
amount to the creditor, who then pays the full amount of their debt to you, the two
amounts owed and owing are offset against each other and only the difference is
settled in cash. This must be reflected in the individual accounts in the sales and
purchase ledgers and in the control accounts in the nominal ledger.
___________________________________________________________________________

205
The double entry for a contra is: -

Dr PLCA
Cr RLCA

The contra value is of the maximum common amount. A contra always has the effect
of reducing both receivables and payables.

14.3.2.2 Returns, Credit Notes and Refunds

When a customer returns goods which have already been paid, he may either be
given a credit note or refunded for the value of these returned goods.

When a credit note is given, the double-entry is: -

Dr Returns In (sales returns)


Cr RLCA

When the customer is refunded: -

Dr RLCA
Cr Bank

The same applies when a customer over-pays an invoice.

14.3.2.3 Interest charged on overdue accounts

An entity may decide to charge interest if a customer does not pay within the
specified credit period.

The double-entry for interest charged on these overdue accounts is: -

Dr RLCA
Cr Interest Receivable (Income (I/S))

14.3.2.4 Discounts

There are two types of discounts: -

1. Trade discount is a reduction in the list price of an article, given by a wholesaler


or manufacturer to a retailer. It is often given in return for bulk purchase orders.
2. Cash/settlement discount is a reduction in the amount payable for the purchase
of goods or services in return for payment in cash, or within an agreed period.

___________________________________________________________________________

206
Trade discounts received are deducted from the cost of purchases. Trade discounts
allowed are deducted from sales. Therefore, sales are recorded net of trade
discounts but inclusive of settlement discounts. Purchases are also recorded
net of trade discounts but inclusive of settlement discounts. Therefore, trade
discounts never appear in the financial statements.

Lecture Example 1

Joe buys goods worth $3,500 from Eddie. On $2,000 worth, he gets trade discount
of 20%, no trade discount is available on the rest. However Joe always makes sure
that he pays within 10 days in order to obtain Eddie's settlement discount of 5%.
How much will Joe pay Eddie?

A. $2,495
B. $2,945
C. $2,800
D. $3,025

Sales Tax and Discounts

Sales tax is calculated on the amount after all discounts, regardless of whether the
discount is taken or not.

Lecture Example 2

Cloud buys goods with a list price of $50,000 from Moon. Cloud receives a trade
discount of 12% from Moon on all its purchases and a further 4% discount if payment
is made within 10 days. Sales tax is charged at 15%.

What figure should Cloud show in Moon’s personal account to record its purchase?

___________________

14.3.3 Proforma Receivables Ledger Control Account

___________________________________________________________________________

207
Receivables Ledger Control A/c
$ $
Balance b/d 7,120 cash received 52,450
Sales 52,500 Discounts allowed 1,250
Dishonoured cheques 1,000 Returns in 800
Irrecoverable debts 300
Balance c/d 5,820
60,620 60,620

14.3.4 Proforma Payables Ledger Control Account

Payables Ledger Control A/c


$ $
Cash paid 29,840 Balance b/d 8,300
Discounts received 100 Purchases 31,100
Returns out 60
Balance c/d 9,400

39,400 39,400

Lecture Example 3

For the year ended 30 September 20X9 the following particulars are available.
$
Sales 63,728
Purchases 39,974
Cash from trade accounts receivable 55,212
Cash to trade accounts payable 37,307
Discount received 1,475
Discount allowed 2,328
Returns inwards 1,002
Returns outwards 535
Irrecoverable debts written off 326
Cash received in respect of debit balances in payables ledger 105
Amount due from customer as shown by receivables ledger,
offset against amount due to the same firm as shown by
payables ledger (settlement by contra) 434
Allowances to customers on goods damaged in transit 212

On 1st Oct 20X8 Trade Rec were $8024 Dr


$ 57 Cr

Trade Pay were $6235 Cr


$105 Dr
___________________________________________________________________________

208
On 30 September 20X9 there were no credit balances in the receivables ledger
except those outstanding on 1 October 20X8, and no debit balances in the payables
ledger.

You are required to write up the following accounts recording the above transactions
bringing down the balances as on 30 September 20X9:
a. Receivables control account
b. Payables control account

14.3.5 Control account balances

Very often, PLCA’s have a credit balance since payables are a liability. However,
there may be situations when there will be a debit balance on a PLCA

a) Returning goods which have been paid for and receiving a ‘credit’ (to us, a debit)
on our account
b) Overpayment
c) Payments in advance

There may be situations when there will be a credit balance on a RLCA

a) Returned goods credit to account


b) Overpayment
c) Payments in advance

14.4 ACCA SYLLABUS GUIDE OUTCOME 4-6:


Perform control account reconciliation for accounts receivable and accounts
payable
Identify errors which would be highlighted by performing control account
reconciliation
Identify and correct errors in control accounts and ledger accounts

Both the receivables and payables control accounts should be balanced regularly
and the balance agreed to the sum of the balances on the memorandum ledgers, the
receivables ledger and the payables ledger respectively.

Therefore, if the balances in the receivables/payables ledgers are added up, they
should agree to the RLCA/PLCA balances. If not, an error must have occurred at the
same point in the system.

14.4.1 Types of error

Errors which affect the control accounts:


___________________________________________________________________________

209
a) Over/undercast SDB, PDB, CB.
b) Transposition error in posting total from SDB/PDB/CB to nominal ledger.
c) Entry omitted from SDB/PDB/CB.

Errors which affect the list of balances (receivables/payables ledger):

a) Omit balance from the list.


b) List a debit balance as a credit/vice versa.
c) Transposition error in filling ledger from books of prime entry.

Errors which affect both the lists of balances and RLCA/PLCA:

a) Details being incorrectly recorded on the original source documentation i.e.


sales/purchase invoice.
b) Loss of original source documentation so it is not recognised anywhere in the
system.

14.4.2 Proforma Receivables Ledger Control Account Reconciliation

Receivables Ledger Control A/c


$ $
Balance b/d X Transposition error in posting X
Sales day book undercast X
Sales omitted from SDB X Balance c/d X
X X

Reconciliation Statement

$ $ $
+ -

Total per listing of receivables ledger balances X


Adjustments:
Balance omitted X
Credit balance listed as debit (2X)
X X X
Balance as per adjusted control account X

___________________________________________________________________________

210
14.4.3 Proforma Payables Ledger Control Account Reconciliation

Payables Ledger Control A/c


$ $
Transposition error in posting X Balance n/d X
Purchase day book undercast X
Balance c/d X Purchases omitted from PDB X
X X

Reconciliation Statement

$ $ $
+ -

Total per listing of payables ledger balances X


Adjustments:
Balance omitted X
Debit balance listed as credit (2X)
X X X
Balance as per adjusted control account X

Lecture Example 4

A receivables ledger control account shows a balance of $35,100, while the list of
balances totals $36,500.
The following discrepancies are discovered:
a. A credit balance of $350 has been included in the list of balances as a debit
b. A refund of $125 has not been posted to the receivables ledger control
account
c. One page of the sales day book has been undercast by $575

What is the reconciled balance?

A. $36,500
B. $35,800
C. $35,225
D. $37,200

Lecture Example 5

In reconciling the payables control account to the payables ledger, the following
discrepancies are noticed:

a. a credit note for $105 has been posted to the wrong side of the control
account;
b. the payables ledger has not been adjusted for a receivables ledger offset of
$2,055;
___________________________________________________________________________

211
c. an account with a credit balance of $348 has been omitted from the list of
payables ledger balances.

The balance on the payables control account is $3,627. The balance on the
payables ledger is $5,124.

What is the reconciled balance?

A. $3,627
B. $3,069
C. $3,417
D. $3,765

___________________________________________________________________________

212
__________________________

KEY NOTES
_________________

___________________________________________________________________________

213
1. The Main Purpose of Control Accounts

The main purposes of control accounts include:

(i) to provide a check on the accuracy of entries in the individual accounts;


(ii) to provide a total of debtors and creditors at any time in an accounting
period;
(iii) to identify errors in the completion of the day books and in posting day
book totals;
(iv) to provide an internal check – where the system is being administered by
several persons it is important that internal controls exist and their work is
checked. The control account should be prepared by a person other than
the one completing the day books.

2. Main Entries in Control Accounts

a. Credit Sales

The double-entry for credit sales is: -

Dr RLCA
Cr Sales

b. Cash received from credit customers

The double-entry for cash received from customers is: -

Dr Bank/Cash
Cr RLCA

c. Credit purchases

The double-entry for credit purchases is: -

Dr Purchases
Cr PLCA

d. Cash paid to suppliers

The double-entry for cash paid to suppliers is: -

Dr PLCA
Cr Bank/Cash

___________________________________________________________________________

214
3. Other Entries in Control Accounts

a. Contras

The double entry for a contra is: -

Dr PLCA
Cr SLCA

The contra value is of the maximum common amount. A contra always has the effect
of reducing both receivables and payables.

b. Returns, Credit Notes and Refunds

When a credit note is given, the double-entry is: -

Dr Returns In (sales returns)


Cr RLCA

When the customer is refunded: -

Dr RLCA
Cr Bank

c. Interest charged on overdue accounts

The double-entry for interest charged on these overdue accounts is: -

Dr RLCA
Cr Interest Receivable (Income (I/S))

d. Discounts

Sales are recorded net of trade discounts but inclusive of settlement discounts.
Purchases are also recorded net of trade discounts but inclusive of settlement
discounts.

The double-entry for discounts allowed to customers is: -

Dr Discounts allowed
Cr RLCA

The double-entry for discounts received from suppliers is: -

Dr PLCA

___________________________________________________________________________

215
Cr Discounts received
Sales tax is calculated on the amount after all discounts, regardless of whether the
discount is taken or not.

4. Control Account Reconciliations

Both the receivables and payables control accounts should be balanced regularly
and the balance agreed to the sum of the balances on the memorandum ledgers, the
receivables ledger and the payables ledger respectively.

Types of error: -

Errors which affect the control accounts:


a) Over/undercast SDB, PDB, CB.
b) Transposition error in posting total from SDB/PDB/CB to nominal ledger.
c) Entry omitted from SDB/PDB/CB.

Errors which affect the list of balances (receivables/payables ledger):


a) Omit balance from the list.
b) List a debit balance as a credit/vice versa.
c) Transposition error in filling ledger from books of prime entry.

Errors which affect both the lists of balances and RLCA/PLCA:


a) Details being incorrectly recorded on the original source documentation i.e.
sales/purchase invoice.
b) Loss of original source documentation so it is not recognised anywhere in the
system.

___________________________________________________________________________

216
__________________________

QUESTION BANK
_________________

___________________________________________________________________________

217
1. The balance on the receivables ledger control account is $52 more than the
list of customer balances.
This could be caused by which one of the following?

A. An invoice for $52 omitted from the sales day book


B. A credit note for $52 omitted from the sales returns day book
C. A customer refund of $52 posted twice to the nominal ledger
D. A credit note for $26 entered twice into the receivables ledger

2. Mabel's supplier has allowed her 5% discount for prompt payment of her
account. How should this be posted?

A. DR Payables ledger control account; CR Discounts allowed


B. DR Discounts allowed; CR Payables ledger control account
C. DR Payables ledger control account; CR Discounts received
D. DR Discounts received; CR Payables ledger control account

3. Andrew buys goods with a list price of $7,200 on which he receives 20% trade
discount. His supplier offers 5% discount for payment within 10 days. Andrew
pays half of the invoiced amount within 10 days and the balance 3 weeks
later.

What is the total amount of money that he will pay for this order?

A. $5,616
B. $5,580
C. $5,400
D. $6,300

4. Which of the following is not the purpose of a receivables ledger control


account?

A. A receivables ledger control account provides a check on the


arithmetical accuracy of the personal ledger
B. A receivables ledger control account helps to locate errors in the trial
balance
C. A receivables ledger control account ensures that there are no errors in
the personal ledger
D. Control accounts deter fraud

___________________________________________________________________________

218
5. In an accounts receivable control account, which of the following lists is
composed only of items which would appear on the credit side of the
account?
A. Cash received from customers, sales returns, irrecoverable debts
written off, contras against amounts due to suppliers in the accounts
payable ledger
B. Sales, cash refunds to customers, irrecoverable debts written off,
discounts allowed
C. Cash received from customers, discounts allowed, interest charged on
overdue accounts, irrecoverable debts written off
D. Sales, cash refunds to customers, interest charged on overdue
accounts, contras against amounts due to suppliers in the accounts
payable ledger.

6. Ordan received a statement from one of its suppliers, Alta, showing a


balance due of $3,980. The amount due according to the payables ledger
account of Alta in Ordan’s records was only $230. Comparison of the
statement and the ledger account revealed the following differences:

(i) A cheque sent by Ordan for $270 has not been allowed for in
Alta’s statement.
(ii) Alta has not allowed for goods returned by Ordan $180.
(iii) Ordan made a contra entry, reducing the amount due to Alta by
$3,200, for a balance due from Alta in Ordan’s receivables
ledger. No such entry has been made in Alta’s records.

What difference remains between the two companies’ records after adjusting
for these items?

A. $460
B. $640
C. $6,500
D. $100

7. A payables ledger control account showed a credit balance of $768,420. The


payables ledger balances totalled $781,200.

Which one of the following possible errors could account in full for the
difference?

A. A contra against a receivables ledger debit balance of $6,390 has been


entered on the credit side of the payables ledger control account.
B. The total of discount allowed $28,400 was entered to the debit of the
payables ledger control account instead of the correct figure for
discount received of $15,620.
___________________________________________________________________________

219
C. $12,780 cash paid to a supplier was entered on the credit side of the
supplier’s account in the payables ledger.
D. The total of discount received $6,390 has been entered on the credit
side of the payables ledger control account.

8. The following receivables ledger control account has been prepared by a


trainee accountant:

Receivables Ledger Control A/c


20X5 $ 20X5 $
01-Jan Balance 318,650 31-Jan Cash from credit 181,140
Credit sales 161,770 customers
Cash sales 84,260 Interest charged on 280
Discounts allowed 1,240 overdue accounts
Irrecoverable debts
written off 1,390
Sales returns from
credit customers 3,990
Balance 379,120
565,920 565,920

What should the closing balance at 31 January 20X5 be after correcting the
errors in the account?

A. $292,380
B. $295,420
C. $292,940
D. $377,200

9. The receivables ledger control account below contains several incorrect


entries

Receivables Ledger Control Account


$ $
Balance 138,400 Credit sales 80,660
Cash received from Contras against credit balances 1,000
credit customers 78,420 in payables ledger
Discounts allowed 1,950
Irrecoverable debts 3,000
written off
Dishonoured cheques from 850
credit customers
Balance 129,360
216,820 216,820

___________________________________________________________________________

220
What should the closing balance be when all the errors are corrected?

A. $133,840
B. $135,540
C. $137,740
D. $139,840

10. The following payables ledger control account contains some errors. All
goods are purchased on credit.

Payables Ledger Control Account


$ $
Purchases 963,200 Opening balance 384,600
Discounts received 12,600 Cash paid to suppliers 988,400
Contras with amounts 4,200 Purchases returns 17,400
receivable in receivables
ledger
Closing balance 410,400

1,390,400 1,390,400

What should the closing balance be when the errors have been corrected?

A. $325,200
B. $350,400
C. $358,800
D. $376,800

___________________________________________________________________________

221
__________________________

ANSWER BANK
_________________

___________________________________________________________________________

222
1. C

2. C

The discount reduces the amount owing to creditors, so it is debited to the


payables ledger control account, and it is credited to 'discounts received' in
the statement of profit or loss. 'Discounts allowed' is used to post discounts
granted to customers.

3. A

$
Purchase price (7,200 x 80%) 5,760
Discount ((5,760/2) x 5%) (144)
5,616

4. C

5. A

6. D

$
Balance per Alta 3,980
Cheque not yet received (270)
Goods returned (180)
Contra Entry (3,200)
Revised Balance per Alta 330
Balance per Ordan (230)
Remaining Difference 100

7. B is correct.

A – The contra should be debited. Hence reduce further the PLCA by


twice the amount, $6,390 x 2 = $12,780.
B – Increase the balance of the PLCA by $12,780 (28,400 – 15,620).
C – Reduce the ledger by twice the amount of $12,780.
D – Reduce the PLCA balance by twice the amount of $6,390.

___________________________________________________________________________

223
8. C

Receivables Ledger Control A/c


20X5 $ 20X5 $
01-Jan Balance 318,650 31-Jan Cash from credit 181,140
Credit sales 161,770 customers
Interest charged on 280 Discounts allowed 1240
overdue accounts Irrecoverable debts
written off 1,390
Sales returns from
credit customers 3,990
Balance 292,940

480,700 480,700

9. B

Receivables Ledger Control Account


$ $
Balance 138,400 Cash received 78,420
Credit sales 80,660 Contras against credit balances 1,000
Dishonoured cheques from 850 in payables ledger
credit customers Discounts allowed 1,950
Irrecoverable debts 3,000
written off
Balance 135,540
219,910 219,910

10. A

Payables Ledger Control Account


$ $
Cash paid to suppliers 988,400 Opening balance 384,600
Discounts received 12,600 Purchases 963,200
Contras with amounts 4,200
receivable in receivables
ledger
Purchases returns 17,400
Closing balance 325,200

1,347,800 1,347,800

___________________________________________________________________________

224
Chapter 15
Bank Reconciliation

15.1 ACCA SYLLABUS GUIDE OUTCOME 1


Understand the purpose of bank reconciliations

We have already discussed the cash book as one of the main books of prime entry.
The cash book is used to record the detailed transactions of receipts and payments
affecting the bank account. These are then posted to the nominal ledger periodically.
At the end of each accounting period, the balance on the cash book should equal the
balance in the nominal ledger cash/bank account.

As an extra control over the cash figure, it should be possible to agree this figure to
an independent figure provided by the bank statement. This is not always a
straightforward agreement as there are many reasons why the two figures may not
be exactly the same. Therefore, we need to produce a reconciliation.

The aim of the reconciliation is to prove the


1. completeness
2. accuracy
3. validity
of cash receipts and payments.

15.2 ACCA SYLLABUS GUIDE OUTCOME 2


Identify the main reasons for differences between the cash book and the bank
statement

The balance on the cash account (which should be the same as the balance in the
cash book) is compared to the balance on the bank statements at a given date.
However, these two balances may not agree. There are various reasons: -

1. Time lag between writing a cheque and the payment appearing on the bank
statement (unpresented cheques)
2. Time lag between depositing amounts into the bank account and these
appearing on the bank statement (unrecorded lodgements)
3. Direct debits and standing orders are not yet recorded in the cash account (or
cash book)
4. Bank charges not recorded in the cash account (or cash book)
5. Errors, such as transposition errors, or casting errors in the cash account (or
cash book)
6. Errors made by the bank on the bank statement

___________________________________________________________________________

225
Therefore, differences between the cash book and the bank statement arise for 3
reasons:

1. Errors – usually in the cash book


2. Omissions – such as bank charges, standing orders and direct debits not
posted in the cash book
3. Timing differences – such as unpresented cheques and unrecorded
lodgements

Always remember: -

In our cash book,


A debit bank balance indicates an asset
but
In the bank statement,
A debit balance indicates a bank overdraft (we owe money to the bank – an asset for
the bank)

In our cash book,


A credit bank balance indicates a liability (overdraft)
but
In the bank statement,
A credit balance indicates a positive balance (the bank owes us money)

___________________________________________________________________________

226
15.3 ACCA SYLLABUS GUIDE OUTCOMES 3 and 4
Correct cash book errors and/or omissions
Prepare bank reconciliation statements

Example – Bank Reconciliation Statement

Bank Account

Balance b/d X Dishonoured cheque X


Direct Credit X Standing order X
Bank charges X
Direct debit X
Balance c/d X
X X

Balance as per bank statement X


less unpresented cheques (X)
add unrecorded lodgements X
add/less bank errors X
Balance as per adjusted cash book X

Lecture Example 1

At 30 September 20X6, the balance in the cash book of Wordsworth Co was


$805.15 debit. A bank statement on 30 September 20X6 showed Wordsworth Co to
be in credit by $1,112.30.

On investigation of the difference between the two sums, it was established that:
1. The cash book had been undercast by $90.00 on the debit side.
2. Cheques paid in not yet credited by the bank amounted to $208.20, called
outstanding lodgements.
3. Cheques drawn not yet presented to the bank amounted to $425.35 called
unpresented cheques.

Required
a. Show the correction to the cash book.
b. Prepare a statement reconciling the balance per bank statement to the
balance per cash book.

___________________________________________________________________________

227
Lecture Example 2

Gemma is reconciling her cash book to the bank statement. Her cash balance is
$2,357 and the balance on her statement is $25 overdrawn. She finds the following
differences:

a. bank charges of $23 and direct debits totalling $100 have not been
posted to the cash book;
b. there are unpresented cheques of $324; she paid in a batch of
cheques two days ago totalling $2,503 and these have not yet been
credited to her account;
c. a cheque she paid in last week for $80 has been dishonoured.

What will the reconciled balance be?

A. $2,154
B. $2,204
C. $2,357
D. $2,277

15.4 ACCA SYLLABUS GUIDE OUTCOMES 5 and 6


Derive bank statement and cash book balances from given information
Identify the bank balance to be reported in the final accounts.

Lecture Example 3

A bank statement shows a balance of $1,200 in credit. An examination of the


statement shows a $500 cheque paid in per the cash book but not yet on the bank
statement and a $1,250 cheque paid out but not yet on the statement. In addition the
cash book shows deposit interest received of $50 but this is not yet on the
statement.

What is the balance per the cash book?

A. $1,900 overdrawn
B. $500 overdrawn
C. $1,900 in hand
D. $500 in hand

___________________________________________________________________________

228
Lecture Example 4

The following information relates to a bank reconciliation.


(i) The bank balance in the cashbook before taking the items below into
account was $8,970 overdrawn.
(ii) Bank charges of $550 on the bank statement have not been entered in the
cashbook.
(iii) The bank has credited the account in error with $425 which belongs to
another customer.
(iv) Cheque payments totalling $3,275 have been entered in the cashbook but
have not been presented for payment.
(v) Cheques totalling $5,380 have been correctly entered on the debit side of
the cashbook but have not been paid in at the bank.

What was the balance as shown by the bank statement before taking the items
above into account?

_______________________

___________________________________________________________________________

229
__________________________

KEY NOTES
_________________

___________________________________________________________________________

230
1. The cash book is used to record the detailed transactions of receipts and
payments affecting the bank account. These are then posted to the nominal
ledger periodically. At the end of each accounting period, the balance on the
cash book should equal the balance in the nominal ledger cash/bank account.
This figure should also agree with the balance on the bank statement.

2. The balance on the cash account/cash book is compared to the balance on


the bank statements at a given date. However, these two balances may not
agree. Differences between the cash book and the bank statement arise for 3
reasons: -
1. Errors – usually in the cash book
2. Omissions – such as bank charges, standing orders and direct debits
not posted in the cash book
3. Timing differences – such as unpresented cheques and unrecorded
lodgements

3. The bank reconciliation is produced after checking that all the items on the
bank statement have been recorded in the cash book. Any items not in the
cash book will need to be recorded. The balance per bank statement must be
adjusted for any timing differences or errors by the bank.

4. Bank reconciliation statement – an example: -

Cash account (cash book)


Balance b/d X Dishonoured cheque X
Bank charges X
Standing orders X
Direct debits X
Balance c/d X
X X

$
Balance per bank statement X RECONCILE
less unpresented cheques (X)
plus unrecorded lodgements X
plus/less bank errors X
Balance per adjusted cash book X

___________________________________________________________________________

231
__________________________

QUESTION BANK
_________________

___________________________________________________________________________

232
1. The following attempt at a bank reconciliation statement has been prepared
by Q Co:
$
Overdraft per bank statement 38,600
Add: deposits not credited 41,200
79,800
Less: outstanding cheques 3,300
Overdraft per cash book 76,500

Assuming the bank statement balance of $38,600 to be correct, what should


the cash book balance be?

A. $76,500 overdrawn, as stated


B. $5,900 overdrawn
C. $700 overdrawn
D. $5,900 cash at bank

2. After checking a business cash book against the bank statement, which of the
following items could require an entry in the cash book?

1. Bank charges
2. Cheque from a customer which was dishonoured
3. Cheque not presented
4. Deposits not credited
5. Credit transfer entered in bank statement
6. Standing order entered in bank statement.

A. 1, 2, 5 and 6
B. 3 and 4
C. 1, 3, 4 and 6
D. 3, 4, 5 and 6

3. At 30 April 20X8 the balance on the bank account in Jim’s general ledger
showed that he had $685 cash at the bank. When he carried out his bank
reconciliation, he found that he had omitted bank charges of $722 for the year
to 30 April 20X8.

What bank balance should be included on Jim’s opening trial balance at 1


May 20X8?

A. $685 debit
B. $685 credit
C. $37 debit
D. $37 credit

___________________________________________________________________________

233
4. The following bank reconciliation statement has been prepared for a
company:
$
Overdraft per bank statement 39,800
Add: Deposits credited after date 64,100
103,900
Less: Outstanding cheques presented after date 44,200
Overdraft per cash book 59,700

Assuming the amount of the overdraft per the bank statement of $39,800 is
correct, what should be the balance in the cash book?

A. $158,100 overdrawn
B. $19,900 overdrawn
C. $68,500 overdrawn
D. $59,700 overdrawn as stated

5. Sigma's bank statement shows an overdrawn balance of $38,600 at 30 June


20X5. A check against the company's cash book revealed the following
differences:
1. Bank charges of $200 have not been entered in the cash book.
2. Lodgements recorded on 30 June 20X5 but credited by the bank on 2
July $14,700.
3. Cheque repayments entered in cash book but not presented for
payment at 30 June 20X5 $27,800.
4. A cheque payment to a supplier of $4,200 charged to the account in
June 20X5 recorded in the cash book as a receipt.
Based on this information, what was the cash book balance before any
adjustments?

A. $43,100 overdrawn
B. $16,900 overdrawn
C. $60,300 overdrawn
D. $34,100 overdrawn

6. When doing bank reconciliation, adjustments have to be made for timing


differences.
Which one of these constitutes a timing difference?

A. Unpresented cheques
B. Unposted direct debits
C. Bank charges
D. Dishonoured cheques

___________________________________________________________________________

234
7. Elaine is preparing her bank reconciliation. She has noted the following:
(i) the bank has levied charges on her account
(ii) a cheque payable to S. Wright has not been presented at the bank

Which of the above errors require an entry in the bank account in her general
ledger?

A. both (i) and (ii)


B. (i) only
C. neither (i) nor (ii)
D. (ii) only

8. The following bank reconciliation statement has been prepared by an


inexperienced bookkeeper at 31 December 20X5.

Bank reconciliation statement


$
Balance per bank statement (overdrawn) 38,640
Add: lodgements not credited 19,270
57,910
Less: unpresented cheques 14,260
Balance per cash book 43,650

What should the final cash book balance be when all the above items have
been properly dealt with?

A. $43,650 overdrawn
B. $33,630 overdrawn
C. $5,110 overdrawn
D. $72,170 overdrawn

9. Which of the following statements about bank reconciliations are correct?


1. In preparing a bank reconciliation, unpresented cheques must be
deducted from a balance of cash at bank shown in the bank statement.
2. A cheque from a customer paid into the bank but dishonoured must be
corrected by making a debit entry in the cash book.
3. An error by the bank must be corrected by an entry in the cash book.
4. An overdraft is a debit balance in the bank statement.

A. 1 and 3
B. 2 and 3
C. 1 and 4
D. 2 and 4

___________________________________________________________________________

235
__________________________

ANSWER BANK
_________________

___________________________________________________________________________

236
1. C
The bank is overdrawn.
$
Overdraft (38,600)
Deposits 41,200
2,600
Unpresented cheques (3,300)
Overdraft (700)

2. A - The other two items are part of the bank reconciliation.

3. D

Cash at bank $685


Less bank changes ($722)
$37 credit (bank o’draft)

4. B

$
Overdraft per bank statement 39,800
Less: deposits credited (64,100)
Add: outstanding cheques 44,200
Overdraft per cash book 19,900

5. A

$
Balance per bank statement (38,600)
Bank charges 200
Lodgements 14,700
Cheque payments (27,800)
Cheque payment misposted 8,400
Balance per cash book (43,100)

6. A
All of the others will require an entry in the cash book

7. B

___________________________________________________________________________

237
8. B

Bank reconciliation statement

Balance per bank (overdrawn) (38,640)


Add: outstanding lodgements 19,270
(19,370)
Less: unpresented cheques (14,260)
Balance per cash book (overdrawn) (33,630)

9. C - Statements 2 and 3 are incorrect. A dishonoured cheque is credited to the


cash book and bank errors do not go through the cash book at all

___________________________________________________________________________

238
Chapter 16
Correction of Errors

16.1 ACCA SYLLABUS GUIDE OUTCOME 1


Identify the types of error which may occur in bookkeeping systems

The following are five frequent types of error: -

1. Errors of transposition: - when two digits in an amount are accidentally


recorded the wrong way round.
2. Errors of omission: - failing to record a transaction at all, or making a debit
or credit entry, but not the corresponding double entry.
3. Errors of principle: - making a double entry in the belief that the transaction
is being entered in the correct accounts, but subsequently finding out that the
accounting entry breaks the 'rules' of an accounting principle or concept.
4. Errors of commission: - where the bookkeeper makes a mistake in carrying
out his or her task of recording transactions in the accounts. Two examples
are: - putting a debit/credit entry in the wrong account; errors of casting
(adding up)
5. Compensating errors: - errors which are, coincidentally, equal and opposite
to one another.

Lecture Example 1

Type of error Example


1. Errors of transposition

2. Errors of omission

3. Errors of principle

4. Errors of commission

5. Compensating errors

Some of these errors can be corrected by journal entry; some require the use of a
suspense account.

1. If the correction involves a double entry in the ledger accounts, then it is done
by using a journal entry in the journal.

___________________________________________________________________________

239
2. When the error breaks the rule of double entry (single entry or error on one
side only), then it is corrected by the use of a suspense account as well as a
journal entry.
16.2 ACCA SYLLABUS GUIDE OUTCOME 2
Identify errors which would be highlighted by the extraction of a trial balance

Errors that can be detected by a trial balance include: -


 Errors of transposition
 Errors of omission (if the omission is one-sided)
 Errors of commission (for e.g. if one-sided, or two debit entries are made)

Other errors will not be detected by extracting a trial balance, but may be spotted by
other controls (such as bank or control account reconciliations).

16.3 ACCA SYLLABUS GUIDE OUTCOME 3


Prepare journal entries to correct errors

Suspense Accounts: -

A suspense account is a temporary account. It never appears in the final accounts.

It is used for two main reasons:


1. To account for a debit or credit entry when the accountant is unsure as to
where it should go
2. To make a preliminary trial balance balance when an error has been detected.

Steps to clear a suspense account: -

1. Determine the original accounting entry which was made.


2. Decide what entry should have been made.
3. Make the required adjustment.

Lecture Example 2

The trial balance of Z failed to agree, the totals being: debit $836,200 credit
$819,700.
A suspense account was opened for the amount of the difference and the following
errors were found and corrected:
1. The totals of the cash discount columns in the cash book had not been posted
to the discount accounts. The figures were discount allowed $3,900 and
discount received $5,100.
2. A cheque for $19,000 received from a customer was correctly entered in the
cash book but was posted to the customer's account as $9,100.

___________________________________________________________________________

240
What will be the remaining balance on the suspense be after the correction of these
errors?

A. $25,300 credit
B. $7,700 credit
C. $27,700 debit
D. $5,400 credit

Lecture Example 3

Which of the following errors could result in a suspense account being required to
'balance' the trial balance?

A. Cash received from credit customers treated as a cash sale


B. A supplier's invoice for $32 recorded as $23 in the purchases account
C. Payments to suppliers of $647 recorded as $674 in the payables ledger
D. One page omitted from the purchase day book

16.4 ACCA SYLLABUS GUIDE OUTCOME 4


Calculate and understand the impact of errors on the statement of profit or
loss, statement of comprehensive income and statement of financial position

When errors are corrected they may affect the business' profit for the year figure. In
order to find the correct figure for profit, a statement of adjustments to profit has to
be prepared.

Proforma – Statement of Adjustment to Profit

$ $ $
+ -
Original profit X
Adjustment:
Over depreciation expense charged X
Unrecorded expense X
Unrecorded sale X
X (X) X
Adjusted Profit X

___________________________________________________________________________

241
Lecture Example 4

At the year end of T Down & Co, an imbalance in the trial balance was revealed
which resulted in the creation of a suspense account with a credit balance of $1,040.

Investigations revealed the following errors.

1. A sale of goods on credit for $1,000 had been omitted from the sales account.
2. Delivery and installation costs of $240 on a new item of plant had been
recorded as a revenue expense.
3. Cash discount of $150 on paying a supplier, JW, had been taken, even
though the payment was made outside the time limit.
4. Purchases of stationery at the end of the period of $240 had been ignored.
5. A purchase of raw materials of $350 had been recorded in the purchases
account as $850.
6. The purchase returns day book included a sales credit note for $230 which
had been entered correctly in the account of the customer concerned, but
included with purchase returns in the nominal ledger.

Required:

i. Prepare journal entries to correct each of the above errors. Narratives


are not required.
ii. Open a suspense account and show the corrections to be made.
iii. Prior to the discovery of the errors, T Down & Co's gross profit for the
year was calculated at $35,750 and the net profit for the year at
$18,500. Calculate the revised gross and net profit figures after the
correction of the errors.

Lecture Example 5

The bookkeeping system of MVP Limited is not computerised, and at 30 September


20X8 the bookkeeper was unable to balance the accounts. The trial balance totals
were: -

Debit $1,796,100
Credit $1,852,817

Nevertheless, he proceeded to prepare draft financial statements, inserting the


difference as a balancing figure in the balance sheet. The draft profit and loss
account showed a profit of $141,280 for the year ended 30 September 20X8.

___________________________________________________________________________

242
He then opened a suspense account for the difference and began to check through
the accounting records to find the difference. He found the following errors and
omissions:

1. $8,980 – the total of the sales returns book for September 20X8, had been
credited to the purchases returns account.
2. $9,600 paid for an item of plant purchased on 1 April 20X8 had been debited
to plant repairs account. The company depreciates its plant at 20% per
annum on a straight line basis, with proportional depreciation in the year of
purchase.
3. The cash discount totals for the month of September 20X8 had not been
posted to the nominal ledger accounts. The figures were:
Discount allowed $836
Discount received $919
4. $580 insurance prepaid at 30 September 20X7 had not been brought down as
an opening balance
5. The balance of $38,260 on the telephone expense account had been omitted
from the trial balance
6. A car held as a fixed asset had been sold during the year for $4,800. The
proceeds of sale were entered in the cash book but had been credited to the
sales account in the nominal ledger. The original cost of the car $12,000, and
the accumulated depreciation to date $8,000, were included in the motor
vehicles account and the accumulated depreciation account. The company
depreciates motor vehicles at 25% per annum on a straight line basis with
proportionate depreciation in the year of purchase but none in the year of
sale.

Required:

1. Open a suspense account for the difference between the trial balance
totals. Prepare the journal entries necessary to correct the errors and
eliminate the balance on the suspense account. Narratives are not
required.
2. Draw up a statement showing the revised profit after correcting the above
errors.

___________________________________________________________________________

243
___________________________

KEY NOTES
_________________

___________________________________________________________________________

244
1. Types of errors: -

1. Errors of transposition: - when two digits in an amount are accidentally


recorded the wrong way round.
2. Errors of omission: - failing to record a transaction at all, or making a debit
or credit entry, but not the corresponding double entry.
3. Errors of principle: - making a double entry in the belief that the transaction
is being entered in the correct accounts, but subsequently finding out that the
accounting entry breaks the 'rules' of an accounting principle or concept.
4. Errors of commission: - where the bookkeeper makes a mistake in carrying
out his or her task of recording transactions in the accounts. Two examples
are: - putting a debit/credit entry in the wrong account; errors of casting
(adding up)
5. Compensating errors: - errors which are, coincidentally, equal and opposite
to one another.

2. Suspense Account

Where the trial balance does not balance a suspense account will be opened and
the errors, once identified, will be corrected via a journal entry. A suspense
account should never appear in the final accounts.

Table 1: Types or error and suspense account

Error type Suspense account involved?


1. Compensating errors – two equal and opposite errors leave
Yes, to correct
the trial balance balancing (this type of error is rare, and can be
each of the errors
because a deliberate second error has been made to force the
as discovered
balancing of the records or to conceal a fraud)
2. Omission – a transaction is not recorded at all No
3. Error of principle – an item is posted to the correct side of
the wrong type of account, as when cash paid for plant repairs
(expense) is debited to plant account (asset) No
(errors of principle are really a special case of errors of
commission, and once again there is a debit and a credit)
4. Error of commission – an item is entered to the correct side
of the wrong account (there is a debit and a credit here, so the No
records balance)
5. Transposition error – an entry in one record is incorrectly
posted to another Examples: cash £10,000 entered in the cash
Yes
book for the purchase of a car is: posted to Motor cars account
as £1,000

___________________________________________________________________________

245
Note: when a balance is omitted, or incorrectly extracted, in preparing a trial
balance, the suspense account is involved.

3. Adjustments to Profit

Where the process of correcting errors requires changes to income and expense
accounts the business’ profit will be affected. In this case a statement of
adjustments to profit has to be prepared to determine the revised profit figure for
the year.

___________________________________________________________________________

246
__________________________

QUESTION BANK
_________________

___________________________________________________________________________

247
1. Sales returns of $460 have inadvertently been posted to the purchase returns,
although the correct entry has been made to the accounts receivable control.

A suspense account needs to be set up for how much?

A. $460 debit
B. $460 credit
C. $920 debit
D. $920 credit

2. The trial balance of a company did not balance, and a suspense account was
opened for the difference.

Which of the following errors would require an entry to the suspense


account in correcting them?

1. A cash payment to purchase a motor van had been correctly entered in


the cash book but had been debited to motor expenses account.
2. The debit side of the wages account had been under cast.
3. The total of the discounts allowed column in the cash book had been
credited to discounts received account.
4. A cash refund to a customer had been recorded by debiting the cash book
and crediting the customer’s account.

A. 1 and 2
B. 2 and 3
C. 3 and 4
D. 2 and 4

3. Which of the following errors would cause a trial balance not to


balance?

1. An error in the addition of the cash book.


2. Failure to record a transaction at all.
3. Cost of a motor vehicle debited to motor expenses account. The cash
entry was correctly made.
4. Goods taken by the proprietor of a business recorded by debiting
purchases and crediting drawings account.

A. 1 only
B. 1 and 2 only
C. 3 and 4 only
D. All four items
___________________________________________________________________________

248
4. The draft statement of profit or loss of Lorca, a limited liability company,
showed a profit of $830,000. However, the trial balance did not balance and a
suspense account with a credit balance of $20,000 has been included in the
statement of financial position

The following errors were found on investigation:

1. The proceeds of issue of 100,000 50c shares at 70c per share were
correctly entered in the cash book but had been credited to sales
account.
2. During the year $8,000 interest received on a holding of loan notes had
been correctly entered in the cash book but debited to interest payable
account.
3. In arriving at the net sales and purchases totals for the year, the
$48,000 balance on the returns outwards account had been transferred
to the debit of sales account and the $64,000 balance on the returns
inwards account had been transferred to the credit of purchases
account.
4. A payment of $4,000 for rent had been correctly recorded in the cash
book but debited to the rent account as $40,000.

Required:

a. Prepare journal entries to correct the errors. Narratives are NOT


required.
b. Find the revised net profit for the year

5. A company’s trial balance failed to agree, and a suspense account was opened
for the difference.

Subsequent checking revealed that discounts allowed $13,000 had been credited
to discounts received account and an entry on the credit side of the cash book for
the purchase of some machinery $18,000 had not been posted to the plant and
machinery account.

Which two of the following journal entries would correct the errors?
Debit Credit
$ $
1. Discounts allowed 13,000
Discounts received 13,000
2. Discounts allowed 13,000
Discounts received 13,000
Suspense account 26,000
3. Suspense account 26,000
___________________________________________________________________________

249
Discounts allowed 13,000
Discounts received 13,000
4. Plant and machinery 18,000
Suspense account 18,000
5. Suspense account 18,000
Plant and machinery 18,000

A. 1 and 4
B. 2 and 5
C. 2 and 4
D. 3 and 5

6. A company’s draft financial statements for 20X5 showed a profit of $630,000.


However, the trial balance did not agree, and a suspense account appeared in
the company’s draft balance sheet.

Subsequent checking revealed the following errors:

1. The cost of an item of plant $48,000 had been entered in the cash book and in
the plant account as $4,800. Depreciation at the rate of 10% per year ($480)
had been charged.
2. Bank charges of $440 appeared in the bank statement in December 20X5 but
had not been entered in the company’s records.
3. One of the directors of the company paid $800 due to a supplier in the
company’s payables ledger by a personal cheque. The bookkeeper recorded a
debit in the supplier’s ledger account but did not complete the double entry for
the transaction. (The company does not maintain a payables ledger control
account).
4. The payments side of the cash book had been understated by $10,000.

Which of the above items would require an entry to the suspense account in
correcting them?

A. All four items


B. 3 and 4 only
C. 2 and 3 only
D. 1, 2 and 4 only

7. With reference to question 6, what would the company’s profit become


after the correction of the above errors?

A. $634,760
B. $624,760
C. $624,440
D. $625,240
___________________________________________________________________________

250
__________________________

ANSWER BANK
_________________

___________________________________________________________________________

251
1. C

The sales returns of $460 have been credited to accounts receivable and also $460
has been credited to purchase returns. Therefore the trial balance needs a debit of
2 × $460 = $920 to balance.

2. B

1. Dr Motor vehicles
Cr Motor expenses

2. Dr Wages
Cr Suspense

3. Dr Discounts received
Dr Discounts allowed
Cr Suspense

4. Dr Receivables
Cr Cash

3. A

The trial balance still agrees if there is an error of omission, commission, principle or
complete reversal of entries.

4. (a)

Dr Cr
$ $
1. Sales 70,000
Share capital 50,000
Share premium 20,000
2. Suspense 16,000
Interest payable 8,000
Interest receivable 8,000
3. Sales 16,000
Purchases 16,000
Suspense 32,000
OR
Suspense 48,000
Sales 48,000
Purchases 64,000
Suspense 64,000
Sales 64,000
Suspense 64,000
___________________________________________________________________________

252
Suspense 48,000
Purchases 48,000
4. Suspense 36,000
Rent 36,000

(b)
$ $ $
+ -
Original profit 830,000
Adjustment:
Sales 70,000
Interest 16,000
Sales/purchases 32,000
Rent 36,000
52,000 (102,000) (50,000)
Adjusted Profit 780,000

5. C

6. B

1. Dr Plant (48,000 – 4,800) 43,200


Cr Cash 43,200

Dr Depreciation expense 4,320


Cr Accounts depreciation 4,320

2. Dr Bank charges 440


Cr Bank 440

3. Dr Suspense 800
Cr Directors’ account 800

4. Dr Suspense 10,000
Cr Cash 10,000

7. D

+ -
$ $ $

Unadjusted profit 630,000


Depreciation 4,320
Bank charges 440
0 (4,760) (4,760)
Adjusted profit 625,240

___________________________________________________________________________

253
Chapter 17
Incomplete Records

17.1 ACCA SYLLABUS GUIDE OUTCOME 1:


Understand and apply techniques used in incomplete record situations:
i. Use of accounting equation
ii. Use of ledger accounts to calculate missing figures
iii. Use of cash and/or bank summaries
iv. Use of profit percentages to calculate missing figures.

Incomplete records problems occur when a business does not have a full set of
accounting records, for one of the following reasons.
 The proprietor of the business does not keep a full set of accounts.
 Some of the business accounts are accidentally lost or destroyed.

Different techniques can be used to find the missing information: -

17.1.1 Use of accounting equation

It is still possible to calculate a profit or loss figure by using the fact that the profit of a
business must be represented by more assets. We list and value the opening and
closing net assets, then calculate the profit as the difference between the two

Profit = Closing net assets - Opening net assets

Allowance must be made for proprietor's drawings and extra capital introduced, so
the formula becomes:

Profit = Closing net assets - Opening net assets + Drawings - Capital


introduced

Lecture Example 1

A business has net assets of $70,000 at the beginning of the year and $80,000 at
the end of the year. Drawings were $25,000 and a lottery win of $5,000 was paid into
the business during the year. What was the profit for the year?

A. $10,000 loss
B. $30,000 profit
C. $10,000 profit
D. $30,000 loss
___________________________________________________________________________

254
17.1.2 Control Accounts

A receivables ledger control account can be prepared to calculate missing credit


sales. However, the figures for the opening and closing receivables of a business
and the cash received from customers must be given.

Receivables Ledger Control A/c


$ $
Opening balance 38,600 Cash received 218,650
Sales (balancing figure) 221,250 Closing balance 41,200

259,850 259,850

The same technique can be used to calculate credit purchases. A payables ledger
control account can be prepared using given figures for opening and closing
payables and cash paid.

Note: -

Total sales = Cash sales + Credit sales


Total purchases = Cash purchases + Credit purchases

Lecture Example 2

Senji does not keep proper accounting records, and it is necessary to calculate her
total purchases for the year ended 31 January 20X4 from the following information:

$
Trade payables
31 January 20X3 130,400
31 January 20X4 171,250
Payments to suppliers 888,400
Cost of goods taken by Senji for her personal use 1,000
Refund received from suppliers 2,400
Discounts received 11,200

Compute the figure for purchases for inclusion in Senji's financial statements.

Lecture Example 3

The following information is available for the year ended 31 December 20X1 for Ski,
a well-run company:

___________________________________________________________________________

255
$
Opening cash 1,000
Closing cash 2,000
Opening balance on the trade payables control account 8,000
Closing balance on the trade payables control account 10,000
Opening balance on the trade receivables control account 12,000
Closing balance on the trade receivables control account 14,000
Cash paid to trade accounts payable in the period 9,000
Opening inventory 6,000
Closing inventory 7,000

Mark-up on cost - 10%

Assuming the information above is complete, what was the sales figure for the
period?

____________

17.1.3 Cash/Bank

A cash account may need to be set up to find the figure missing for proprietor’s
drawings or cash stolen. Details of cash receipts and payments plus details of
opening and closing balances must be given.

Lecture Example 4

B Co maintains a cash float of $50. In 20X7, all receipts from credit customers were
banked, after the following payments from the till had been made:

$
General expenses 4,500
Drawings 6,250

Total banking in the year amounted to $28,454, and opening and closing trade
receivables were $1,447 and $1,928 respectively.

Required

Based on the information above what was the value of sales made during the year?

___________________________________________________________________________

256
17.1.4 Cost Structure

Margin: gross profit is expressed as a percentage of sales

For example a margin of 25% gives:

Sales 100%
Cost of sales 75%
Gross profit 25%

Mark-up: gross profit is expressed as a percentage of cost of sales,

For example a mark-up of 35% gives:

Sales 135%
Cost of sales 100%
Gross profit 35%

Cost of sales = opening inventories + purchases – closing inventories

Lecture Example 5

Aluki fixes prices to make a standard gross profit percentage on sales of 33 1/3%. The
following information is available for the year ended 31 January 20X4 to compute her
sales total for the year:

$
Inventory
1 February 20X3 243,000
31 January 20X4 261,700
Purchases 595,400
Purchases returns 41,200

Calculate the sales figure for the year ended 31 January 20X4.

Lecture Example 6

A business usually has a mark-up of 20% on cost of sales. During a year, its sales
were $90,000. What was the cost of sales?

__________

___________________________________________________________________________

257
Lecture Example 7

Brown has budgeted sales for the coming year of $175,000. He achieves a constant
gross mark-up of 40% on cost. He plans to reduce his inventory level by $13,000
over the year. What will Brown's purchases be for the year?

17.1.5 Goods Drawn By Proprietor

The owners of the business may at times take goods or cash from the business for
their own use. This is known as drawings.

Cash drawings

Dr Drawings
Cr Cash

Goods taken for own use

Dr Drawings
Cr Purchases

These are recorded at the cost to the business not at selling price. They are taken
out of purchases and not recorded against inventories.

Lecture Example 8

A business has opening inventories of $273 and makes purchases during the year of
$2,781. The proprietor removes goods costing $87 for his own use. The business
achieves a constant mark-up of 20% on cost and records sales for the year of
$3,360.

What is the cost of closing inventories?

______________

17.1.6 Goods destroyed, stolen or lost

When inventory is stolen, destroyed or otherwise lost, the loss must be accounted for
depending on whether or not these goods were insured against the loss.
___________________________________________________________________________

258
If the lost goods were not insured,

Debit expense (e.g. admin expenses in the I/S)


Credit cost of sales
If the lost goods were insured,

Debit insurance claim account (current asset in SFP)


Credit cost of sales

Lecture Example 9

On 1 September 20X6, a business had inventory of $380,000. During the month,


sales totalled $650,000 and purchases $480,000. On 30 September 20X6 a fire
destroyed some of the inventory. The undamaged goods in inventory were valued at
$220,000. The business operates with a standard gross profit margin of 30%.

Based on this information, what is the cost of the inventory destroyed in the
fire?

A. $185,000
B. $140,000
C. $405,000
D. $360,000

___________________________________________________________________________

259
__________________________

KEY NOTES
_________________

___________________________________________________________________________

260
1. Incomplete records problems occur when a business does not have a full set of
accounting records, for one of the following reasons.

 The proprietor of the business does not keep a full set of accounts.
 Some of the business accounts are accidentally lost or destroyed.

2. Different techniques can be used to find the missing information: -

2.1 Use of accounting equation

Profit = Closing net assets - Opening net assets

Allowance must be made for proprietor's drawings and extra capital introduced.

Profit = Closing net assets - Opening net assets + Drawings - Capital


introduced

2.2 Control Accounts

A receivables ledger control account can be prepared to calculate missing credit


sales. However, the figures for the opening and closing receivables of a business
and the cash received from customers must be given.

The same technique can be used to calculate credit purchases. A payables ledger
control account can be prepared using given figures for opening and closing
payables and cash paid.

2.3 Cash/Bank

A cash account may need to be set up to find the figure missing for proprietor’s
drawings or cash stolen .Details of cash receipts and payments plus details of
opening and closing balances must be given.

2.4 Cost Structure

Margin: gross profit is expressed as a percentage of sales

For example a margin of 25% gives:

Sales 100%
Cost of sales 75%
Gross profit 25%

___________________________________________________________________________

261
Mark-up: gross profit is expressed as a percentage of cost of sales,

For example a mark-up of 35% gives:

Sales 135%
Cost of sales 100%
Gross profit 35%

Cost of sales = opening inventories + purchases – closing inventories

3. Goods Drawn By Proprietor

Cash drawings

Dr Drawings
Cr Cash

Goods taken for own use

Dr Drawings
Cr Purchases

These are recorded at the cost to the business. They are taken out of purchases
and not recorded against inventories.

___________________________________________________________________________

262
__________________________

QUESTION BANK
_________________

___________________________________________________________________________

263
1. The net assets of Altese, a trader, at 1 January 20X3 amounted to $128,000.
During the year to 31 December 20X3, Altese introduced a further $50,000 of
capital and made drawings of $48,000. At 31 December 20X3, Altese's net
assets totalled $184,000. Using this information compute Altese's total profit for
the year ended 31 December 20X3.

2. The profit earned by a business in 20X7 was $72,500. The proprietor injected
new capital of $8,000 during the year and withdrew goods for his private use
which had cost $2,200.
If net assets at the beginning of 20X7 were $101,700, what were the closing net
assets?

A. $35,000
B. $39,400
C. $168,400
D. $180,000

3. A business has compiled the following information for the year ended 31 October
20X2:
$
Opening inventory 386,200
Purchases 989,000
Closing inventory 422,700
The gross profit as a percentage of sales is always 40%

Based on these figures, what is the sales revenue for the year?

A. $1,333,500
B. $1,587,500
C. $2,381,250
D. The sales revenue figure cannot be calculated from this information

4. A sole trader took some goods costing $800 from inventory for his own use. The
normal selling price of the goods is $1,600.
Which of the following journal entries would correctly record this?

A. Dr Inventories $800
Cr Purchases $800

B. Dr Drawings $800
Cr Purchases $800

C. Dr Sales $1,600
Cr Drawings $1,600
___________________________________________________________________________

264
D. Dr Drawings $800
Cr Sales $800
5. A sole trader fixes her prices by adding 50 per cent to the cost of all goods
purchased. On 31 October 20X3, a fire destroyed a considerable part of the
inventory and all inventory records.

Her trading account for the year ended 31 October 20X3 included the following
figures:
$ $
Sales 281,250
Cost of Sales
Opening Inventories 183,600
Purchases 249,200
432,800
Closing Inventories 204,600
Cost of Sales 228,200
Gross Profit 53,050

Using this information, what inventory loss has occurred?

A. $61,050
B. $87,575
C. $40,700
D. $110,850

6. Adam, a sole trader has net assets at 31 December 20X1 of $65,250. During the
year he made a loss of $3,000, he took inventory for his own use of $850 and
removed cash of $2,250.

If he introduced capital of $5,000 during the year, what was the capital as at 1
January 20X1?

______________

7. On 30 April 20X1 part of the inventory of Neutron, a limited liability company, was
destroyed by fire.

The following information is available.


• Inventory at 1 April 20X1 $99,600
• Purchases for April 20X1 $177,200
• Sales for April 20X1 $260,000
• Inventory at 30 April 20X1 – undamaged items $64,000
• Standard gross profit percentage on sales 30%

Based on this information, what was the cost of the inventory destroyed?
___________________________________________________________________________

265
_________________

ANSWER BANK
_________________

___________________________________________________________________________

266
1.
$
Opening capital 128,000
Capital introduced 50,000
178,000
less: Drawings 48,000
130,000
Closing capital 184,000
Profit is therefore 54,000

2. D
I = P + Ci – D
= $(72,500 + 8,000 – 2,200)
= $78,300
Therefore, closing net assets = $(101,700 + 78,300) = $180,000.

3. B
Opening inventory 386,200
Purchases 989,000
Closing inventory (422,700)
Cost of sales 952,500

952,500 × 100/60 = 1,587,500

4. B - The selling price is not relevant to this adjustment.

5. C - Cost of sales = $281,250 × 2/3 = $187,500


Loss of inventory = $228,200 – 187,500 = $40,700

6. $66,350

Net assets = capital


$'000
Capital at 1.1.X1 (balancing figure) 66,350
Additional capital 5,000
71,350
Drawing (cash) (2,250)
(inventory) (850)
Loss (3,000)
Net assets @ 31.12.X1 65,250

7. $30,800
$
Theoretical gross profit 30% × $260,000 78,000
Actual gross profit:
$260,000 – $99,600 – $177,200 + $64,000 47,200
Shortfall – missing inventory 30,800
___________________________________________________________________________

267
Chapter 18
Provisions and Contingencies
IAS 37 – Provisions, Contingent Liabilities and Contingent Assets

18.1 ACCA SYLLABUS GUIDE OUTCOMES: 1- 6


Understand the definition of “provision”, “contingent liability” and “contingent
asset”
Distinguish between and classify items as provisions, contingent liabilities or
contingent assets
Identify and illustrate the different methods of accounting for provisions,
contingent liabilities and contingent assets
Calculate provisions and changes in provisions
Account for the movement in provisions
Report provisions in the final accounts

18.1.1 Provisions

A provision is a liability of uncertain timing or amount.

IAS 37 requires a provision be recognised when all of the following apply:

1. an entity has a present obligation (legal or constructive) as a result of a past


event
2. it is probable that an outflow of resources embodying economic benefits will
be required to settle the obligation
3. a reliable estimate can be made of the amount of the obligation

Therefore, a provision is made for something which will probably happen. It should
be recognised when it is probable that a transfer of economic events will take place
and when its amount can be estimated reliably.

Provisions can be distinguished from other liabilities (e.g. trade payables and
accruals) due to the uncertainty concerning the timing or amount of the future
expenditure required in settlement. In contrast, trade payables are liabilities to pay
for goods that have been received and invoiced, hence the timing and amount of the
expenditure is agreed with the supplier.

A provision is accounted for as follows: -

Dr Expense (I/S)
Cr Provision (SOFP)

___________________________________________________________________________

268
The required provision will be reviewed at each year end and increased or
decreased as necessary.
To increase a provision:

Dr Expense (I/S)
Cr Provision (SOFP)

To decrease a provision:

Dr Provision (SOFP)
Cr Expense (I/S)

Measurement of Provision

The amount recognised as a provision should be the best estimate of the


expenditure required to settle the present obligation at the end of the reporting
period.

Provisions for one-off events (restructuring, environmental clean-up, settlement of a


lawsuit) are measured at the most likely amount.

Provisions for large populations of events (warranties, customer refunds) are


measured at a probability-weighted expected value.

Worked out example

A company sells goods with a warranty for the cost of repairs required in the first 2
months after purchase.

Past experience suggests:


1. 88% of the goods sold will have no defects
2. 7% will have minor defects
3. 5% will have major defects

If minor defects were detected in all products sold, the cost of repairs will be
$24,000; if major defects were detected in all products sold, the cost would be
$200,000.

What amount of provision should be made?

(88% x 0) + (7% x 24,000) + (5% x 200,000) = $11,680.

Lecture Example 1

___________________________________________________________________________

269
A business has been told by its lawyers that it is likely to have to pay $10,000
damages for a product that failed. The business duly set up a provision at 31
December 20X7. However, the following year, the lawyers found that damages were
more likely to be $50,000. How is the provision treated in the accounts at:
(a) 31 December 20X7?
(b) 31 December 20X8?

Disclosure note

For each class of provision, an entity should disclose: -


1. the net book value at the beginning and the end of the period
2. additional provisions made in the period, including increases to existing
provisions
3. amounts utilised during the period
4. unused amounts reversed during the period

An entity should also disclose, for each class of provision: -


1. a brief description of the nature of the obligation and the expected timing of
any resulting outflows of economic benefits
2. an indication about the uncertainties about the amount and timing of those
outflows
3. the amount of any expected reimbursement, stating the amount of any asset
that has been recognised for that expected reimbursement

18.1.2 Contingent Liabilities

Contingent liabilities are:


(a) possible obligations that arise from past events and whose existence will be
confirmed only by the occurrence or nonoccurrence of one or more uncertain
future events not wholly within the control of the entity
(b) present obligations that arise from past events but are not recognised
because:
i. they are not probable that an outflow of resources embodying
economic benefits will be required to settle the obligation; or
ii. the amount of the obligation cannot be measured with sufficient
reliability

Contingent liabilities should not be recognized in financial statements but they should
be disclosed, unless the possibility of any outflow is remote. The required
disclosures are:
 A brief description of the nature of the contingent liability;
 An estimate of its financial effect;
 An indication of the uncertainties that exist relating to the amount or timing
of any outflow; and
 The possibility of any reimbursement.

___________________________________________________________________________

270
Disclosure Note

Unless the possibility of any outflow is remote, for each class of contingent liability,
an entity should disclose at the end of the reporting period, a brief description of the
nature of the contingent liability and where practicable: -
1. an estimate of its financial effect
2. an indication of the uncertainties relating to the amount or timing of any
outflow; and
3. the possibility of any reimbursement

Lecture Example 2

Amazon Inc. has been sued for the following two alleged infringement of law:
1. unauthorized use of a trademark; the claim is for $100 million
2. non-payment of end-of-service severance pay and gratuity to 5,000
employees who were terminated without Amazon Inc. giving any reason; the
class action lawsuit is claiming $3 million.

Legal counsel has communicated to Amazon Inc. this assessment of the two
lawsuits:

Lawsuit 1: the chances of this lawsuit are remote


Lawsuit 2: it is probable that Amazon Inc. would have to pay the displaced
employees, but the best estimate of the amount that would be payable if the plaintiff
succeeds against the entity is $2 million.

What accounting treatment is required for these 2 lawsuits?

18.1.3 Contingent assets

Contingent assets are possible assets that arise from past events and whose
existence will be confirmed only by the occurrence or nonoccurrence of one or more
uncertain future events not wholly within the control of the entity.

A contingent asset must not be recognized. Only when the realization of the related
economic benefits is virtually certain should recognition take place. At that point, the
asset is no longer a contingent asset!

Contingent assets must only be disclosed in the notes if they are probable. A brief
description of the contingent asset must be provided together with an estimate of its
financial effect and details of any uncertainties.

___________________________________________________________________________

271
Disclosure Note

Where an inflow of economic benefits is probable (contingent asset), an entity should


disclose a brief description of the nature of the contingent assets at the end of the
reporting period and, where practicable, an estimate of their financial effect.

Lecture Example 3

How does a company account for a contingent asset that is not probable?

A. By way of note
B. As an asset in the statement of financial position
C. It does nothing

Lecture Example 4

An employee dismissed in August 20X3 began an action for damages for wrongful
dismissal in October 20X3.

She is claiming $100,000 in damages. Aluki is resisting the claim and the company’s
lawyers have advised that the employee has a 30% chance of success in her claim.

The financial statements currently include a provision for the $100,000 claim.

Required:

Explain to the directors how this matter should be treated in the financial statements
for the year ended 30 September 20X3, stating the relevant accounting standards.

___________________________________________________________________________

272
__________________________

KEY NOTES
_________________

___________________________________________________________________________

273
1 Provisions

A provision is a liability of uncertain timing or amount.

IAS 37 requires a provision be recognised when all of the following apply:

1. an entity has a present obligation (legal or constructive) as a result of a past


event
2. it is probable that an outflow of resources embodying economic benefits will
be required to settle the obligation
3. a reliable estimate can be made of the amount of the obligation

A provision is accounted for as follows: -

Debit Expense Account (I/S)


Credit Provision Account (SOFP)

2 Contingent Liabilities

Contingent liabilities are:


(a) possible obligations that arise from past events and whose existence will be
confirmed only by the occurrence or nonoccurrence of one or more uncertain
future events not wholly within the control of the entity
(b) present obligations that arise from past events but are not recognised
because:
iii. they are not probable that an outflow of resources embodying
economic benefits will be required to settle the obligation; or
iv. the amount of the obligation cannot be measured with sufficient
reliability

Contingent liabilities should not be recognized in financial statements but they should
be disclosed, unless the possibility of any outflow is remote.

3 Contingent assets

Contingent assets are possible assets that arise from past events and whose
existence will be confirmed only by the occurrence or nonoccurrence of one or more
uncertain future events not wholly within the control of the entity.

Contingent assets must only be disclosed in the notes if an inflow of economic


benefits is probable, otherwise they should be ignored. If the probability of an inflow
___________________________________________________________________________

274
of economic benefits is virtually certain then the asset is not a contingent asset and
should be recognised in the financial statements.

___________________________________________________________________________

275
___________________

QUESTION BANK
___________________

___________________________________________________________________________

276
1. Which of the following statements about the requirements of IAS 37
Provisions, contingent liabilities and contingent assets are correct?
1. A contingent asset should be disclosed by note if an inflow of
economic benefits is probable.
2. No disclosure of a contingent liability is required if the possibility of
a transfer of economic benefits arising is remote.
3. Contingent assets must not be recognised in financial statements
unless an inflow of economic benefits is virtually certain to arise.

A. All three statements are correct


B. 1 and 2 only
C. 1 and 3 only
D. 3 only

2. The following items have to be considered in finalising the financial


statements of Q, a limited liability company:
1. The company gives warranties on its products. The company’s
statistics show that about 5% of sales give rise to a warranty claim.
2. The company has guaranteed the overdraft of another company. The
likelihood of a liability arising under the guarantee is assessed as
possible.

What is the correct action to be taken in the financial statements for these
items?

___________________________________________________________________________

277
3. Which of the following statements about contingent assets and contingent
liabilities are correct?
1. A contingent asset should be disclosed by note if an inflow of economic
benefits is probable.
2. A contingent liability should be disclosed by note if it is probable that a
transfer of economic benefits to settle it will be required, with no
provision being made.
3. No disclosure is required for a contingent liability if it is not probable
that a transfer of economic benefits to settle it will be required.
4. No disclosure is required for either a contingent liability or a contingent
asset if the likelihood of a payment or receipt is remote.

A. 1 and 4 only
B. 3 only
C. 2, 3 and 4
D. 1, 2 and 4

4. A company is being sued for $10,000 by a customer. The company’s lawyers


reckon that it is likely that the claim will be upheld. Legal fees are currently
$5,000.

How should the company account for this?

A. Provision
B. Contingent liability
C. Contingent asset

___________________________________________________________________________

278
______________________

ANSWER BANK
______________

___________________________________________________________________________

279
1. A

2. A – In case 2, the likelihood of the liability arising is POSSIBLE.

3. A – If a liability is probable, a provision should be done.

4. A

___________________________________________________________________________

280
Chapter 19
Preparing the Financial Statements of Limited
Liability Companies

There are some fundamental differences between the accounts of sole traders and
partnerships and limited liability companies. The following are perhaps the most
significant.

a) Legislation governing the activities of limited liability companies tends to be


very extensive. It may specify that the annual accounts of a company must be
filed with a government bureau and so available for public inspection; and
they often contain detailed requirements on the minimum information which
must be disclosed in a company's accounts. Also, the financial statements of
companies must be audited annually.

b) The owners of a company (its shareholders) may be very numerous. Their


capital is shown differently from that of a sole trader; and similarly the
'appropriation account' of a company is different.

c) The liability for the debts of the business in a sole trader or partnership is
unlimited, which means that if the business runs up debts that it is unable to
pay, the proprietors will become personally liable for the unpaid debts, and
would be required, if necessary, to sell their private possessions in order to
repay them. On the other hand, limited liability companies offer limited liability
to their owners. Limited liability means that the maximum amount that an
owner stands to lose in the event that the company becomes insolvent and
cannot pay off its debts, is his share of the capital in the business.

19.1 ACCA SYLLABUS GUIDE OUTCOME 1:


Understand the capital structure of a limited liability company including:
a. Ordinary shares
b. Preference shares
c. Loan notes

The owners' capital in a limited liability company consists of share capital. When a
company is originally set up, it issues shares. These are paid for by investors, who
then become shareholders of the company. Shares are issued in units of 10 cents,
25 cents, 50 cents, $1 or even $2. The 'face value' of the shares is called their par
value or nominal value, e.g. 100,000 shares of $1 each par value were issued at $1
each.

___________________________________________________________________________

281
However, shares may be issued at a price higher than their par value, e.g. the
company may issue 20,000 shares of $1 each at $1.25 per share. This excess over
the par value is called share premium.
1. Authorised capital is the maximum amount of share capital that a company
is empowered to issue. The amount of authorised share capital can change
by agreement.
For example, a company's authorised share capital might be 10,000,000
ordinary shares of $1 each.

2. Issued capital is the amount at nominal value of share capital that has been
issued to shareholders. This amount of issued share capital cannot exceed
the amount of authorised capital. Therefore, the company with authorised
share capital of 10,000,000 ordinary shares of $1 might have issued
6,000,000 shares. It may issue 4,000,000 more shares at some time in the
future.

3. Called-up capital. When shares are issued, a company may not always be
paid the full amount for the shares at once. It might call up only a part of the
issue price, and wait until a later time before it calls up the remainder.
For example, if a company issues 6,000,000 ordinary shares of $1, it might
call up only, say, 80 cents per share. Although the issued share capital would
be $6,000,000, the called-up share capital would only be $4,800,000.

4. Paid-up capital. When capital is called up, some shareholders might delay
their payment (or even default on payment). Paid-up capital is the amount of
called-up capital that has been paid.
For example, if a company issues 6,000,000 ordinary shares of $1 each, calls
up 80 cents per share, but only receives payments of $3,600,000, the capital
not yet paid up would be $1,200,000 (4,800,000 – 3,600,000)

19.1.1 Preference shares

Preference shares carry the right to a final dividend which is expressed as a


percentage of their par value: e.g. a 5% $1 preference share carries a right to an
annual dividend of 5c.

Preference dividends have priority over ordinary dividends. The managers of a


company are obliged to pay preference dividend first. If the preference shares are
cumulative, it means that before a company can pay any ordinary dividend it must
not only pay the current year's preference dividend, but must also make good any
arrears of preference dividends which were not paid in previous years.

Also, preference shareholders have priority over ordinary shareholders to a return of


their capital if the company goes into liquidation. However, preference shares do not
carry a right to vote.

Preference shares may be either redeemable or irredeemable


___________________________________________________________________________

282
Redeemable preference shares mean that the company will repay the nominal
value of those shares at a later date.
For example, 'redeemable 6% $1 preference shares 20X8' means that the company
will pay these shareholders $1 for every share they hold on a certain date in 20X8.
Redeemable preference shares are treated like loans and are included as non-
current liabilities in the statement of financial position. However, if the redemption is
due within 12 months, the preference shares will be classified as current liabilities.
Dividends paid (6c per share in our example) on redeemable preference shares are
included as a finance costs (added to interest paid) in the statement of profit or loss.

Irredeemable preference shares form part of equity and their dividends are treated
as appropriations of profit.

19.1.2 Ordinary shares

Ordinary shares carry no right to a fixed dividend but ordinary shareholders are
entitled to all profits. In fact, the amount of ordinary dividends fluctuates from year to
year.

Ordinary shares normally carry voting rights. Therefore, ordinary shareholders are
the effective owners of a company. They own the 'equity' of the business including
any reserves of the business. Ordinary shareholders are sometimes referred to as
equity shareholders.

19.1.3 Loan Notes

Limited liability companies may issue loan stock or bonds to raise finance. These are
non-current liabilities but are different from share capital: -
a) Shareholders are the owners of a company, while providers of loan capital are
creditors of the company.
b) Shareholders receive dividends whereas loan holders are entitled to a fixed
rate of interest every year. This interest is an expense in the statement of
profit or loss and is calculated on the par value, regardless of its market value.
c) Loan holders have to be paid interest when due. Otherwise, they can take
legal action against the company if their interest is not paid. Therefore, loan
stock is generally less risky than shares.

19.2 ACCA SYLLABUS GUIDE OUTCOME 2:


Record movements in the share capital and share premium accounts

Lecture Example 1

A company made an issue for cash of 1,000,000 50c shares at par. What is the
double-entry of this transaction?
___________________________________________________________________________

283
Lecture Example 2

A company made an issue for cash of 1,000,000 50c shares at a premium of 30c per
share.

Which of the following journal entries correctly records the issue?

Debit Credit
$ $
A. Share capital 500,000
Share premium 300,000
Bank 800,000
B. Bank 800,000
Share capital 500,000
Share premium 300,000
C. Bank 1,300,000
Share capital 1,000,000
Share premium 300,000
D. Share capital 1,000,000
Share premium 300,000
Bank 1,300,000

19.3 ACCA SYLLABUS GUIDE OUTCOME 3:


Identify and record the other reserves which may appear in the company
statement of financial position

When describing ordinary shareholders, we have said that these own the ‘equity’ of
the business including any reserves. Shareholders' equity consists of: -

a) Share capital (at nominal value)


b) Share premium – the difference between the issue price of the share
and its par value
c) Revaluation surplus – a non-distributable reserve representing
unrealised profits on the revalued assets (refer to Chapter 10)
d) Other reserves – very often, these are revenue reserves which may
either have a specific purpose (e.g. asset replacement reserve) or not
(e.g. general reserve)
e) Retained earnings – these are profits earned by the company and
which have been retained by the business, i.e. they have not been paid
out as dividends, taxes or transferred to another reserve. This reserve
usually increases from year to year as companies do not normally
distribute all their profits.

___________________________________________________________________________

284
19.4 ACCA SYLLABUS GUIDE OUTCOME 4:
Define a bonus (capitalisation) issue and its advantages and disadvantages

A company may wish to increase its share capital without needing to raise additional
finance. A bonus issue raises no funds.

A company can make a bonus issue to re-classify some of its reserves as share
capital. Any reserve may be re-classified in this way, including a share premium
account or other reserve. Therefore, these reserves will be debited and share capital
credited. Such a re-classification increases the capital base of the company and
gives greater protection to the company’s creditors.

Advantages: -
a) Increases share capital without reducing present shareholders' holdings
b) Capitalises reserves, therefore less is available for distribution as dividends

Disadvantages: -
a) Does not increases cash
b) If profits fall, the payment of dividends could be jeopardised

19.5 ACCA SYLLABUS GUIDE OUTCOME 5:


Record and show the effects of a bonus (capitalisation) issue in the statement
of financial position

Accounting Treatment

Dr Share Premium
Cr Share Capital

A bonus issue is always done at nominal value.

___________________________________________________________________________

285
Lecture Example 3

At 31 December 20X1 the capital structure of a company was as follows:


$
Ordinary share capital
100,000 shares of 50c each 50,000
Share premium account 180,000

During 20X2 the company made a bonus issue of 1 share for every 2 held, using the
share premium account for the purpose, and later issued for cash another 60,000
shares at 80c per share.

What is the company’s capital structure at 31 December 20X2?

Ordinary share capital Share premium account


$ $
A. 130,000 173,000
B. 105,000 173,000
C. 130,000 137,000
D. 105,000 137,000

19.6 ACCA SYLLABUS GUIDE OUTCOME 6:


Define a rights issue and its advantages and disadvantages

A rights issue is an issue of shares for cash. These shares are usually issued at a
discount to the current market price. The 'rights' are offered to existing shareholders,
who can sell them if they wish.

Advantages: -
a) Raises cash
b) Reserves are available for future dividend distribution

Disadvantages: -
a) If a shareholder sells his rights, he will be losing (diluting) his control in the
company

___________________________________________________________________________

286
19.7 ACCA SYLLABUS GUIDE OUTCOME 7:
Record and show the effects of a right issue in the statement of financial
position

Accounting Treatment

Dr Cash
Cr Share Capital
Cr Share Premium

Lecture Example 4

At 30 June 20X2 a company’s capital structure was as follows:


$
Ordinary share capital
500,000 shares of 25c each 125,000
Share premium account 100,000

In the year ended 30 June 20X3 the company made a rights issue of 1 share for
every 2 held at $1 per share and this was taken up in full. Later in the year the
company made a bonus issue of 1 share for every 5 held, using the share premium
account for the purpose.
What was the company’s capital structure at 30 June 20X3?

Ordinary share capital Share premium account


$ $
A. 450,000 25,000
B. 225,000 250,000
C. 225,000 325,000
D. 212,500 262,500

19.8 ACCA SYLLABUS GUIDE OUTCOME 8:


Record dividends in ledger accounts and the financial statements

Dividends are an appropriation of retained earnings to shareholders. They are not an


expense in the statement of profit or loss.

Accounting Treatment

Dr Retained Earnings (SOFP)


Cr Dividends Payable (SOFP)

___________________________________________________________________________

287
Dividends can be paid during the year (interim dividends) or at the end of the year
(final dividends). The final dividend will only be accounted for if it has been declared
before year end. Otherwise, it will be disclosed as a note to the financial statements.

Lecture Example 5

A company's issued share capital consists of $100,000 in 5% $1 preference shares


and $50,000 in 50c ordinary shares. Its net profit for the year was $176,000 and the
directors paid an ordinary dividend for the year of 10c per share.

What is the company's retained profit for the year?

Lecture Example 6

The issued share capital of Alpha, a limited liability company, is as follows:


$
Ordinary shares of 10c each 1,000,000
8% Preference shares of 50c each 500,000

In the year ended 31 October 20X2, the company has paid the preference dividend
for the year and an interim dividend of 2c per share on the ordinary shares. A final
ordinary dividend of 3c per share is proposed.

What is the total amount of dividends relating to the year ended 31 October 20X2?
A. $580,000
B. $90,000
C. $130,000
D. $540,000

19.9 ACCA SYLLABUS GUIDE OUTCOME 9:


Calculate and record finance costs in ledger accounts and the financial
statements

The interest expense incurred on loan stock and bonds will be shown as an expense
called ‘finance costs' in the statement of profit or loss. We have also seen that
dividends paid on redeemable preference shares are also included as finance costs.

Accounting Treatment

Dr Finance Costs (I/S)


Cr Bank

___________________________________________________________________________

288
Lecture Example 7

At 30 June 20X2 a company had $1m 8% loan notes in issue, interest being paid
half-yearly on 30 June and 31 December.

On 30 September 20X2 the company redeemed $250,000 of these loan notes at par,
paying interest due to that date.

On 1 April 20X3 the company issued $500,000 7% loan notes, interest payable half-
yearly on 31 March and 30 September.

What figure should appear in the company’s statement of profit or loss for interest
payable in the year ended 30 June 20X3?

A. $88,750
B. $82,500
C. $65,000
D. $73,750

Lecture Example 8

A company's share capital consists of 20,000 25c ordinary shares all, of which were
issued at a premium of 20%. The market value of the shares is currently 70c each.

What would the balance on ordinary share capital be?

A. $14,000
B. $6,000
C. $5,000

19.10 ACCA SYLLABUS GUIDE OUTCOME 10:


Record income tax in the statement of profit or loss of a company including
the under and overprovision of tax in the prior year

Lecture Example 9

A company has a tax liability brought forward of $16,000. The liability is finally
agreed at $17,500 and this is paid during the year. The company estimates that the
tax liability based on the current year’s profits will be $25,000. Calculate the tax
expense and the tax payable for the year

___________________________________________________________________________

289
Further Question: 1

The issued share capital of RCA Ltd is as follows:

$
Ordinary shares of 20 cents each 1,000,000
8% preference shares of 50 cents each 500,000
In the year ended 30 September 2010, the company paid the preference dividend for
the year and an interim dividend of 3 cents per share on the ordinary shares. A final
ordinary dividend of 5 cents per share was declared on 29 September 2010.

Calculate the total amount of dividends accounted for in the year ended 30
September 2010.
$_________

Further Question: 2

Pulis plc has the following capital structure:

$
400,000 shares of $0.25 each 100,000
Share premium account 150,000

It makes 1 for 5 rights issue at $1.45, which is fully subscribed.

Calculate the balances on the share capital and share premium accounts after the
rights issue.

Further Question: 3

At 31 December 20X8 the capital structure of Pilot Ltd was as follows:

$
200,000 ordinary shares of $0.50 each 100,000
Share premium account 360,000

During 20X9, the company made a 1 for 2 bonus issue, using the share premium
account for the purpose, and later issued for cash another 120,000 shares at $1.60
per share.

Calculate the balances on the company’s share capital and premium accounts as at
31 December 20X9.

___________________________________________________________________________

290
Further Question: 4

The following figures relating to tax of Plot plc are provided:

Tax payable at 1 April 2010 28,600


Tax paid during the year ended 31 March 2011 25,400

The estimated tax liability for the year ended 31 March 2011 is $31,200
Calculate:
a) The tax expense in the statement of profit or loss.
b) The tax due at 31 March 2011 which will be included in the SFP.

___________________________________________________________________________

291
__________________________

KEY NOTES
_________________

___________________________________________________________________________

292
1. Types of share capital

Authorised share capital: the maximum amount of share capital that a company is
empowered to issue.

Issued share capital: the amount of share capital that has been issued to
shareholders.

Called-up share capital: the amount the company has asked shareholders to pay,
for the time being, on shares issued to them.

Paid-up share capital: the amounts actually paid by shareholders on shares issued
to them.

2. Capital Structure

Preference shares carry the right to a final dividend which is expressed as a


percentage of their par value. Preference dividends have priority over ordinary
dividends. Also, preference shareholders have priority over ordinary shareholders to
a return of their capital if the company goes into liquidation. However, preference
shares do not carry a right to vote.

Preference shares may be either redeemable or irredeemable. Redeemable


preference shares mean that the company will repay the nominal value of those
shares at a later date. Irredeemable preference shares form part of equity and their
dividends are treated as appropriations of profit.

Ordinary shares carry no right to a fixed dividend but ordinary shareholders are
entitled to all profits. Ordinary shares normally carry voting rights.

Limited liability companies may issue loan stock or bonds to raise finance. These
are non-current liabilities and the interest is an expense in the statement of profit or
loss

3. Shareholders’ Equity

Shareholders' equity consists of: -

a) Share capital (at nominal value)


b) Share premium – the difference between the issue price of the share and its
par value
c) Revaluation surplus – a non-distributable reserve representing unrealised
profits on the revalued assets
d) Other reserves – very often, these are revenue reserves which may either
have a specific purpose (e.g. asset replacement reserve) or not (e.g. general
reserve)

___________________________________________________________________________

293
e) Retained earnings – these are profits earned by the company and which have
been retained by the business

4. Bonus Issue

A company may wish to increase its share capital without needing to raise additional
finance. A bonus issue raises no funds. A bonus issue increases the capital base of
the company and gives greater protection to the company’s creditors

Accounting Treatment

Dr Share Premium
Cr Share Capital

A bonus issue is always done at nominal value.

5. Rights Issue

A rights issue is an issue of shares for cash. These shares are usually issued at a
discount to the current market price. The 'rights' are offered to existing shareholders,
who can sell them if they wish.

Accounting Treatment

Dr Cash
Cr Share Capital
Cr Share Premium

6. Dividends

Dividends are an appropriation of retained earnings to shareholders. They are not an


expense in the statement of profit or loss.

Accounting Treatment

Dr Retained Earnings (SOFP)


Cr Dividends Payable (SOFP)

7. Finance Costs

The interest expense incurred on loan stock and bonds will be shown as an expense
called ‘finance costs' in the statement of profit or loss.

Accounting Treatment
Dr Finance Costs (I/S)
Cr Bank
___________________________________________________________________________

294
__________________________

QUESTION BANK
_________________

___________________________________________________________________________

295
1. The equity capital of a limited liability company comprises

A. Ordinary share capital, preference share capital and retained earnings


B. Ordinary share capital
C. Ordinary share capital and retained earnings
D. Preference share capital

2. When a company makes a rights issue of equity shares which of the following
effects will the issue have?

1. Working capital is increased


2. Share premium account is reduced
3. Investments are increased

A. 1 only
B. 1 and 2
C. 3 only

3. A limited liability company issued 50,000 ordinary shares of 25c each at a


premium of 50c per share. The cash received was correctly recorded but the full
amount was credited to the ordinary share capital account.

Which of the following journal entries is needed to correct this error?

Debit Credit
$ $
A. Share premium account 25,000
Share capital account 25,000
B. Share capital account 25,000
Share premium account 25,000
C. Share capital account 37,500
Share premium account 37,500
D. Share capital account 25,000
Cash 25,000

___________________________________________________________________________

296
4. Which of the following journal entries could correctly record a bonus
(capitalisation) issue of shares?

Debit Credit
$ $
A. Cash 100,000
Ordinary share capital 100,000
B. Ordinary share capital 100,000
Share premium 100,000
C. Share premium 100,000
Ordinary share capital 100,000
D. Investments 100,000
Cash 100,000

5. Which of the following statements are correct?

1. A company might make a rights issue if it wished to raise more equity


capital.
2. A rights issue might increase the share premium account whereas a bonus
issue is likely to reduce it.
3. A rights issue will always increase the number of shareholders in a
company whereas a bonus issue will not.

A. 1 and 2
B. 1 and 3
C. 2 and 3
D. 2 and 4

___________________________________________________________________________

297
__________________________

ANSWER BANK
_________________

___________________________________________________________________________

298
1. C

2. A

3. B – The share premium is 50,000. Shares x 50c = $25,000.

4. C

5. A – A rights issue is offered to existing shareholders.

___________________________________________________________________________

299
Chapter 20
Financial Statements for Companies

IAS 1 (revised) “Presentation of Financial Statements” prescribes the basis for


presentation of general purpose financial statements to ensure comparability both
with the entity’s financial statements of previous periods and with the financial
statements of other entities. It sets out overall requirements for the presentation of
financial statements, guidelines for their structure and minimum requirements for
their content.

A complete set of financial statements comprises:


(i) a statement of financial position as at the end of the period;
(ii) a statement of comprehensive income for the period;
(iii) a statement of changes in equity for the period;
(iv) a statement of cash flows for the period;
(v) notes, comprising a summary of significant accounting policies and
other explanatory information; and
(vi) a statement of financial position as at the beginning of the earliest
comparative period when an entity applies an accounting policy
retrospectively or makes a retrospective restatement of items in its
financial statements, or when it reclassifies items in its financial
statements.

An entity shall present a complete set of financial statements (including comparative


information) at least annually.

20.1 ACCA SYLLABUS GUIDE OUTCOMES 1 and 2:


Prepare extracts of a statement of profit or loss and statement of profit or loss
and other comprehensive income from given information
Prepare extracts of a statement of financial position from given information

One of the statements introduced by IAS 1 (revised) is the statement of


comprehensive income. This statement presents all items of income and expense
recognized in profit or loss together with all other items recognized in income and
expense. Entities may present all items together in a single statement or present two
linked statements – one displaying the items of income and expense recognised in
the statement of profit or loss and the other statement beginning with profit or loss
and displaying all the items included in ‘other comprehensive income’.

Therefore, whereas the statement of profit or loss includes all realised gains and
losses (e.g. net profit for the year), the statement of comprehensive income would
include both the realised and unrealised gains and losses (e.g. revaluation surplus).
___________________________________________________________________________

300
20.1.1 Proforma 1: One single statement

Statement of comprehensive income for the year ended 31 March 20X8


20X8 20X7
$’000 $’000
Revenue X X
Cost of sales (X) (X)
Gross profit X X
Other income
X X
Distribution costs (X) (X)
Administrative expenses (X) (X)
Finance costs (X) (X)
Investment income X X
Profit before tax X X
Income tax expense (X) (X)
Profit for the year X X
Other comprehensive income:
Gains on property revaluation X X
Total comprehensive income for the year X X

20.1.2 Proforma 2: Two separate statements

Statement of profit or loss for the year ended 31 March 20X8


20X8 20X7
$’000 $’000
Revenue X X
Cost of sales (X) (X)
Gross profit X X
Other income X X
Distribution costs (X) (X)
Administrative expenses (X) (X)
Finance costs (X) (X)
Investment income X X
Profit before tax X X
Income tax expense (X) (X)
Profit for the year X X

Statement of comprehensive income for the year ended 31 March 20X8


20X8 20X7
$’000 $’000
Profit for the year X X

___________________________________________________________________________

301
Other comprehensive income:
Gains on property revaluation X X
Total comprehensive income for the year X X
20.1.3 Statement of financial position as at 31 March 20X8
$'000
ASSETS
Non-current assets
Property, plant and equipment X
Other intangible assets X
X
Current assets
Inventories X
Trade receivables X
Other current assets X
Cash and cash equivalents X
X
Total assets X
EQUITY AND LIABILITIES
Equity
Share capital X
Share premium account X
Revaluation surplus X
Retained earnings X
X
Non-current liabilities
Long term borrowings X
Long term provisions X
Current liabilities
Trade payables X
Short term borrowings X
Current tax payable X
Short term provisions X
Total equity and liabilities X

20.2 ACCA SYLLABUS GUIDE OUTCOME 3:


Identify the components of the statement of changes in equity

The revised statement of changes in equity separates owner and non-owner


changes in equity. It includes only details of transactions with owners, with all non-
owner changes in equity presented as a single line – total comprehensive income.

___________________________________________________________________________

302
Statement of changes in equity – Proforma

Share Share Revaluation Retained Total


Capital Premium Reserve Earnings
‘000 ‘000 ‘000 ‘000 ‘000
Balance at 31 March 20X7 X X X X X
Changes in accounting X X
policy
Restated balance X X X X X
Issue of share capital X X X
Dividends (X) (X)
Total comprehensive X X X
income
Balance at 31 March 20X8 X X X X X

Lecture Example 1

At 1 July 20X3 the statement of financial position of Sugar, a limited liability


company, contained the following items:
$m
Issued share capital – ordinary shares of 50c 100
Share premium account 140
Revaluation surplus 1 60
Retained earnings 120
––––
420
––––
During the year ended 30 June 20X4 the following events took place:
(i) On 1 July 20X3 the company issued 200m ordinary shares, ranking equally
with those already in issue, at $1.40 per share.
(ii) Some land held by the company as a non-current asset was sold for $100m.
The land had originally cost $25m and was revalued to $85m in 20X2, giving
rise to the revaluation surplus of $60m shown above.
(iii) The company’s draft pre-tax profit for the year ended 30 June 20X4 was
$40m.
(iv) Dividends totalling 2c per share were paid in the year on the enlarged capital.

Required:
Prepare the company’s statement of changes in equity for the year ended 30
June 20X4.

___________________________________________________________________________

303
Lecture Example 2

Which of the following statements about company financial statements is/are correct,
according to International Financial Reporting standards?

1. A material profit or loss on the sale of part of the entity must appear in the
statement of comprehensive income as an extraordinary item.
2. Dividends paid and proposed should be included in the statement of
comprehensive income.
3. The statement of comprehensive income must show separately any
material profit or loss from operations discontinuing during the year.
4. The statement of changes in equity must not include unrealised gains or
losses.
A. 1, 2 and 3
B. 2 and 4
C. 3 only

20.3 ACCA SYLLABUS GUIDE OUTCOME 3:


Explain the purpose of disclosure notes

Notes to the accounts are prepared for the following three purposes:
(i) present information about the basis of preparation of the financial
statements and the specific accounting policies used;
(ii) disclose the information required by IFRSs that is not presented elsewhere
in the financial statements; and
(iii) provide information that is not presented elsewhere in the financial
statements, but is relevant to an understanding of any of them.

20.4 ACCA SYLLABUS GUIDE OUTCOME 4:


Draft the following disclosure notes

We have already prepared the disclosure notes when dealing with the relevant
standard. However, these are the disclosure notes you would need to know for your
exams.

20.4.1 Property, Plant and Equipment

For each class of property, plant and equipment, disclose:

 basis for measuring carrying amount


 depreciation method(s) used
 useful lives or depreciation rates
___________________________________________________________________________

304
 gross carrying amount and accumulated depreciation and impairment losses
 reconciliation of the carrying amount at the beginning and the end of the
period, showing:

o additions
o disposals
o acquisitions through business combinations
o revaluation increases or decreases
o impairment losses
o reversals of impairment losses
o depreciation
o net foreign exchange differences on translation
o other movements

If property, plant, and equipment is stated at revalued amounts, certain additional


disclosures are required:

 the effective date of the revaluation


 whether an independent valuer was involved
 the methods and significant assumptions used in estimating fair values
 for each revalued class of property, the carrying amount that would have been
recognised had the assets been carried under the cost model
 the revaluation surplus, including changes during the period and any
restrictions on the distribution of the balance to shareholders

Land Machinery Office Total


and Equipment
Buildings
$ $ $ $
Cost or valuation
At 1 January 2010 50,000 10,000 8,000 68,000
Revaluation surplus 12,000 2,000 2,000 16,000
Additions in year 4,000 4,000 - 8,000
Disposals in year (1,000) (1,000) - (2,000)
At 31 December 2010 65,000 15,000 10,000 90,000

Depreciation
At 1 January 2010 16,000 6,000 4,000 26,000
Charge for year 4,000 3,000 2,000 9,000
Eliminated on disposals (500) (500) - (1,000)
At 31 December 2010 19,500 8,500 6,000 34,000

Carrying Amount
At 31 December 2010 45,500 6,500 4,000 56,000
At 1 January 2010 34,000 4,000 4,000 42,000

___________________________________________________________________________

305
20.4.2 Intangible non-current assets (IAS 38)

For each class of intangible asset, disclose

 useful life or amortisation rate


 amortisation method
 gross carrying amount
 accumulated amortisation and impairment losses
 line items in the statement of profit or loss in which amortisation is included
 reconciliation of the carrying amount at the beginning and the end of the
period showing:
o additions (business combinations separately)
o assets held for sale
o retirements and other disposals
o revaluations
o impairments
o reversals of impairments
o amortisation
o foreign exchange differences
o other changes
 basis for determining that an intangible has an indefinite life
 description and carrying amount of individually material intangible assets
 certain special disclosures about intangible assets acquired by way of
government grants
 information about intangible assets whose title is restricted
 contractual commitments to acquire intangible assets

Additional disclosures are required about:

 intangible assets carried at revalued amounts


 the amount of research and development expenditure recognised as an
expense in the current period

Development expenditure
$
Net book value at 1 April 20X0 X
Additions X
Amortisation charge (X)
Disposals (X)
Net book value at 31 March 20X1 X

At 31 March 20X0
Cost X
Accumulated amortisation (X)
Net book value X

___________________________________________________________________________

306
At 31 March 20X1
Cost X
Accumulated amortisation (X)
Net book value X

20.4.3 Provisions, Contingent Liabilities and Contingent Assets (IAS 37)

Provisions: -

At 1 April 20x0 X
Increase in period X
Released in period (X)
At 31 March 20x1 X

Contingent liabilities should not be recognized in financial statements but they should
be disclosed, unless the possibility of any outflow is remote. The required
disclosures are:
 A brief description of the nature of the contingent liability;
 An estimate of its financial effect;
 An indication of the uncertainties that exist relating to the amount or timing
of any outflow; and

Where an inflow of economic benefits is probable, an entity should disclose: -


1. a brief description of its nature; and where practicable
2. an estimate of the financial effect

20.4.4 Events after the reporting period (IAS 10)

IAS 10 requires these three disclosures: -

1. the date when the financial statements were authorised for issue and who
gave that authorisation.
2. if information is received after the end of the reporting period about conditions
that existed at the end of the reporting period, disclosures that relate to those
conditions should be updated in the light of the new information.
3. where non-adjusting events after the reporting period are of such significance
that non-disclosure would affect the ability of the users of financial statements
to make proper evaluations and decisions, disclosure should be made for
each such significant category of non-adjusting event regarding the nature of
the event and an estimate of its financial effect or a statement that such an
estimate cannot be made.

___________________________________________________________________________

307
20.4.5 Inventories (IAS 2)

The financial statements should disclose the following: -

 accounting policy for inventories


 carrying amount, generally classified as merchandise, supplies, materials,
work in progress, and finished goods. The classifications depend on what is
appropriate for the entity
 carrying amount of any inventories carried at fair value less costs to sell
 amount of any write-down of inventories recognised as an expense in the
period

20.5 IAS 18 - Revenue

IAS 18, Revenue, prescribes the requirements for the recognition of revenue arising
from an entity’s ordinary activities.

20.5.1 Measurement of Revenue

Revenue is to be measured at the fair value of the consideration received or


receivable. By fair value, we mean “the amount for which an asset can be
exchanged, or a liability settled, between knowledgeable parties in an arm’s length
transaction.”

Generally, revenue is recognized when the entity has transferred to the buyer the
significant risks and rewards of ownership and when the revenue can be measured
reliably.

20.5.2 Scope

IAS 18 covers the revenue from: -


1 Sale of goods
2 Rendering of services
3 Interest, royalties and dividends

20.5.2.1 Revenue from the sale of goods

Revenue from the sale of goods should be recognized when all of the following
criteria are satisfied: -

1 The significant risks and rewards of ownership of the goods have been
transferred to the buyer

___________________________________________________________________________

308
2 The seller retains neither continuing managerial involvement to the degree
usually associated with ownership nor effective control over the goods sold
3 The amount of the revenue can be reliably measured
4 It is probable that economic benefits associated with the transaction will flow
to the seller
5 The costs incurred or to be incurred in respect of the transaction can be
measured reliably

20.5.2.2 Revenue from the rendering of services

For revenue arising from the rendering of services, revenue should be recognised by
reference to the stage of completion of the transaction at the end of the reporting
period. The following criteria must be met:

1 the amount of revenue can be measured reliably;


2 it is probable that the economic benefits will flow to the seller;
3 the stage of completion at the balance sheet date can be measured reliably; and
4 the costs incurred, or to be incurred, in respect of the transaction can be
measured reliably.

20.5.2.3 Interest, royalties and dividends

Interest, royalties and dividends are included as income because they arise from the
use of an entity’s assets by other parties. They are recognised as revenue when the
economic benefits are expected to flow to the enterprise and the amount of revenue
can be measured reliably.

Revenue does not include sales taxes, value added taxes or any other tax which is
collected for third parties.

Lecture Example 3

Xtra Ltd, a new company manufacturing and selling consumable products, has come
out with an offer to refund the cost of purchase within one month of sale if the
customer is not satisfied with the product.

When should Xtra Ltd recognize the revenue?

A. When goods are sold to the customers


B. After one month of sale
C. Only if goods are not returned by the customers after the period of one
month
D. At the time of sale along with an offset to revenue of the liability of the
same amount for the possibility of the return

___________________________________________________________________________

309
Lecture Example 4

Revenue from an artistic performance is recognized once: -

A. The audience register for the event online


B. The tickets for the concert are sold
C. Cash has been received from the ticket sales
D. The event takes place

___________________________________________________________________________

310
__________________________

KEY NOTES
_________________

___________________________________________________________________________

311
1. A complete set of financial statements comprises:
(i) a statement of financial position as at the end of the period;
(ii) a statement of comprehensive income for the period;
(iii) a statement of changes in equity for the period;
(iv) a statement of cash flows for the period;
(v) notes, comprising a summary of significant accounting policies and
other explanatory information; and
(vi) a statement of financial position as at the beginning of the earliest
comparative period when an entity applies an accounting policy
retrospectively or makes a retrospective restatement of items in its
financial statements, or when it reclassifies items in its financial
statements.

2. The statement of comprehensive income includes both the realised gains and
losses from the statement of profit or loss and the unrealised gains and losses
from the statement of financial position.

3. The statement of changes in equity shows the movements in share capital


and reserves (equity) which are included in the statement of financial position.

4. Notes to the accounts are prepared to:


a. present information about the basis of preparation of the financial
statements and the specific accounting policies used;
b. disclose the information required by IFRSs that is not presented
elsewhere in the financial statements; and
c. provide information that is not presented elsewhere in the financial
statements, but is relevant to an understanding of any of them.

5. IAS 18 covers the revenue from: -


a. Sale of goods
b. Rendering of services
c. Interest, royalties and dividends

Generally, revenue is recognized when the entity has transferred to the buyer
the significant risks and rewards of ownership and when the revenue can be
measured reliably.

___________________________________________________________________________

312
__________________________

QUESTION BANK
_________________

___________________________________________________________________________

313
1. Which of the following items are required to be disclosed in a limited liability
company’s financial statements according to IAS 1 Presentation of Financial
Statements?

1. Authorised share capital


2. Finance costs
3. Depreciation and amortization
4. Issued share capital

A. 1, 2 and 3 only
B. 2, 3 and 4 only
C. All four items

2. Which of the following could appear as separate items in the statement of


changes in equity required by IAS I Presentation of Financial Statements as
part of a company’s financial statements?

1. Gain on revaluation of land.


2. Loss on sale of investments.
3. Prior year adjustments.
4. Proceeds of an issue of ordinary shares.
5. Dividends proposed after the year end.

A. 1, 3 and 4 only
B. 1, 2 and 4 only
C. 1 and 3 only
D. All five items

3. Which of these statements about limited liability companies is/are correct?

1. A company might make a bonus (capitalisation) issue to raise funds for


expansion.
2. The profit or loss on the disposal of part of a company’s operations
must be disclosed in the statement of comprehensive income as an
extraordinary item if material.
3. Both realised and unrealised gains and losses may be included in the
statement of changes in equity required by IAS 1 Presentation of
Financial Statements.

A. 1 and 3
B. 2 and 3
C. 1 and 2
D. 3 only

___________________________________________________________________________

314
4. Which of the following statements regarding a limited liability company
statement of comprehensive income is correct?

A. Accounting standards define the expenses which are reported


under 'cost of sales'
B. 'Depreciation' appears as a separate heading
C. Interest payable is deducted from profit after taxation
D. Irrecoverable debts will be included under one of the statutory
expense headings (usually administrative expenses)

5. According to the illustrative financial structure in IAS 1 (revised) Presentation


of financial statements, dividends paid during the year should be disclosed in:

A. Statement of comprehensive income (statement of profit or loss)


B. Statement of changes in equity
C. Statement of financial position
D. None of these

6. Which of the following statements is incorrect?

1. All non-current assets must be depreciated.


2. If property is revalued, the revaluation surplus appears only in the
statement of comprehensive income.
3. If a tangible non-current asset is revalued, all tangible assets of the
same class should be revalued.
4. In a company’s published statement of financial position, tangible
assets and intangible assets must be shown separately.

A. 1 and 2
B. 1 and 3
C. 2 and 3
D. 3 and 4

___________________________________________________________________________

315
_________________________

ANSWER BANK
_________________

___________________________________________________________________________

316
1. C - All of these items are disclosed, either in the financial statements or in
the notes.

2. A - The loss on sale of investments will be recognised in the statement of


comprehensive income

3. D - A bonus issue does not raise any funds (no cash involved) and items are
no longer classified as extraordinary.

4. D - The contents of cost of sales are not defined by any IAS; net profit is
calculated after interest; depreciation will be included under the relevant
statutory heading

5. B – Dividends are not an expense. Dividends declared but still due at year
end go into SFP and SOCIE.

6. A – Land is usually not depreciated; gain on revaluation is also included in


the statement of changes in equity.

___________________________________________________________________________

317
Chapter 21
IAS 10: EVENTS AFTER THE REPORTING
PERIOD
21.1 ACCA SYLLABUS GUIDE OUTCOME 1:
Define an event after the reporting period in accordance with International
Financial Reporting Standards

According to IAS 10, “Events after the reporting period” are those events, both
favourable and unfavourable, that occur between the end of the reporting period and
the date when the financial statements are authorised for issue”.

These events can have important effects on the financial statements.

21.2 ACCA SYLLABUS GUIDE OUTCOME 2:


Classify events as adjusting or non-adjusting

Two types of events can be identified:

a. those that provide evidence of conditions that existed at the end of the
reporting period (adjusting events); and
b. those that are indicative of conditions that arose after the end of the reporting
period (non-adjusting events).

21.2.1 Examples of adjusting events given in IAS 10 are:

a. the resolution of a court case, as the result of which a provision has to be


recognised instead of the disclosure by note of a contingent liability;
b. evidence of impairment of assets;
c. bankruptcy of a major customer;
d. sale of inventories at prices suggesting the need to reduce the figure in the
Statement of Financial Position to the net value actually realized;
e. discovery of fraud or errors that show the financial statements were incorrect.

___________________________________________________________________________

318
21.2.2 Examples of non-adjusting events given in IAS 10 are:

a. decline in market value of investments;


b. announcement of a plan to discontinue part of the enterprise;
c. major purchases and sales of assets;
d. destruction of a major asset by fire etc;
e. sale of a major subsidiary;
f. major dealings in the company's ordinary shares;

Further provisions covered by IAS 10: -

a. Authorisation for issue of financial statements

An enterprise should disclose the date when the financial statements were
authorised for issue and who gave that authorisation. If the owners or others have
the power to amend the financial statements after issue, that fact should be
disclosed.

b. Going concern

If the management decides after the end of the reporting period that it is necessary
to liquidate the enterprise, the financial statements should not be prepared on a
going concern basis.

c. Dividends

If an entity declares dividends after the reporting period, the entity shall not
recognise those dividends as a liability at the end of the reporting period. That is a
non-adjusting event.

21.3 ACCA SYLLABUS GUIDE OUTCOME 3:


Distinguish between how adjusting and non-adjusting events are reported in
the financial statements.

Financial statements should be adjusted for adjusting events. This means that the
amounts in the financial statements should be changed.

Non-adjusting events do not, by definition, require an adjustment to the financial


statements, but if they are of such importance that non-disclosure would affect the
ability of users of the financial statements to make proper evaluations and decisions,
the enterprise should disclose by note:

a. the nature of the event; and


b. an estimate of its financial effect, or a statement that such an estimate cannot
be made.

___________________________________________________________________________

319
Lecture Example 1

Which of the following statements are correct, according to IAS 10 Events after
the reporting period?

1. Details of all adjusting events must be disclosed by note to the financial


statements.
2. A material loss arising from the sale, after the reporting period, of inventory
valued at cost in the statement of financial position must be reflected in the
financial statements.
3. If the market value of investments falls materially after the end of the reporting
period, the details must be disclosed by note.
4. Events after the reporting period are those that occur between the end of the
reporting period and the date when the financial statements are authorised for
issue.

A 1 and 2 only
B 1, 3 and 4
C 2 and 3 only
D 2, 3 and 4
7J–INTAO1.1INT

7J–INTAr 1.1INT
Lecture Example 2

Which of the following events after the statement of financial position date would
normally qualify as adjusting events according to IAS 10 Events after the reporting
period?

1. The bankruptcy of a credit customer with a balance outstanding at the end of


the reporting period.
2. A decline in the market value of investments.
3. The declaration of an ordinary dividend.
4. The determination of the cost of assets purchased before the end of the
reporting period.

A 1, 3, and 4
B 1 and 2 only
C 2 and 3 only
D 1 and 4 only

___________________________________________________________________________

320
Lecture Example 3

Which of the following events between the end of the reporting period and the date
the financial statements are authorised for issue must be adjusted in the financial
statements?

1. Declaration of equity dividends.


2. Decline in market value of investments.
3. The announcement of changes in tax rates.
4. The announcement of a major restructuring.

A 1 only
B 2 and 4
C 3 only
D None of them

___________________________________________________________________________

321
__________________________

KEY NOTES
_________________

___________________________________________________________________________

322
Key Definitions

 Events after the reporting period:

Those events, both favourable and unfavourable, that occur between the end of the
reporting period and the date when the financial statements are authorised for issue.

 Adjusting events:

Those events that provide evidence of conditions that existed at the end of the
reporting period.

 Non-adjusting events:

Those events that are indicative of conditions that arose after the end of the
reporting period.

Typical examples of adjusting events given in IAS 10 are:

a. the resolution of a court case;


b. evidence of impairment of assets;
c. bankruptcy of a major customer;
d. sale of inventories at prices lower that their cost;
e. discovery of fraud or errors.

Typical examples of non-adjusting events given in IAS 10 are:

a. decline in market value of investments;


b. announcement of a plan to discontinue part of the enterprise;
c. major purchases and sales of assets;
d. destruction of a major asset by fire etc;
e. sale of a major subsidiary;
f. major dealings in the company's ordinary shares

Further provisions covered by IAS 10: -

a. Authorisation for issue of financial statements

An enterprise should disclose the date when the financial statements were
authorised for issue and who gave that authorisation.

b. Going concern

If the management decides after the end of the reporting period that it is necessary
to liquidate the enterprise, the financial statements should not be prepared on a
going concern basis.

___________________________________________________________________________

323
c. Dividends

If an entity declares dividends after the reporting period, the entity shall not
recognise those dividends as a liability at the end of the reporting period. That is a
non-adjusting event.

Accounting for adjusting and non-adjusting events: -

Financial statements should be adjusted for adjusting events.

For material non-adjusting events, the enterprise should disclose by note:

a. the nature of the event; and


b. an estimate of its financial effect, or a statement that such an estimate cannot
be made.

___________________________________________________________________________

324
__________________________

QUESTION BANK
_________________

___________________________________________________________________________

325
1. Which of the following events occurring after the reporting period are
classified as adjusting, if material?

1. The sale of inventories valued at cost at the end of the reporting period for
a figure in excess of cost.
2. A valuation of land and buildings providing evidence of an impairment in
value at the year end.
3. The issue of shares and loan notes.
4. The insolvency of a customer with a balance outstanding at the year end.

A 1 and 3
B 2 and 4
C 2 and 3
D 1 and 4

2. Clogs Co has proposed dividends of $10,000 after the end of the


reporting period. What is the correct treatment according to IAS 10?

A Adjust for the dividends


B Disclose in a note to the financial statements
C Do nothing

3. Which of the following would be a non-adjusting event after the


reporting period when preparing financial statements at 30 September
20X0 according to IAS 10, Events after the reporting period?

A An insurance claim is agreed on 10 October 20X0 for compensation for


a fire in September which destroyed part of the warehouse inventory
B A decision is made on 9 October 20X0 to sell the group's major trading
activities in Eastern Europe
C Inventory valued at $30,000 is judged no longer saleable
D Notification received on 11 October 20X0 that a customer owing
$50,000 as at 30 September 20X0 has gone into liquidation

4. When does an event after the reporting period require changes in the
financial statements?

A Never
B If it provides further evidence of conditions existing at the end of the
reporting period

___________________________________________________________________________

326
5. A receivable has been written off as irrecoverable. However, the
customer suddenly pays the written off amount after the end of the
reporting period. Is this event:

A Adjusting
B Non-adjusting

___________________________________________________________________________

327
_________________

ANSWER BANK
_________________

___________________________________________________________________________

328
1. B – Inventories should always be valued at lower of cost and NRV.

2. B – Non–adjusting

3. B

A, C and D are adjusting events. B is a post year-end decision not clarifying


the position at the end of the reporting period and is therefore non-adjusting.

4. B

5. A

___________________________________________________________________________

329
Chapter 22
Accounting Policies, Changes in Accounting
Estimates and Errors

22.1 ACCA SYLLABUS OUTCOME 1:


Understand the provision of International Financial Reporting Standards
governing financial statements regarding changes in accounting policies.
.
IAS 8 prescribes criteria for selecting and changing accounting policies and also sets
out the requirements for changes in accounting estimates and corrections of errors.

Accounting policies are specific principles, bases, conventions, rules and practices
applied by an entity in preparing and presenting financial statements. Once selected,
accounting policies must be applied consistently for similar transactions, other
events and conditions. They may be changed only if the change
b) is required by a standard or an interpretation;
c) results in financial statements providing reliable and or relevant information.

22.2 ACCA SYLLABUS OUTCOME 2:


Identify the appropriate accounting treatment if a company changes a material
accounting policy.

A change in accounting policies must be applied retrospectively. That is to say, the


new policy is applied to transactions, other events and conditions as if the policy had
always been applied. The practical impact of this is that corresponding amounts (or
“comparatives”) presented in financial statements must be restated as if the new
policy had always been applied. The impact of the new policy on the retained
earnings prior to the earliest period presented should be adjusted against the
opening balance of retained earnings.

Lecture Example 1

Accurate Ltd changed its accounting policy in 20X8 with respect to the valuation of
inventories. Up to 20X7, inventories were valued using a weighted-average cost
method. In 20X8, the method changed to first-in, first-out, as it was considered to
more accurately reflect the usage and flow of inventories. The impact on inventory
valuation was determined to be:

At December 31, 20X6: - an increase of $10,000


___________________________________________________________________________

330
At December 31, 20X7: - an increase of $15,000
At December 31, 20X8: - an increase of $20,000

The statements of profit or loss prior to adjustment are: -

20X8 20X7
$ $
Revenue 250,000 200,000
Cost of sales 100,000 80,000
Gross profit 150,000 120,000
Administration costs 60,000 50,000
Selling and distribution costs 25,000 15,000
Net profit 65,000 55,000

Required: -

Present the change in accounting policy in the statement of profit or loss and the
adjusted retained earnings in accordance with the requirements of IAS 8.

Changes in accounting estimates are not examinable in the F3/FFA Paper

22.3 ACCA SYLLABUS OUTCOME 3:


Understand the provision of IFRS governing financial statements regarding
material errors which result in prior period adjustments

Prior-period errors are omissions from, and misstatements in, financial statements
for one or more prior periods arising from a failure to use, or misuse of, reliable
information that was available at the time and could reasonably be expected to have
been obtained and taken into account in the preparation and presentation of financial
statements. Misstatements or omissions are “material” if they could, either
individually or cumulatively, influence the decisions of users of financial statements.

Discovery of material errors relating to prior periods shall be corrected by restating


comparative figures in the financial statements for the year in which the error is
discovered.

Lecture Example 2

The auditor of Roman Co noticed in 20X8 that, in 20X7, the entity had omitted to
record in its books of accounts an amortisation of development expenditure of
$30,000.

___________________________________________________________________________

331
The following are extracts from the statement of profit or loss for the years ended 31
December 20X7 and 20X8, before correction of the error: -

20X8 20X7
$ $
Gross Profit 300,000 345,000
General and administrative expenses (90,000) (90,000)
Selling and distribution costs (30,000) (30,000)
Amortisation expense (30,000) XXXXX
Net income before income taxes 150,000 225,000
Income taxes (30,000) (45,000)
Net Profit 120,000 180,000

The retained earnings of Roman Co for 20X7 and 20X8 before correction of the error
are: -

Retained earnings, beginning of 20X7 $45,000


Retained earnings, end of 20X7 $225,000
Retained earnings, end of 20X8 $345,000

The income tax rate was 20% for both years

Required: -

Prepare the accounting treatment prescribed by IAS 8 for the correction of the errors.

___________________________________________________________________________

332
__________________________

KEY POINTS
_________________

___________________________________________________________________________

333
1. Accounting policies: -

Accounting policies are specific principles, bases, conventions, rules and


practices applied by an entity in preparing and presenting financial
statements.

They may be changed only if the change


a) is required by a standard or an interpretation;
b) results in financial statements providing reliable and or relevant
information.

A change in accounting policies must be applied retrospectively.


Comparatives presented in financial statements must be restated as if the
new policy had always been applied. The impact of the new policy on the
retained earnings prior to the earliest period presented should be adjusted
against the opening balance of retained earnings.

2. Prior-period errors: -

Prior-period errors are omissions from, and misstatements in, financial


statements for one or more prior periods.

Discovery of material errors relating to prior periods shall be corrected by


restating comparative figures in the financial statements for the year in which
the error is discovered. Therefore, the adjustment is also done retrospectively.

___________________________________________________________________________

334
__________________________

QUESTION BANK
_________________

___________________________________________________________________________

335
1. XYZ Co changes its method of valuation of inventories from weighted-average
method to first-in, first-out method. XYZ Co should account for this change as

A. A change in account policy and account for it prospectively


B. A change in account policy and account for it retrospectively
C. Account for it as a correction of an error and account for it
prospectively
D. Account for it as a correction of an error and account for it
retrospectively

2. Change in accounting policy does not include: -

A. Change in useful life from 10 years to 7 years


B. Change of method of valuation of inventory from FIFO to weighted-
average
C. Change of method of valuation of inventory from weighted-average to
FIFO

___________________________________________________________________________

336
__________________________

ANSWER BANK
_________________

___________________________________________________________________________

337
1. B - Changes in accounting policies and prior–period errors are always
accounted for retrospectively.

2. A - Change in useful life is a change in an accounting estimate. Take the NBV


before the change and calculate on remaining useful years.

___________________________________________________________________________

338
Chapter 23
STATEMENTS OF CASH FLOW

23.1 ACCA SYLLABUS GUIDE OUTCOMES 1 AND 2:


Differentiate between profit and cash flow
Understand the need for management to control cash flow

A business may appear profitable on its statement of profit or loss, however if its
cash outflow exceeds its cash inflow over a prolonged period then it will not survive.

Readers of a company's financial statements might also be misled by a reported


profit figure.

1. Shareholders might believe that if a company makes a profit after tax, then
this is the amount which it could afford to pay as a dividend.
2. Employees might believe that if a company makes profits, it can afford to pay
higher wages next year.
3. Survival of a business entity depends not so much on profits as on its ability to
pay its debts when they fall due.

Indeed, a business must generate sufficient cash from its operations to reward the
various stakeholders e.g., shareholders and lenders. An expanding company might
have negative operating cash flow as it builds up the level of its inventories and
receivables in line with the increased turnover. However, an increase in working
capital without an increase in turnover might indicate operational inefficiencies and
will lead to liquidity problems.

23.2 ACCA SYLLABUS GUIDE OUTCOME 3:


Recognise the benefits and drawbacks to users of the financial statements of
a statement of cash flows

One of the most useful financial statements produced by a business is the statement
of cash flow because it provides a clear and understandable picture of cash
movements over the financial year. A statement of cash flow provides useful
additional information that is not provided by the statement of profit or loss. For
example, it identifies whether cash has increased or decreased from one year to the
next and also where the cash has come from.

Statements of cash flow are a useful addition to the financial statements of a


company because accounting profit is not the only indicator of performance. They
concentrate on the sources and uses of cash and are a useful indicator of a
company's liquidity and solvency. Also, users of accounts can readily understand

___________________________________________________________________________

339
cash flows, as opposed to statements of profit or loss and statements of financial
position which are subject to manipulation by the use of different accounting policies.
However, the main weakness of a statement of cash flow is that it is a historic
statement. Therefore, it does not indicate whether the business will be able to meet
its debts in the future. A more helpful statement would be a forecast statement of
cash flow.

23.3 ACCA SYLLABUS GUIDE OUTCOME 4:


Classify the effect of transactions on cash flows

IAS 7, Statements of Cash Flows, splits cash flows into the following headings:
1. Cash flows from operating activities
2. Cash flows from investing activities
3. Cash flows from financing activities

Cash flows

outflows inflows

Cash Cash
equivalents

Cash on hand Short term highly liquid


Demand deposits investments e.g. current
investments

23.3.1 Cash flows from operating activities


These represent cash flows derived from operating or trading activities. There
are two methods which can be used to find the net cash from operating
activities: - direct and indirect method. These will be discussed in the next
sections.

23.3.2 Cash flows from investing activities


These are related to the acquisition or disposal of any non-current assets or
investments together with returns received in cash from investments, i.e.
dividends and interest.
___________________________________________________________________________

340
23.3.3 Cash flows from financing activities
Financing cash flows comprise receipts from or repayments to external
providers of finance in respect of principal amounts of finance. For e.g.:
(i) Cash proceeds from issuing shares
(ii) Cash proceeds from issuing debentures, loans, notes, bonds,
mortgages and other short or long term borrowings
(iii) Cash repayments of amounts borrowed
(iv) Dividends paid to shareholders

In order to calculate such figures the closing statement of financial position figure for
debt or share capital and share premium is compared with the opening position for
the same items.

23.3.4 Statement of cash flows for the year ended 31 December 20X7
(INDIRECT METHOD)
$000 $000
Cash flows from operating activities
Profit before taxation 3,390
Adjustment for:
Depreciation 450
Investment income (500)
Interest expense 400
3,740
Increase in trade and other receivables (500)
Decrease in inventories 1,050
Decrease in trade payables (1,740)
Cash generated from operations 2,550
Interest paid (270)
Income taxes paid (900)
Net cash from operating activities 1,380

Cash flows from investing activities


Purchase of property, plant and equipment (900)
Proceeds from sale of equipment 20
Interest received 200
Dividends received 200
Net cash used in investing activities (480)

Cash flows from financing activities


Proceeds from issue of share capital 250
Proceeds from long-term borrowings 250
Dividends paid* (1,290)
Net cash used in financing activities (790)
Net increase in cash and cash equivalents 110
Cash and cash equivalents at beginning of period 120
Cash and cash equivalents at end of period 230
* This could also be shown as an operating cash flow.
___________________________________________________________________________

341
23.4 ACCA SYLLABUS GUIDE OUTCOME 5:
Calculate the figures needed for the statement of cash flows including:
i) Cash flows from operating activities
ii) Cash flows from investing activities
iii) Cash flows from financing activities
Prepare extracts from statements of cash flows from given information

Lecture Example 1

Extracts from ACD Co’s statements of financial position show the following items of
property, plant and equipment at net book value:

30 June
20X7 20X6
$ $
Property, plant and equipment
Freehold property 1,230,000 750,000
Plant and equipment 465,000 380,000
Furniture and fixtures 90,000 105,000

The building element of the freehold property was depreciated by $6,000 and then
revalued on 30 June 20X7 by $95,000. Plant and equipment, which had cost
$49,000 when purchased in January 20X2 on which $35,000 of depreciation had
been charged, was disposed of in November 20X6 for $8,000. Depreciation on the
plant and equipment for the year amounted to $37,000. Depreciation of $55,000 has
been charged on furniture and fixtures.

a. What is the total figure for depreciation in ‘cash flows from operating
activities’ in respect of property, plant and equipment? ______________

b. What is the figure for the profit or loss on disposal to be included in


‘cash flows from operating activities’? _______________

c. What is the total expenditure on property, plant and equipment included


under ‘cash flows from investing activities’?

d. What is the figure for proceeds from disposal of plant and equipment to
be included under ‘cash flows from investing activities’? ____________

___________________________________________________________________________

342
Lecture Example 2

These extracts have been taken from the accounts of Clarkes Co.

Statement of financial position (extracts)


31 October 31 October
20X8 20X7
Current liabilities
Income tax payable 9,850 8,750

The income tax charge during the year was $14,500.

What will appear as “income tax paid” in the statement of cash flows for the
year ended 31 October 20X8? ____________

Lecture Example 3

These extracts have been taken from the accounts of Johns Co.

Statement of financial position (extracts)


31 October 31 October
20X8 20X7
Current liabilities
Dividends payable 9,750 5,750

Dividends charged to retained earnings during the year were $15,500.

What will appear as “dividends paid” in the statement of cash flows for the
year ended 31 October 20X8?

A. $5,750
B. $11,500
C. $15,500
D. $21,250

23.5 ACCA SYLLABUS GUIDE OUTCOME 6:


Calculate the cash flow from operating activities using the indirect and direct
method

23.5.1 The direct method

In the direct method, the cash records of the business are analysed for the period,
picking out all payments and receipts relating to operating activities. These are
summarised to give the net figure for the cash flow statement. Not many businesses

___________________________________________________________________________

343
adopt this approach as it can be quite time consuming. However, this is the preferred
method under IAS 7.

$000 $000
Cash flows from operating activities
Cash receipts from customers 30,150
Cash payments to suppliers and employees (27,600)
Cash generated from operations 2,550
Interest paid (270)
Income taxes paid (900)
Net cash from operating activities 1,380

Lecture Example 4

The following information is available about the transactions of Mermot, a limited


liability company, for the year ended 31 December 20X1.

$000
Depreciation 880
Cash paid for expenses 2,270
Increase in inventories 370
Cash paid to employees 2,820
Decrease in receivables 280
Cash paid to suppliers 4,940
Decrease in payables 390
Cash received from customers 12,800
Net profit before taxation 2,370

Mermot has no interest payable or investment income.

Required: -
Compute Mermot’s net cash flow from operating activities for the company’s
cash flow statement for the year ended 31 December 2001 using: -

a. Direct method
b. Indirect method

___________________________________________________________________________

344
Lecture Example 5

The statements of financial position of RCA Malta, a limited liability company, at 30


June 20X5 and 20X6 are as follows

Notes:
1. The depreciation charge for the year was $13,000,000
2. $6,200,000 was paid during the year to settle the income tax liability at 30
June 20X5.
3. The additional loan notes were issued on 1 January 2006. All interest due was
paid on 31 December 20X5 and 30 June 20X6.
4. Dividends paid during the year totalled $4,000,000.

Required:
Prepare the statement of cash flow for the company for the year ended 30 June
20X6, using the format in IAS 7 Statements of Cash Flow.

___________________________________________________________________________

345
Further Questions

1. At the start of the accounting period the company has a tax liability of $50 and
at the reporting date a tax liability of $90. During the year the tax charged in
the statement of profit or loss was $100.
Required: Calculate the tax paid
2. At the start of the accounting period the company has PPE with a carrying
amount of $100. At the reporting date the carrying amount of the PPE is $300.
During the year depreciation charged was $20, a revaluation surplus of $60
was recorded and PPE with a carrying amount of $15 was sold.
Required: Calculate the cash paid to buy new PPE.
3. At the start of the accounting period the company has retained earnings of
$500 and at the reporting date retained earnings are $700. During the
reporting period a profit for the year of $450 was reported.
Required: Calculate the dividend paid.
4. Extracts from the financial statements are as follows:

Operating profit 80,000

Investment income 12,000

Finance costs (10,000)

Profit before tax 82,000

Tax (32,000)

Profit for the year 50,000

Other comprehensive income

Revaluation gain 40,000

Total comprehensive income 90,000

___________________________________________________________________________

346
Closing balance Opening balance

Inventory 30,000 25,000

Receivables 20,000 26,000

Current liabilities

Trade payables 14,000 11,000

Additional information
During the year depreciation of $50,000 and amortisation of $40,000 was charged to
profit.

Receipts from customers, combined with cash sales, were $800,000, payments to
suppliers of raw materials $400,000, other operating cash payments were $100,000
and cash paid on behalf and to employees was $126,000.

Interest paid is $12,000 and taxation paid is $13,000.

Required :
(a) Using the direct method, prepare the operating activities section of the statement
of cash flows.
(b) Using the indirect method, determine the operating activities section of the
statement of cash flows.

Statement of Cash Flows – Further Illustrations2

2 Clendon T ., Cash Flow Statements, February 2016, http://www.accaglobal.com/gb/en/student/exam -support-resources/fundamentals-


exams-study-resources/f3/technical-articles/cashflow-statements.html
___________________________________________________________________________

347
1. At the start of the accounting period the company has a tax liability of $50 and
at the reporting date a tax liability of $90. During the year the tax charged in
the statement of profit or loss was $100.
Required: Calculate the tax paid.

Income Tax Payable A/c

Tax paid (missing


figure) 60 Bal b/d 50

Bal c/d 90 P/L (tax expense) 100


150 150

2. At the start of the accounting period the company has PPE with a carrying
amount of $100. At the reporting date the carrying amount of the PPE is $300.
During the year depreciation charged was $20, a revaluation surplus of $60
was recorded and PPE with a carrying amount of $15 was sold.
Required: Calculate the cash paid to buy new PPE.

PPE A/c

Bal b/d 100 Depnexp 20


Revaluation reserve 60 Disposal 15
Bank ( missing figure) 175 Bal c/d 300
335 335

3. At the start of the accounting period the company has retained earnings of
$500 and at the reporting date retained earnings are $700. During the
reporting period a profit for the year of $450 was reported.
Required: Calculate the dividend paid

Retained earnings A/c

Dividends paid (missing


figure) 250 Bal b/d 500

___________________________________________________________________________

348
Bal c/d 700 Profit for the year 450
950 950

4. Below find the extracts from the financial statements: -

Additional information

During the year depreciation of $50,000 and amortisation of $40,000 was charged to
profit.

___________________________________________________________________________

349
Receipts from customers, combined with cash sales, were $800,000, payments to
suppliers of raw materials $400,000, other operating cash payments were $100,000
and cash paid on behalf and to employees was $126,000.

Interest paid is $12,000 and taxation paid is $13,000.

Required:-
(a) Using the direct method prepare the operating activities section of the statement
of cash flows.
(b) Using the indirect method determine the operating activities section of the
statement of cash flows.

Operating Activities - Direct Method


Cash received from customers 800,000
Cash paid to suppliers -400,000
Cash paid to employees -100,000
Other operating payments -126,000
Cash generated from operations 174,000
Interest paid -12,000
Income taxes paid -13,000
Net cash from operating activities 149,000

Operating Activities - Indirect Method


Profit before tax 82,000
Adjustments:
Depreciation 50,000
Amortisation 40,000
Investment income -12,000
Finance cost 10,000
170,000
Increase in inventory (30 - 25) -5,000
Decrease in receivables (20 - 26) 6,000
Increase in payables (14 - 11) 3,000
Cash generated from operations 174,000
Interest paid -12,000
Income taxes paid -13,000
Net cash from operating activities 149,000

___________________________________________________________________________

350
__________________________

KEY NOTES
_________________

1. What is the difference between profit and cash?


___________________________________________________________________________

351
A business may appear profitable on its statement of profit or loss, however if its
cash outflow exceeds its cash inflow over a prolonged period then it will not survive.

An expanding company might have negative operating cash flow as it builds up the
level of its inventories and receivables in line with the increased turnover. However,
an increase in working capital without an increase in turnover might indicate
operational inefficiencies and will lead to liquidity problems.

2. The advantages and disadvantages of a statement of cash flows

Advantages: -

i. It provides a clear and understandable picture of cash movements over the


financial year.
ii. A statement of cash flow provides useful additional information that is not
provided by the statement of profit or loss. For example, it identifies whether
cash has increased or decreased from one year to the next and also where
the cash has come from.
iii. It is a useful addition to the financial statements of a company because
accounting profit is not the only indicator of performance. Statements of cash
flow concentrate on the sources and uses of cash and are a useful indicator of
a company's liquidity and solvency.
iv. Users of accounts can readily understand cash flows, as opposed to
statements of profit or loss and statements of financial position which are
subject to manipulation by the use of different accounting policies.

Disadvantages: -

i. It is a historic statement. Therefore, it does not indicate whether the business


will be able to meet its debts in the future.

3. Difference between cash and cash equivalents

Cash comprises cash on hand and on demand deposits, less bank overdrafts.

Cash equivalents are short term, highly liquid investments such as current asset
investments (shares) which can be converted into known amounts of cash relatively
quickly without having a major impact on the entity’s activities.

4. The Statement of Cash Flows

___________________________________________________________________________

352
Cash flows

Operating Investing Financing


Activities Activities Activities

Cash and cash Acquisition and Cash raised from


equivalents disposal of non- the issue of shares
generated and current assets and and loans; cash
used by trading return on used in repayment
activities investments of loans and
payments of
dividends

Direct Method Indirect


Method

___________________________________________________________________________

353
__________________________

QUESTION BANK
_________________

1. IAS 7 requires the statement of cash flows to open with the calculation of net
cash from operating activities arrived at by adjusting net profit before taxation.
Which of the following lists consists only of items which could appear in such a
calculation?

___________________________________________________________________________

354
A. Depreciation, increase in receivables, decrease in payables, proceeds from sale
of equipment, increase in inventories
B. Increase in payables, decrease in inventories, profit on sale of plant,
depreciation, decrease in receivables
C. Increase in payables, proceeds from sale of equipment, depreciation, decrease
in receivables, increase in inventories
D. Depreciation, interest paid, proceeds from sale of equipment, decrease in
inventories.

2. A draft statement of cash flows contains the following calculation of cash flows
from operating activities:
$m
Profit before tax 13
Depreciation 2
Decrease in inventories (3)
Decrease in trade and other receivables 5
Decrease in trade payables 4
Net cash inflow from operating activities 21

Which corrections need to be made to the calculation?

3. A limited liability company sold a building at a profit.

How will this transaction be treated in the company's statement of cash flows?

Proceeds of sale Profit on sale


A. Cash inflow under financing activities Add to profit in calculating cash flow
from operating activities
B. Cash inflow under investing activities Deducted from profit in calculating
cash flow from operating activities
C. Cash inflow under investing activities Added to profit in calculating cash
flow from operating activities
D. Cash inflow under financing activities Deducted from profit in calculating
cash flow from operating activities

4. Which of the following items could appear as items in a company’s


statement of cash flows?

1. A bonus issue of shares


2. A rights issue of shares
3. Revaluation of non-current assets
4. Dividends paid
___________________________________________________________________________

355
A. All four items
B. 1, 3 and 4 only
C. 2 and 4 only
D. 2 and 3 only

5. Which of the following assertions about cash flow statements is/are


correct?

1. A cash flow statement prepared using the direct method produces a different
figure for operating cash flow from that produced if the indirect method is used.
2. Rights issues of shares do not feature in cash flow statements.
3. A surplus on revaluation of a non-current asset will not appear as an item in a
cash flow statement.
4. A profit on the sale of a non-current asset will appear as an item under Cash
Flows from Investing Activities in a cash flow statement.

A. 1 and 4
B. 2 and 3
C. 3 only
D. 2 and 4

___________________________________________________________________________

356
__________________________

ANSWER BANK
_________________

1. B
___________________________________________________________________________

357
2. Decrease in inventories should be added, decrease in payables should be
deducted.

3. B - The proceeds will appear under investing activities and any profit will be
deducted under operating activities.

4. C Only cash items appear in the statement of cash flows. Therefore items 1
and 3 are incorrect as they do not involve cash movements.

5. C - A rights issue is for cash. A bonus issue is “for free”; hence it is not
included in the statement of cash flows.

Chapter 24
GROUP ACCOUNTING:
CONSOLIDATED

___________________________________________________________________________

358
STATEMENT OF FINANCIAL POSITION -
SUBSIDIARY

24.1 ACCA SYLLABUS GUIDE OUTCOME 1:


Define and describe the following terms in the context of group accounting:-
i. Parent
ii. Subsidiary
iii. Control
iv. Consolidated or group financial statements
v. Non-controlling interest
vi. Trade/simple investment

i. Parent : - an entity that has one or more subsidiaries.

ii. Subsidiary: - an entity, including an unincorporated entity such as a


partnership, that is controlled by another entity (known as the parent).

iii. Control: - the power to govern the financial and operating policies of an entity
so as to obtain benefits from its activities

iv. Consolidated or group financial statements: - the financial statements of a


group presented as those of a single economic entity.

v. Non-controlling interest: – Non-controlling interest (NCI) arises when the


parent entity controls a subsidiary but does not own 100% of it; e.g. if P owns
only 70% of the ordinary shares of S, there is a NCI of 30%

vi. Trade/simple investment: - an investment in the shares of another entity,


that is held for the accretion of wealth, and is not an associate or a subsidiary.
Trade investments are shown as investments under non-current assets in the
consolidated SFP of the group.

24.2 ACCA SYLLABUS GUIDE OUTCOME 2:


Identify subsidiary within a group structure

___________________________________________________________________________

359
Owns more than 50% of equity shares
i.e.
P controls S

P is an individual legal entity, known as the parent. The parent is an entity that has
one or more subsidiaries.

S is an individual legal entity, known as the subsidiary.

P owns more than 50% of the ordinary shares of S. It has enough voting power to
appoint all the directors of S. P has the power to govern the financial and operating
policies of an entity so as to obtain benefits from its activities.

Although P and S remain distinct, in economic substance, they can be regarded as a


single unit, the group.

Although control is usually based on ownership of more than 50% of voting power,
IAS 273 lists the following situations where control exists, even when the parent
owns only 50% or less of the voting power of an enterprise.

(a) The parent has power over more than 50% of the voting rights by virtue of
agreement with other investors

(b) The parent has power to govern the financial and operating policies of
the enterprise by statute or under an agreement

(c) The parent has the power to appoint or remove a majority of members of
the board of directors (or equivalent governing body)

(d) The parent has power to cast a majority of votes at meetings of the
board of directors

Consolidated financial statements present the results of the group; they do not
replace the financial statements of the individual group companies.

24.3 ACCA SYLLABUS GUIDE OUTCOMES 3 AND 4:


Describe the components of and prepare a consolidated statement of financial
position or extracts thereof including:-

3 IAS 27 – Consolidated and separate financial statements

___________________________________________________________________________

360
(i) Fair value adjustments at acquisition on land and buildings (excluding
depreciation adjustments)

(ii) Fair value of consideration transferred from cash and shares (excluding
deferred and contingent consideration)

(iii) Elimination of inter-group trading balances (excluding cash and goods


in transit)

(iv) Removal of unrealised profit arising on inter-group trading

(v) Acquisition of subsidiaries part-way through the financial year

Calculate goodwill (excluding impairment of goodwill) using the full goodwill


method only

24.3.1 Preparing a consolidated SFP:

1. Take the individual accounts of the parent and subsidiary and cancel out
items which appear as an asset in one company and a liability in another, e.g.
receivables in one company and payables in another.
2. Add together all the uncancelled assets and liabilities throughout the group on
a line by line basis.
3. The investment in the subsidiary (S) shown in the parent’s (P) statement of
financial position is replaced by the net assets of S.

The consolidated statement of financial position shows:

 The net assets of the whole group (P + S)

 The share capital of the group which always equals the share capital of P only
and

 The retained profits, comprising profits made by the group (i.e. all of P’s
historical profits + profits made by S post-acquisition).

24.3.2 Goodwill

The value of a company will normally exceed the value of its net assets. The
difference is goodwill. This goodwill represents assets not shown in the statement of
financial position of the acquired company such as the reputation of the business
and the loyalty of staff.
Where less than 100% of the subsidiary is acquired, the value of the subsidiary
comprises two elements:

 The value of the part acquired by the parent;


___________________________________________________________________________

361
 The value of the part not acquired by the parent, known as the non-controlling
interest.

Positive goodwill is:-

1. An intangible non-current asset in the SFP


2. Tested annually for impairment (amortisation of goodwill is not permitted).
Impairment of goodwill is not examinable for F3 purposes

Negative goodwill:-

1. Arises where the cost of the investment is less that the value of net assets
purchased.
2. Negative goodwill is credited directly to the statement of profit or loss.

Although there are two methods in which goodwill may be calculated following the
update to IFRS 3, only the full goodwill method is examined in F3: -

Fair value of non-controlling interest method (Full Goodwill)

This results in 100% of the goodwill being shown in the group statement of financial
position – that belonging to the shareholders of the parent and that belonging to the
non-controlling interest.

24.3.3 Pre- and Post-Acquisition Profits

Pre-acquisition profits are the reserves which exist in a subsidiary company at the
date when it is acquired.

These are included in the goodwill calculation.

Post-acquisition profits are profits made and included in the retained earnings of
the subsidiary company since acquisition.

They are included in group reserves.

Only the group share of the post-acquisition reserves of S is included in the group
statement of financial position, i.e. the reserves of S which arose after acquisition by
P.

N.B. Where the acquisition occurs during the financial year, it is important to
calculate the value of profits at the date of acquisition using time-apportionment,

24.3.4 Non-controlling Interest (NCI)

___________________________________________________________________________

362
As mentioned in Section 1 above, a parent may not own all of the shares in the
subsidiary, e.g. if P owns only 70% of the ordinary shares of S, there is a non-
controlling interest of 30%.

Accounting treatment of a non-controlling interest

 In the consolidated statement of financial position, include all of the net assets of
S
 Transfer back the net assets of S which belong to the non-controlling interest
within the capital and reserves section of the consolidated statement of financial
position. A proportion of goodwill on acquisition is also transferred back to the
NCI.

Lecture Example 1:

The following balances relate to P and S on 31 December 2009.

Parent Co. Subsidiary Co.

$ $
Investment in S 200
Other Net Assets 100 400

Share Capital 100 100


Retained Earnings 200 300

P acquired 80% of S on 1 January 2009 when S’s retained earnings were $80. On
the date of acquisition, the fair value of the non-controlling shareholding in S was
$36.

Prepare the consolidated SFP of the group

Lecture Example 2:

___________________________________________________________________________

363
The following balances relate to P and S on 31 December 2009.

Parent Co. Subsidiary Co.

$ $
Investment in S 200
Other Net Assets 400 500

Share Capital 100 100


Retained Earnings 500 400

P acquired 70% of S on 1 January 2009 when S’s retained earnings were $140. On
the date of acquisition, the fair value of the non-controlling shareholding in S was
$72.

Prepare the consolidated SFP of the group

24.3.5 Fair Value of Assets and Liabilities

The fair value of assets and liabilities is defined in IFRS 3 as ‘the amount for which
an asset could be exchanged or a liability settled between knowledgeable, willing
parties in an arm’s length transaction’.

IFRS 3 requires that the subsidiary’s assets and liabilities are recorded at their fair
value for the purposes of the calculation of goodwill and production of consolidated
accounts.

Adjustments will therefore be required where the subsidiary’s accounts themselves


do not reflect fair value.

(1) Adjust both columns of the net assets calculation to bring the net assets to fair
value at acquisition and reporting date.

(2) At the reporting date, make the adjustment on the face of the SFP when adding
across assets and liabilities.

Lecture Example 3:

___________________________________________________________________________

364
Draft SFPs of P and S on 31 March 2009 are as follows:

Parent Co. Subsidiary Co.

$ $
Cost of investment S 200
Other Net Assets 800 500

Share Capital 100 100


Retained Earnings 900 400

Two years ago P acquired 90% of S when S’s retained earnings were $100. At
acquisition, the fair value of S’s net assets exceeded their book value by $10. Any
difference in fair value is due to land.

On the date of acquisition, the fair value of the non-controlling share of P in S was
$26.
Prepare the consolidated SFP of the group.

Lecture Example 4:

Below are the SFP of P and S as at 31 December 2004.

Parent Co. Subsidiary Co.

$ $
Investment in S 100
Other Net Assets 200 140

Share Capital 100 40


Retained Earnings 200 100

P acquired 80% of S two years ago when S’s retained earnings were $50. At that
date, S’s PPE had a fair value of $10 in excess of the carrying value.

On the date of acquisition, the fair value of the non-controlling shareholding in S was
$20.

Prepare the consolidated SFP of the group.

Lecture Example 5:

___________________________________________________________________________

365
Below are the SFP of P and S as at 31 December 2004.

Parent Co. Subsidiary Co.

$ $
Investment in S 200
Other Net Assets 300 300

Share Capital 100 100


Retained Earnings 400 200

P acquired 60% of S when S’s retained earnings were $110. At that date, S’s land
had a fair value of $10 in excess of book value.

On the date of acquisition, the fair value of the non-controlling shareholding in S was
$88.

Prepare the consolidated SFP of the group.

24.3.6 Share for share exchanges

Share for share exchanges form part, or all, of the cost of investment which is used
in the goodwill calculation.

If this exchange has yet to be accounted for, the double entry is always: -

Dr Cost of Investment
Cr Share capital (with the nominal value of P shares given out)
Cr Share premium (with the premium)

Lecture Example 6:

P acquired 80% of S shares via a 2 for 1 share exchange. At the date of acquisition,
the following balances were in the books of H and S:

Parent Co. Subsidiary Co.


Share Capital ($1) $400 (0.50c) $400
Share Premium $100 $50

The share price of P was $2 at the date of acquisition. This has not been accounted
for.

Show the accounting treatment required to account for the share exchange.
Lecture Example 7:

___________________________________________________________________________

366
Parent Co. Subsidiary Co.
Share Capital ($1) $100 ($1) $100
Share Premium $100 $100

P acquired 80% of S shares via a 3 for 2 share exchange. The share price of P at
acquisition was $3. This has not been accounted for.

Show the accounting treatment required to account for the share exchange.

Lecture Example 8:

Able Co. bought 51,000 shares in Baker on 1.1.2011. Baker had 60,000 shares in
issue on this date.

Able Co. paid $1.25 for every share in Baker and gave Baker’s shareholders 3
shares for every 2 shares acquired. The nominal value of Able’s shares is $1 per
share and their fair value at the date of acquisition has $2.30.
What was the consideration Able has paid to control Baker?

24.3.7 Intra Group Balances

If the companies within the same group trade with each other, then this will probably
lead to:

 A receivables account in one company’s SFP


 A payables account in the other company’s SFP.

These are amounts owing within the group rather than outside the group and
therefore they must not appear in the consolidated statement of financial position.
They are therefore cancelled against each other on consolidation.

Lecture Example 9:

___________________________________________________________________________

367
Berino, a limited liability company, owns 70% of the shares in Muggie. Berino has
payables of $244,000. Muggie has payables of $40,000 of which $6,000 is owed to
Berino. Berino has receivables of $360,000 and Muggie has receivables of
$150,000.

What amounts should be recorded for consolidated receivables and payables


in the group accounts of Berino?

Payables Receivables
$ $
A. 278,000 504,000
B. 194,600 352,800
C. 284,000 510,000
D. 290,000 516,000

24.3.8 Unrealised Profit

Unrealised profit may arise within a group scenario on:


a. Inventory where companies trade with each other
b. Non-current assets where one company has transferred an asset to the other
company within the same group.

a. Adjustment for unrealised profit in inventory

(1) Determine the value of closing inventory which has been purchased from the
other company in the group.
(2) Use mark-up or margin to calculate how much of that value represents profit
earned by the selling company.
(3) Make the adjustments according to who the seller is.
If the seller is the parent company:

Dr Group retained earnings


Cr Group inventory (deduct the profit when adding P’s inventory + S’s
inventory on the face of the consolidated SFP).

If the seller is the subsidiary:

Dr Subsidiary retained earnings


Cr Group inventory (deduct the profit when adding P’s inventory + S’s
inventory on the face of the consolidated SFP).

Lecture Example 10:

___________________________________________________________________________

368
H sells to S some goods at a selling price of $100. H makes 20% profit margin. S
sold ⅓ of these goods at cost.

What is the unrealised profit?

Lecture Example 11:

H sells to S goods worth $600. H makes 20% profit margin. S sells $200 of these
goods at cost.

What is the unrealised profit?

Lecture Example 12:

H sells to S goods worth $1000. H makes 40% profit margin. S sold $400 worth of
goods (at cost).

What is the unrealised profit?

Lecture Example 13:

S sells goods to H for $600. S makes a 20% mark up. H has goods at cost left in
stock worth $200.

What is the unrealised profit?

___________________________________________________________________________

369
b. Adjustment for unrealised profit in the transfer of non-current assets

Occasionally, a non-current asset is transferred within the group (say from a parent
to a subsidiary). The parent may have manufactured the asset as part of its normal
production (and therefore included the sale in revenue), or it may have transferred
an asset previously used as part of its own non-current assets. If the transfer is
done at cost, then, in the first case, the cost of the asset must be removed from both
revenue and cost of sales. In the second case, no elimination would be required.
If one company sells non-current assets to another company in the same group at a
profit, adjustments must be made for:

1. Profit on sale
2. Depreciation

The whole scenario has to be recreated as if the sales have never occurred.

Carrying value at reporting date X


Carrying value at reporting date if intra-group transfer
had not occurred X

Adjustment X

The double-entry of this adjustment is: -

Dr Retained Earnings of the seller


Cr Non-Current Assets (P’s NCA + S’s NCA – Adjustment for UP)

Lecture Example 14:

H sells PPE to S costing $1000 for a selling price of $1500, depreciation at 10% per
annum.

What is the unrealised profit?

Lecture Example 15:

S sold a machine with a NBV of $100 000 to H at a transfer price of $120 000 at the
year start. Group policy dictates that the machine is depreciated over its remaining
life of 5 years.

Calculate the unrealised profit on the sale of the machine.

___________________________________________________________________________

370
Further Questions4

Question 1

Green Co owns the following investments in other companies:

Non-equity shares
Equity shares held
held
Violet Co 80% Nil
Amber Co 25% 80%
Black Co 45% 25%

Green Co also has appointed five of the seven directors of Black Co.

Which of the following investments are accounted for as subsidiaries in the


consolidated accounts of Green Co Group?

A. Violet only
B. Amber only
C. Violet and Black
D. All of them

Question 2

4Ref: “Preparing Simple Consolidated Financial Statements” by J. Lucas, F3/FFA examiner, April 2012,
http://www.accaglobal.com/content/dam/acca/global/PDF-students/2012s/sa_apr12_f3ffa_finstats.pdf
___________________________________________________________________________

371
Pink Co acquired 80% of Scarlett’s Co ordinary share capital on 1 January 2012.

As at 31 December 2012, extracts from their individual statements of financial


position showed:

Pink Co Scarlett Co
$ $
Current assets:
Receivables 50,000 30,000
Current liabilities:
Payables 70,000 42,000

As a result of trading during the year, Pink Co’s receivables balance included an
amount due from Scarlett of $4,600.

What should be shown as the consolidated figure for receivables and payables?

Receivables Payables
$ $
A. 80,000 112,000
B. 75,400 112,000
C. 74,000 103,600
D. 75,400 107,400

Question 3

Red Co acquired 80% of Blue Co’s 40,000 $1 ordinary share capital on 1 January
2012 for a consideration of $3.50 cash per share.

The fair value of the non-controlling interest was $50,000 and the fair value of the net
assets acquired was $145,000.

What should be recorded as goodwill on acquisition of Blue Co in the consolidated


financial statements?

A. $17,000
B. $45,000
C. $46,000
D. $112,000

___________________________________________________________________________

372
__________________________

KEY NOTES
_________________

___________________________________________________________________________

373
1. Parent: - an entity that has one or more subsidiaries.

2. Subsidiary: - an entity, including an unincorporated entity such as a


partnership, that is controlled by another entity (known as the parent).

3. Control: - the power to govern the financial and operating policies of an entity
so as to obtain benefits from its activities.

4. Consolidated or group financial statements: - the financial statements of a


group presented as those of a single economic entity.

5. Non-controlling interest: – Non-controlling interest (NCI) arises when the


parent entity controls a subsidiary but does not own 100% of it.

6. Trade/simple investment: - an investment in the shares of another entity, that


is held for the accretion of wealth, and is not an associate or a subsidiary.

7. Although control is usually based on ownership of more than 50% of voting


power, IAS 27 lists other situations where control exists, even when the
parent owns only 50% or less of the voting power of an enterprise.

P has enough voting power to appoint all the directors of S. P has the power
to govern the financial and operating policies of an entity so as to obtain
benefits from its activities.

8. Goodwill represents assets not shown in the statement of financial position of


the acquired company such as the reputation of the business and the loyalty
of staff.

9. Positive goodwill is:-

 An intangible non-current asset in the SFP


 Tested annually for impairment

10. Negative goodwill:-

 Arises where the cost of the investment is less that the value of net assets
purchased.
 Negative goodwill is credited directly to the statement of profit or loss.

11. Pre-acquisition profits are the reserves which exist in a subsidiary company at
the date when it is acquired.

___________________________________________________________________________

374
12. Post-acquisition profits are profits made and included in the retained earnings
of the subsidiary company since acquisition. Only the group share of the
post-acquisition reserves of S is included in the group statement of financial
position, i.e. the reserves of S which arose after acquisition by P.
13. The value of the part not acquired by the parent is known as the non-
controlling interest.

In the consolidated statement of financial position, include all of the net assets
of S. Transfer back the net assets of S which belong to the non-controlling
interest within the capital and reserves section of the consolidated statement
of financial position. A proportion of goodwill on acquisition is also transferred
back to the NCI.

14. The fair value of assets and liabilities is defined as ‘the amount for which an
asset could be exchanged or a liability settled between knowledgeable, willing
parties in an arm’s length transaction’.

15. Share for share exchanges

Dr Cost of Investment
Cr Share capital (with the nominal value of P shares given out)
Cr Share premium (with the premium)

16. Amounts owing within the group rather than outside the group must not
appear in the consolidated statement of financial position. They are cancelled
against each other on consolidation.

17. Unrealised profit may arise within a group scenario on:


 Inventory where companies trade with each other
 Non-current assets where one company has transferred an asset to the
other company within the same group.

18. Adjustment for unrealised profit in inventory

If the seller is the parent company:

Dr Group retained earnings


Cr Group inventory

If the seller is the subsidiary:

Dr Subsidiary retained earnings


Cr Group inventory

19. Adjustment for unrealised profit in the transfer of non-current assets


___________________________________________________________________________

375
Carrying value at reporting date X
Carrying value at reporting date if intra-group transfer
had not occurred X

Adjustment X
The double-entry of this adjustment is: -

Dr Retained Earnings of the seller


Cr Non-Current Assets

___________________________________________________________________________

376
__________________________

QUESTION BANK
_________________

1. At 1 May 2009 Tibor purchased six million of Kinnot’s ten million $1 ordinary
shares for $6,000,000. At that date Kinnot had net assets with a fair value of
___________________________________________________________________________

377
$8,450,000 and its share price was $1.10. It is group policy to value the non-
controlling interest at the fair value of the subsidiary’s identifiable net assets
using the market value of the shares at acquisition.

What is the total goodwill on acquisition of Kinnot?

A. $930,000
B. $2,450,000
C. $1,550,000
D. $1,950,000

2. Scarfell has 10 million $1 issued ordinary shares. At 1 May 2009 Snowdon


purchased 70% of Scarfell’s $1 ordinary shares for $8,000,000. At that date
Scarfell had net assets with a fair value of $8,750,000 and its share price was
$1·20. It is group policy to value the non-controlling interest at the fair value of the
subsidiary’s identifiable net assets using the market value of the shares at
acquisition.

What was the total goodwill arising on acquisition at 1 May 2009?

A. $4,400,000
B. $350,000
C. $750,000
D. $2,850,000

3. Grape Company has a 49% shareholding in each of the following three


companies:
___________________________________________________________________________

378
Pear Company: Grape Company has the right to appoint or remove a
majority of the directors of Pear Company.
Peach Company: Grape Company has more than half the voting rights in
Peach Company as a result of an agreement with other investors.
Plum Company: Grape Company has the power to govern the financial and
operating policies of Plum Company.

Which of these companies are subsidiaries of Grape Company for financial


reporting purposes?

A. Pear Company and Peach Company only


B. Plum Company and Peach Company only
C. Plum Company and Pear Company only
D. Plum Company, Pear Company and Peach Company

4. Kinder Co has 5 million $1 issued ordinary shares. At 1 May 2010 Peak Co


purchased 60% of Kinder Co’s $1 ordinary shares for $4,000,000. At that date
Kinder Co had net assets with a fair value of $4,750,000 and a share price of
$1·10. Peak Co valued the non-controlling interest in Kinder Co at acquisition as
$2,200,000.

What is the total goodwill on acquisition at 1 May 2010?

A. $1,150,000
B. $1,750,000
C. $750,000
D. $1,450,00

___________________________________________________________________________

379
5. Tomsett Co, a limited liability company, owns 65% of the shares in Frew Co. Frew
Co owes Tomsett Co $5,000. Tomsett Co has receivables of $300,000 and Frew
Co has receivables of $130,000.

What are the consolidated receivables for Tomsett Co?

A. $425,000
B. $381,250
C. $379,500
D. $435,00

6. Which of the following statements apply when producing a consolidated


statement of financial position?

(i) All inter-group balances should be eliminated.


(ii) Inter-group profit in year-end inventory should be eliminated.
(iii) Closing inventory held by subsidiaries needs to be included at fair value.

A. (i) only
B. (i), (ii) and (iii)
C. (i) and (ii) only
D. (iii) only

7. On 30 June 20X2, H acquired 80% of the share capital of S. The non-controlling


interest had a fair value of $1,300,000.
Extracts from the statement of financial position of S at 30 June 20X2 and 30 June
20X6 are shown below:

Statement of Financial Position


30 June 20X2 30 June 20X6
$ $
Ordinary share capital 1,000,000 1,000,000
Share premium account 400,000 400,000
Retained earnings 4,700,000 5,600,000

What figure for non controlling interest should appear in the consolidated
statement of financial position as at 30 June 20X6?

A. $1,220,000
B. $1,300,000
C. $1,480,000
D. $1,400,000

___________________________________________________________________________

380
8. Wheddon Co purchased 60,000 ordinary shares in Raleigh Co for $85,000 five years
ago, when Raleigh Co’s retained earnings were $20,000.

Raligh Co’s equity and reserves at 31 July 20X9 were as follows:


Ordinary shares $1 80,000
Retained earnings 70,000

The fair value of the non-controlling interest at acquisition was $22,000.

What was the goodwill arising on acquisition of Raleigh Co?

A. $7,000
B. $10,000
C. $25,000
D. $43,000

___________________________________________________________________________

381
__________________________

ANSWER BANK
_________________

___________________________________________________________________________

382
1. D

$
Consideration transferred 6,000,000
Fair value of non-controlling interest (4,000,000 x $1.10) 4,400,000
10,400,000
Less fair value of net assets at acquistion (8,450,000)
Goodwill = 1,950,000

2. D

$
Consideration 8,000,000
Fair value of non-controlling interest (3 million x $1·20) 3,600,000
11,600,000
Less fair value of net assets at acquisition (8,750,000)
Goodwill 2,850,000

3. D

4. D

$
Consideration 4,000,000
Fair value of non-controlling interest (2 million x $1·10) 2,200,000
–––––––––
6,200,000
Less fair value of net assets at acquisition (4,750,000)
–––––––––
Goodwill 1,450,000
–––––––––

5. A

Tomsett Co’s receivables 300,000 + Frew Co receivables 130,000 less 5,000 due
from Tomsett Co = 425,000

6. C

___________________________________________________________________________

383
7. C
$000
At acquisition 1,300
% Post acquisition (5,600 – 4,700) x 20% 180
––––––
1,480
––––––

8. A

$
Cost 85,000
NCI 22,000
Shares (80,000)
Retained Earnings (20,000)
––––––
7,000
––––––

___________________________________________________________________________

384
Chapter 25
GROUP ACCOUNTING:
CONSOLIDATED
STATEMENT OF COMPREHENSIVE INCOME -
SUBSIDIARY

25.1 ACCA SYLLABUS GUIDE OUTCOME 1:


Describe the components of and prepare a consolidated statement of profit or
loss or extracts thereof including:-
1. Elimination of inter-group trading balances (excluding cash and goods
in transit)

2. Removal of unrealised profit arising on inter-group trading

3. Acquisition of subsidiaries part-way through the financial year

Basic principles

1. From sales revenue to profit after tax, include all of P’s income and expenses
plus all of S’s income and expenses (where a mid-year acquisition has occurred,
these must be time-apportioned).

2. Once the profit after tax is calculated, deduct share profits due to the non-
controlling interest.

Non-controlling interest

This is calculated as: NCI% x subsidiary’s profit after tax (taken from S’s column of
consolidation schedule).

Dividends

A payment of a dividend by S to P must be cancelled. Any dividend income shown


in the consolidated statement of profit or loss must arise from investments other than
those in subsidiaries or associates.

Unrealised Profits

The adjustment to unrealised profit should be shown as an increase to cost of sales


(add to cost of sales). It affects the books of the SELLER.

___________________________________________________________________________

385
Sales and Purchases

Intra-group trading must be eliminated from the consolidated statement of profit or


loss.

Consolidated sales revenue = P’s revenue + S’s revenue – intra-group sales


Consolidated cost of sales = P’s COS + S’s COS – intra-group sales

Interest on loan

If loans are outstanding between group companies, intra-group loan interest will be
paid and received. Both the loan and loan interest must be excluded from the
consolidated results.

Transfers of non-current assets

If one group company sells a non-current asset to another group company, the
following adjustments are needed in the statement of profit or loss:-
1. Any profit or loss arising on the transfer must be deducted
2. The depreciation charge must be adjusted so that it is based on the cost of
the asset to the group

Mid-year acquisitions

If a subsidiary is acquired part way through the year, then it is important to time
apportion the results of S in the year of acquisition. Unless indicated otherwise,
assume that revenue and expenses accrue evenly.

Lecture Example 1:

Several years ago H acquired 80% of the ordinary share capital of S. Their results
for the year ended 31 December 2005 were as follows:

Statement of profit or loss

H S
(80%)
Revenue 100 100
COS (40) (40)
Expenses (40) (40)
Profit after Tax 20 20___

___________________________________________________________________________

386
Prepare the consolidated statement of profit or loss for the year ended 31
December 2005.

Lecture Example 2:

Several years ago H acquired 80% of the ordinary share capital of S. Their results
for the year ended 31 December 2005 were as follows:

Statement of profit or loss

H S
(80%)
Revenue 1000 800
COS (600) (200)
Expenses (100) (100)
Tax (100) (100)__
Profit after Tax 200 400___

H acquired 80% of S. At that date, 3 years ago, S’s PPE had a fair value of $100 in
excess of the carrying value and a 5 year useful economic life. Depreciation is
charged to COS.

Prepare the consolidated statement of profit or loss for the year ended 31
December 2005.

Lecture Example 3:

Exe Co acquired 70% of the ordinary share capital of Barle Co six years ago. The
following information relates to Barle Co for the year ended 30 September 20X3.
$
Sales revenue 480,000
Cost of sales 270,000
Administration expenses 90,000
Taxation 30,000

What is the profit attributable to the non-controlling interest in the


consolidated statement of profit or loss?
A. $27,000

B. $63,000

C. $36,000

D. $84,000

___________________________________________________________________________

387
Other Comprehensive Income

The consolidated statement of comprehensive income is produced using the


consolidated statement of profit or loss as a basis. Remember that in F3/FFA, the
only item of other comprehensive income you may have is the revaluation of PPE.
This is shared between the owners of the parent and NCI according to the
percentage of their investment.

Example

S Co made a $30,000 revaluation gain on its property during the year. P Co has
acquired 70% of the equity of S Co five years ago.

Other comprehensive income attributable to: -

Owners of the parent (70% x 30,000) 21,000


Non-controlling interest (30% x 30,000) 9,000

Lecture Example 45:

Purple Co acquired 70% of the voting share capital of Silver Co on 1 October 2011.

The following extracts are from the individual statements of profit or loss of the two
companies for the year ended 30 September 2012:

Purple Co Silver Co
$ $
Revenue 79,300 29,900
Cost of sales (54,990) (17,940)
Gross Profit 24,310 11,960

Purple Co had made sales to Silver Co during the year of $5,000. Purple Co had
originally purchased the goods at a cost of $4,000. Half of these items remained in
inventory at the year end.

What should be the consolidated revenue for the year ended 30 September 2012?
______________

What should be the consolidated cost of sales for the year ended 30 September
2012? ___________

5Ref: “Preparing Simple Consolidated Financial Statements” by J. Lucas, F3/FFA examiner, April 2012,
http://www.accaglobal.com/content/dam/acca/global/PDF-students/2012s/sa_apr12_f3ffa_fins tats.pdf
___________________________________________________________________________

388
__________________________

KEY NOTES
_________________

___________________________________________________________________________

389
1. Non-controlling interest
This is calculated as: NCI% x subsidiary’s profit after tax (taken from S’s
column of consolidation schedule).

2. Dividends
A payment of a dividend by S to P must be cancelled. Any dividend income
shown in the consolidated statement of profit or loss must arise from
investments other than those in subsidiaries or associates.

3. Unrealised Profits
The adjustment to unrealised profit should be shown as an increase to cost of
sales. It affects the books of the SELLER.

4. Sales and Purchases


Intra-group trading must be eliminated from the consolidated statement of
profit or loss.

Consolidated sales revenue = P’s revenue + S’s revenue – intra-group sales


Consolidated cost of sales = P’s COS + S’s COS – intra-group sales

5. Interest on loan
If loans are outstanding between group companies, intra-group loan interest
will be paid and received. Both the loan and loan interest must be excluded
from the consolidated results.

6. Transfers of non-current assets


If one group company sells a non-current asset to another group company,
the following adjustments are needed in the statement of profit or loss:-
 Any profit or loss arising on the transfer must be deducted
 The depreciation charge must be adjusted so that it is based on the cost of
the asset to the group

7. Mid-year acquisitions
If a subsidiary is acquired part way through the year, then it is important to
time apportion the results of S in the year of acquisition. Unless indicated
otherwise, assume that revenue and expenses accrue evenly.

___________________________________________________________________________

390
__________________________

QUESTION BANK
_________________

___________________________________________________________________________

391
1. The summarised statements of profit or loss of Big Co and Small Co, for the year
ended 31 October 2010, are provided below. Big Co acquired 3,600,000 ordinary
shares in Small Co for $5,250,000 on 1 November 2009 when the retained earnings
of Small Co were $300,000. On the same date, Big Co also acquired 40% of Small
Co’s loan notes of $400,000.

Statement of profit or loss for the year ended 31 October 2010

Big Co Small Co
$000 $000
Revenue 9,600 3,900
Cost of sales (5,550) (2,175)
––––––– ––––––
Gross profit 4,050 1,725
Distribution costs (1,050) (480)
Administrative expenses (1,650) (735)
Finance costs – (25)
Income from Small Co: Loan note interest 10 –
Dividends 150 –
––––––– ––––––
Profit before tax 1,510 485
Income tax expense (600) (120)
––––––– ––––––
Profit for the year 910 365
––––––– ––––––

The following information is also available:

(i) Small Co’s total share capital consists of 6,000,000 ordinary shares of $1
each.

(ii) It is group policy to value the non-controlling interest at full fair value. The fair
value of the non-controlling interest at the acquisition date was $3,200,000.

(iii) During the year ended 31 October 2010, Big Co sold goods costing $200,000
to Small Co for $300,000. At 31 October 2010, 50% of these goods remained
in Small Co’s inventory.

Required:

a) Calculate the goodwill arising on the acquisition of Small Co.

$000 $000

___________________________________________________________________________

392
Consideration transferred
NCI
Assets at acquisition:
Share capital
Retained earnings

Goodwill on acquisition

b) Complete the consolidated statement of profit or loss for Big Co for the
year ended 31 October 2010.

Big Co
Consolidated statement of profit or loss for the year ended 31 October 2010

$000
Revenue
Cost of sales
Gross profit
Distribution costs
Administrative expenses
Finance cost
Profit before tax
Income tax expense
Profit for the year

Profit attributable to:


Owners of the parent
Non-controlling interest

___________________________________________________________________________

393
2. You are presented with the following information for Bradshaw, a limited liability
company, and its subsidiary, Martin:

Statement of profit or loss for the year ended 31 October 2009

Bradshaw Martin
$000 $000
Revenue 125,000 77,900
Cost of sales (65,000) (38,500)
–––––––– ––––––––
Gross profit 60,000 39,400
Distribution costs (6,750) (8,050)
Administrative expenses (17,500) (9,780)
Finance costs – (20)
Income from Martin: Loan note interest 15 –
Dividends 5,200 –
–––––––– ––––––––
Profit before tax 40,965 21,550
Income tax expense (19,250) (10,850)
–––––––– ––––––––
Profit for the year 21,715 10,700
–––––––– ––––––––

Statements of financial position as at 31 October 2009

Bradshaw Martin
ASSETS $000 $000 $000 $000
Non-current assets
Property, plant and equipment 75,000 31,901
Investments:
$1 ordinary shares in Martin at cost 34,000 –
Martin loan notes 150 –
––––––– ––––––
109,150 31,901
Current assets
Inventory, at cost 9,750 4,162
Receivables 17,125 11,325
Cash and cash equivalents 3,150 30,025 1,255 16,742
––––––– –––––– –––––– ––––––
Total assets 139,175 48,643
–––––– ––––––
EQUITY AND LIABILITES
Capital and Reserves
$1 Ordinary shares 77,000 23,150
Retained earnings 35,362 9,538

___________________________________________________________________________

394
––––––– ––––––
Total equity 112,362 32,688
Non-current liabilities
10% Loan note – 200
Current liabilities
Payables 16,613 9,500
Tax 10,200 6,255
Total liabilities 26,813 15,755
–––––– ––––––– ––––– ––––––
Total equity and liabilities 139,175 48,643
––––––– ––––––

The following information is also available:

(i) Bradshaw purchased 18,520,000 $1 ordinary shares in Martin on 1 November


2008. At that date Martin’s retained earnings were $5,338,000.

(ii) It is group policy to value the non-controlling interest at full fair value. The fair
value of the non-controlling interest at the acquisition date was $7,408.

(iii) Bradshaw owns $150,000 of Martin’s loan notes. The annual interest of
$15,000 due to Bradshaw has not been paid and is included in Martin’s
payables and Bradshaw’s receivables.

(iv) During the year ended 31 October 2009 Bradshaw sold goods to Martin for
$15,000,000. Bradshaw made a profit on these goods of $2,500,000. Martin
still has all of these goods in inventory at 31 October 2009.

(v) At 31 October 2009 Martin owed Bradshaw $3,000,000 for some of the goods
that Bradshaw supplied during the year.

(vi) All Martin’s dividends of $6,500,000 were paid in the financial year ended 31
October 2009.

Required:

(a) Calculate the goodwill arising on the acquisition of Martin as at 1


November 2008.

___________________________________________________________________________

395
$000 $000
Consideration transferred
NCI
Assets at acquisition
Share capital
Retained earnings

(b) Prepare the following financial statements for Bradshaw:


(i) the consolidated statement of profit or loss for the year ended 31
October 2009;

Bradshaw
Consolidated statement of profit or loss for the year ended 31 October 2009

$000
Revenue
Cost of sales

Gross profit
Distribution costs
Administrative expenses
Finance costs

Profit before tax


Income tax expense

PROFIT FOR THE YEAR

Profit attributable to:


Owners of the parent
Non-controlling interest

___________________________________________________________________________

396
(ii) the consolidated statement of financial position as at 31 October
2009.

Bradshaw
Consolidated statement of financial position as at 31 October 2009

ASSETS $000 $000


Non-current assets
Property, plant and equipment
Intangible – goodwill

Current assets
Inventory, at cost
Receivables
Cash and cash equivalents

Total assets

EQUITY AND LIABILITY


Capital and Reserves
$1 Ordinary shares
Retained earnings

Non-controlling interest

Total equity
Non-current liabilities
10% Loan note
Current liabilities
Payables
___________________________________________________________________________

397
Tax
Total current liabilities

Total equity and liabilities

Prepare your answers to the nearest $000. (CAT Paper T6 Section B Question 1)

___________________________________________________________________________

398
__________________________

ANSWER BANK
_________________

___________________________________________________________________________

399
1. (a) Goodwill on acquisition of Small Co

$000 $000
Consideration transferred 5,250
NCI 3,200
Assets at acquisition:
Share capital 6,000
Retained earnings 300 (6,300)

Goodwill on acquisition 2,150

(b) Big Co
Consolidated statement of profit or loss for the year ended 31 October 2010

$000
Revenue (9,600 + 3,900 – 300) 13,200
Cost of sales {5,550 + 2,175 – 300 + (50% x 100)} (7,475)
Gross profit 5,725
Distribution costs (1,530)
Administrative expenses (2,385)
Finance cost (25 – 10) (15)
Profit before tax 1,795
Income tax expense (720)
Profit for the year 1,075

Profit attributable to:


Owners of the parent 929
Non-controlling interest (365 x 40%) 146
1,075

___________________________________________________________________________

400
2. (a) Goodwill on acquisition of Martin

$000 $000
Consideration transferred 34,000
NCI (23,150 – 18,520) x $1·6 7,408
Assets at acquisition
Share capital 23,150
Retained earnings 5,338 (28,488)
––––––
12,920
––––––

(b) (i) Bradshaw


Consolidated statement of profit or loss for the year ended 31 October 2009

$000
Revenue (125,000 + 77,900 – 15,000) 187,900
Cost of sales (65,000 + 38,500 - 15,000 + 2,500*) (91,000)
––––––
Gross profit 96,900
Distribution costs (14,800)
Administrative expenses (27,280)
Finance costs (20 – 15) (5)
––––––
Profit before tax 54,815
Income tax expense (30,100)
––––––
PROFIT FOR THE YEAR 24,715

Profit attributable to:


Owners of the parent 22,575
Non-controlling interest (20% x 10,700) 2,140
––––––
24,715

___________________________________________________________________________

401
(ii) Bradshaw
Consolidated statement of financial position as at 31 October 2009

ASSETS $000 $000


Non-current assets
Property, plant and equipment 106,901
Intangible – goodwill 12,920
––––––––
119,821
Current assets
Inventory, at cost (9,750 + 4,162 – 2,500*) 11,412
Receivables (17,125 + 11,325 - 3,000** - 15***) 25,435
Cash and cash equivalents 4,405 41,252
–––––––– ––––––––
Total assets 161,073

EQUITY AND LIABILITY


Capital and Reserves
$1 Ordinary shares 77,000
Retained earnings (W1) 36,222
––––––––
113,222
Non-controlling interest (W2) 8,248
––––––––
Total equity 121,470
Non-current liabilities
10% Loan note (200 – 150) 50
Current liabilities
Payables (16,613 + 9,500 – 3,000** – 15***) 23,098
Tax 16,455
––––––––
Total current liabilities 39,553
––––––––
Total equity and liabilities 161,073

Notes:

* Remove the unrealised profit on goods still held at year-end ($2,500,000),


** Remove intra-group balances ($3,000,000),
*** Remove intra-group balances ($15,000)

___________________________________________________________________________

402
Workings
W1 - Retained earnings as at 31 October 2009

$000 $000
Bradshaw as per statement of
financial position 35,362
Less unrealised profit (2,500)
Martin :
Retained earnings 9,538
Pre-acquisition reserves (5,338)
–––––
4,200
Group share (80% x $4,200,000) 3,360
––––––
36,222

W2 - Non-controlling interest

NCI at acquisition date 7,408


Share of post-acquisition profit {(9,538 – 5,338) x 20%} 840
––––––
8,248

___________________________________________________________________________

403
Chapter 26
GROUP ACCOUNTING:
ASSOCIATE

26.1 ACCA SYLLABUS GUIDE OUTCOME 1:


Define and identify an associate and significant influence and identify the
situations where significant influence or participating interest exists
Describe the key features of a parent-associate relationship and be able to
identify an associate within a group structure

IAS 28 defines an associate as:

An entity over which the investor has significant influence but not control or joint
control and that is neither a subsidiary nor an interest in joint venture.

Significant influence is the power to participate in the financial and operating policy
decisions of the investee but is not in control or joint control over those policies.

There are several indicators of significant influence, but the most important are
usually considered to be a holding of between 20% and 50% of the voting shares
and board representation.

The existence of significant influence by an investor is usually evidenced in one or


more of the following ways:

 representation on the board of directors or equivalent governing body of the


investee
 participation in the policy-making process
 material transactions between the investor and the investee
 interchange of managerial personnel
 provision of essential technical information

26.2 ACCA SYLLABUS GUIDE OUTCOME 2:


Describe the principle of equity accounting

Equity accounting brings an associate investment into the parent company’s


financial statements initially at cost.

The basic principle of equity accounting is that P Co should take account of its share
of the earnings of A Co whether or not A Co distributes the earnings as dividends.
A’s sales revenue, cost of sales, expenses and revenue are not added with those of
the group. Instead the group share only of A’s profit after tax is included in the
consolidated statement of profit or loss as a single amount.
___________________________________________________________________________

404
P Co should also include its share of A Co’s other comprehensive income in its
consolidated statement of comprehensive income.

In the consolidated statement of financial position, the associate is included as a


non-current asset investment, calculated as:

$’000
Cost of investment X
P’s share of post acquisition profits of A X
Less: impairment losses of A (X)
X

Lecture Example 1:

X has a 40% shareholding in each of the following three companies:


P: X has the right to appoint or remove a majority of the directors of P.
Q: X has significant influence over the affairs of Q.
R: X has the power to govern the financial and operating policies of R.

Which of these companies are subsidiaries of X for financial reporting purposes?

A. Q and R only

B. P and R only

C. P and Q only

D. P, Q and R

Lecture Example 2:

IAS 28 Investments in Associates governs the identification of associates.


Which of the following would suggest that an entity is an associate of another entity?

A. The investing entity has owned its share since the incorporation of the
investee entity.

B. The investor holds greater than 20% but less than 50% of the voting power of
the investee.

C. The investing entity has some influence over other entities in the same
industry.

D. The investor often trades with the investee.

___________________________________________________________________________

405
Lecture Example 36:

Which of the following investments owned by Indigo Co should be equity accounted


in the consolidated financial statements?
• 30% of the non-voting preference share capital in Yellow Co
• 18% of the ordinary share capital in Blue Co with directors of Indigo Co having two
of the five places on the board of Blue Co
• 45% of the ordinary share capital of Red Co, with directors of Indigo Co having four
of the six places on the board of Red Co

A. 1 and 2
B. 2 only
C. 1 and 3 only
D. 2 and 3 only

6Ref: “Preparing Simple Consolidated Financial Statements” by J. Lucas, F3/FFA examiner, April 2012,
http://www.accaglobal.com/content/dam/acca/global/PDF-students/2012s/sa_apr12_f3ffa_finstats.pdf
___________________________________________________________________________

406
__________________________

KEY NOTES
_________________

___________________________________________________________________________

407
1. An associate is an entity over which the investor has significant influence but not
control or joint control and that is neither a subsidiary nor an interest in joint
venture.

2. Significant influence is the power to participate in the financial and operating


policy decisions of the investee but is not in control or joint control over those
policies.

3. There are several indicators of significant influence, but the most important are
usually considered to be a holding of between 20% and 50% of the voting shares
and board representation.

4. Equity accounting brings an associate investment into the parent company’s


financial statements initially at cost.

5. P Co should take account of its share of the earnings of A Co whether or not A


Co distributes the earnings as dividends.

6. A’s sales revenue, cost of sales, expenses and revenue are not added with
those of the group.

7. In the consolidated statement of financial position, the associate is included as a


non-current asset investment, calculated as:

$’000
Cost of investment X
P’s share of post acquisition profits of A X
Less: impairment losses of A (X)
X

___________________________________________________________________________

408
__________________________

QUESTION BANK
_________________

___________________________________________________________________________

409
1. Define an ‘associate’ relationship and give some examples that might
demonstrate such a relationship exists.

2. Which of the following statements regarding the method of consolidation is true?


(1) Subsidiaries are consolidated in full
(2) Associates are equity accounted

A. Neither statement
B. Statement 1 only
C. Both statements
D. Statement 2 only

3. Bingo is an associate of Tingo.

How should profits generated by Bingo be shown in the consolidated accounts of


Tingo?

A. All the profits after tax generated by Bingo are included in the consolidated
statement of profit or loss of Tingo as a single amount
B. All the profits after tax generated by Bingo are included by consolidating the
revenue and expenses of Bingo on a line by line basis from revenue down to
profit for the year
C. Tingo’s share of Bingo’s profit after tax is included by the payment of a
dividend from Bingo to Tingo, which is shown in the consolidated statement of
profit or loss of Tingo
D. Tingo’s share of Bingo’s profit after tax is included in the consolidated
statement of profit or loss of Tingo as a single amount

___________________________________________________________________________

410
__________________________

ANSWER BANK
_________________

___________________________________________________________________________

411
1. An associate is defined as an entity in which an investor has significant influence
and which is neither a subsidiary nor a joint venture of the investor. Significant
influence can be determined by the holding of voting rights (usually shares) in
the entity. If an investor holds 20% to 50% of the voting power of the investee,
then the investor will usually have significant influence over the investee, unless
it can be clearly demonstrated this is not the case.

The following are examples that might demonstrate the existence of significant
influence:
(a) A representative of the investor on the board of directors of the
investee.
(b) The participation by the investor in the policy making process of the
investee.
(c) Material transactions between investee and investor.
(d) The interchange of management personnel between the two
companies.
(e) The provision of essential technical information by the investor to the
investee.

2. C

3. D

___________________________________________________________________________

412
___________________________________________________________________________

413
Chapter 27
INTERPRETATION OF FINANCIAL
STATEMENTS

27.1 ACCA SYLLABUS GUIDE OUTCOME 1:


Describe how the interpretation and analysis of financial statements is used in
a business environment.
Explain the purpose of interpretation of ratios

The financial statements of a business provide important financial information for


people outside the business (external users) who do not have access to the internal
accounts. For example, current and potential shareholders can see how much profit
a business made, the value of its assets and the level of cash reserves. Although
these figures are useful, they do not mean a great deal by themselves. To
summarise and present financial information in a more understandable form, they
need to be properly analysed using accounting ratios and then compared with either
the previous year’s ratios or against averages for the industry.

The lack of detailed information available to the external user is a considerable


disadvantage in undertaking ratio analysis. There may simply be insufficient data to
calculate all of the required ratios. Comparisons with previous year’s ratios can be
difficult especially if there have been changes in accounting policies or in the nature
of the business. Comparability between companies may be impaired due to different
accounting policies and different environments in which the two companies are
operating.

Lecture Example 1:

Which of the following statements is true?

A. The interpretation of an entity’s financial statements using ratios is only useful


for potential investors.

B. Ratios based on historical data can predict the future performance of an


entity.

C. The analysis of financial statements using ratios provides useful information


when compared with previous performance or industry averages.

D. An entity’s management will not assess an entity’s performance using


financial ratios.

___________________________________________________________________________

414
27.2 ACCA SYLLABUS GUIDE OUTCOME 2:
Calculate key accounting ratios: -
1. Profitability

2. Liquidity

3. Efficiency

4. Position

Explain the interrelationships between ratios.

Calculate and interpret the relationship between the elements of the financial
statements with regard to profitability, liquidity, efficient use of resources and
financial position.

Draw valid conclusions from the information contained within the financial
statements and present these to the appropriate user of the financial
statements.

Explain the interrelationships between ratios

Calculate and interpret the relationship between the elements of the financial
statements with regard to profitability, liquidity, efficient use of resources and
financial position

Draw valid conclusions from the information contained within the financial
statements and present these to the appropriate user of the financial
statements

27.2.1 Profitability Ratios

27.2.1.1 Return on Capital Employed (ROCE)

A business buys assets such as trucks, computers, etc to help makes its operations
more efficient, cut down on costs and make bigger profits.

ROCE shows how well a business has generated profit from its long-term financing.

It is expressed in the form of a percentage, and the higher the percentage, the
better.

___________________________________________________________________________

415
ROCE is calculated either:

Profit Before Interest and Tax


Total Assets – Current Liabilities (Capital Employed)

OR

Profit before Interest and Tax


Shareholder’s Equity + long-term liabilities

How can firms increase the ROCE ratio?

Movements in return on capital employed are best interpreted by examining profit


margins and asset turnover (in more detail below) as ROCE is made up of these
component parts.

Firms can increase their ROCE ratio by:


(a) Cutting costs so as to increase the profit margin ratio
(b) Increasing the revenue made from their assets, i.e. more efficient use of
assets

Limitations of using ROCE ratio

Be careful when using the ROCE ratio because it does not always yield the correct
percentage.

For instance, a company may simply run down its old assets. This means the
denominator “Total Assets – Current Liabilities” (value of assets is lower) will be
lower and so give a higher ROCE percentage.

In this case, there has been no improvement in operations of the company, in fact
the firm is cutting down on potentially profitable capital investments.

Note

Always compare a company’s ROCE to the interest rate it is charged. The ROCE
needs to be higher.

Similarly if a company pays off a 5% loan, while its current ROCE is 10%, then this is
illogical. It should use the money to get 10% not pay off a loan which only costs 5%.

___________________________________________________________________________

416
27.2.1.2 Asset Turnover

Asset turnover shows how efficiently management have utilised assets to generate
revenue.

It is calculated as: -
__________Revenue_________
Total assets – current liabilities

When looking at the components of the ratio, a change will be linked to either a
movement in revenue, a movement in net assets, or both.

An increase in asset turnover can result from: -


(a) a significant increase in sales revenue
(b) the business entering into a sale and operating lease agreement, then the
asset base would become smaller, thus improving the result.

27.2.1.3 Return on Equity (ROE)

The ROE ratio reveals how much profit has been made in comparison to shareholder
equity.

A business that has a high return on equity is more likely to be one that is capable of
generating cash internally.

Profit after tax – preference dividends


Equity shareholders funds

27.2.1.4 Gross Profit Margin

The gross profit margin looks at the performance of the business at the direct trading
level.

Gross profit
Revenue

Variations in the Gross Profit Margin are as a result of:


(i) changes in the selling price/sales volume
(ii) changes in cost of sales.

___________________________________________________________________________

417
For example, cost of sales may include inventory write downs that may have
occurred during the period due to damage or obsolescence, exchange rate
fluctuations or import duties.

27.2.1.5 Net Profit Margin

The net profit margin is generally calculated by comparing the profit before interest
and tax of a business to revenue.

Profit before interest and tax


Revenue

However, the examiner may specifically request the calculation to include profit
before tax.

Analysing the net profit margin enables you to determine how well the business has
managed to control its indirect costs during the period. In the exam, when
interpreting operating profit margin, it is advisable to link the result back to the gross
profit margin.

For example, if gross profit margin deteriorated in the year then it would be expected
that the net profit margin would also fall. However, if this is not the case, or the fall is
not so severe, it may be due to good indirect cost control or perhaps there could be a
one-off profit on disposal distorting the operating profit figure.

It is important to note that the profit margin and asset turnover together explain the
ROCE.

Profit Margin x Asset Turnover = ROCE

PBIT x Sales = PBIT


Sales Capital Employed Capital Employed

Lecture Example 2

Comparator assembles computer equipment from bought-in components and


distributes them to various wholesalers and retailers. It has recently subscribed to an
inter-firm comparison service. Members submit accounting ratios as specified by the
operator of the service, and in return, members receive the average figures for each
of the specified ratios taken from all of the companies in the same sector that
subscribe to the service.
___________________________________________________________________________

418
The specified ratios and the average figures for Comparator’s sector are shown
below.

Ratios of companies reporting a full year’s results for periods ending between
1 July 2003 and 30 September 2003:

Return on capital employed 22.1%


Net assets turnover 1.8 times
Gross profit margin 30%
Net profit (before tax) margin 12.5%
Current ratio 1.6:1
Quick ratio 0.9:1
Inventory holding period 46 days
Accounts receivable collection period 45 days
Accounts payable payment period 55 days
Debt to equity 40%
Interest cover 7 times
Dividend yield 6%
Dividend cover 3 times

Comparator’s financial statements for the year to 30 September 2003 are set out
below:

Statement of profit or loss $000


Sales revenue 2,305
Cost of sales (1,870)
______
Gross profit 435
Other operating expenses (215)
______
Operating profit 220
Interest payable (34)
______
Profit before taxation 186
Income tax (90)
______
Profit after taxation 96
______

___________________________________________________________________________

419
Extracts of changes in equity:
Retained earnings – 1 October 2002 179
Net profit for the period 96
Dividends paid (interim $60,000; final $30,000) (90)
______
Retained earnings – 30 September 2003 185
______

Statement of Financial Position $000 $000


Non-current assets (note (i)) 540

Current Assets
Inventory 275
Accounts receivable 320
Bank nil 595
_____
_____
1,135
_____
Share Capital and Reserves
Ordinary shares (25 cents each) 150
Retained Earnings 185
_____
335

Non-current liabilities
8% loan notes 300

Current liabilities
Bank overdraft 65
Trade accounts payable 350
Taxation 85 500
_____ ______
1,135
______

Notes

1) The details of the non-current assets are:

Cost Accum depn Net book value


$000 $000 $000
At 30 September 2003 3,600 3,060 540
___________________________________________________________________________

420
2) The market price of Comparator’s shares throughout the year averaged $6.00
each.

Required:-

a. Calculate the profitability ratios for Comparator equivalent to those


provided by the inter-firm comparison service.
b. Assess the performance of Comparator based on the ratios calculated in
(1) above.

(December 2003 ACCA Paper 2.5 adjusted)

27.2.2 Liquidity Ratios

27.2.2.1 Current Ratio

___Current Assets___
Current Liabilities

The current ratio considers how well a business can cover the current liabilities with
its current assets. It is a common belief that the ideal for this ratio is between 1.5 and
2 : 1 so that a business may comfortably cover its current liabilities should they fall
due.

However this ideal should be considered in the context of the company: the nature of
the assets in question, the company’s ability to borrow further to meet liabilities and
the stability of its cash flows.

For example, a business in the service industry would have little or no inventory and
therefore could have a current ratio of less than 1. This does not necessarily mean
that it has liquidity problems so it is better to compare the result to previous years or
industry averages.

___________________________________________________________________________

421
27.2.2.2 Quick Ratio

Current Assets – Inventories


Current Liabilities

One of the problems with the current assets ratio is that the assets counted include
inventories which may or may not be quickly sellable (or which may only be sellable
quickly at a lower price).

The ideal ratio is thought to be 1:1, but as with the current ratio, this will vary
depending on the industry in which the business operates.

The quick ratio is also known as the acid test ratio. This name is used because it is
the most demanding of the commonly used tests of short term financial stability.
When assessing both the current and the quick ratios, remember that both of these
ratios can be too high. This would mean too much cash is being tied up in current
assets as opposed to new more profitable investments.

It is important to look at the information provided within the question to consider


whether or not the company has an overdraft at year-end. The overdraft is an
additional factor indicating potential liquidity problems and this form of finance is both
expensive (higher rates of interest) and risky (repayable on demand)

Lecture Example 3

Following from Lecture Example 1:-

a. Calculate the liquidity ratios for Comparator equivalent to those


provided by the inter-firm comparison service.

b. Assess the liquidity position of Comparator based on the ratios


calculated in (1) above.

27.2.3 Efficiency Ratios: control of receivables and inventory

27.2.3.1 Inventory Turnover Period

Closing (or average) Inventory x 365


COS

___________________________________________________________________________

422
This ratio calculates how long goods to be sold stay in stock.

Generally, the lower the number of days that inventory is held the better as holding
inventory for long periods of time constrains cash flow and increases the risk
associated with holding the inventory. The longer inventory is held the greater the
risk that it could be subject to theft, damage or obsolescence. However, a business
should always ensure that there is sufficient inventory to meet the demand of its
customers.

27.2.3.2 Receivables Collection Period (in days)

Trade Receivables x 365


Credit Sales

This ratio calculates how long credit customers take to pay.

A short credit period for receivables will aid a business’ cash flow. However, some
businesses base their strategy on long credit periods to achieve higher sales in
highly competitive markets.

If the receivables days are shorter compared to the prior period, it could indicate
better credit control or potential settlement discounts being offered to collect cash
more quickly whereas an increase in credit periods could indicate a deterioration in
credit control or potential bad debts.

27.2.3.3 Payables Payment Period (in days)

__Trade Payables__ x 365


Credit Purchases7

This ratio calculates how long the company takes to pay its suppliers.

An increase in payables days could indicate that a business is having cash flow
difficulties and is therefore delaying payments. It is important that a business pays
within the agreed credit period to avoid conflict with suppliers.

If the payables days are reducing, this indicates suppliers are being paid more
quickly. This could be due to credit terms being tightened or taking advantage of
early settlement discounts being offered.

7 Take cost of sales if credit purchases are not given


___________________________________________________________________________

423
27.2.3.4 Working Capital Cycle (cash cycle)

A company only gets cash once an item has been in stock and then the debtor pays
(Inventory days + receivables days).

This total should then be reduced by the payable days (the company doesn’t need
the cash until the end of this).

So, the working capital cycle (in days) is:


Inventory (in days) + Receivables (in days) – Payables (in days)

This needs to be kept as small as possible for liquidity purposes.

Lecture Example 4

Following from Lecture Example 1:-

a. Calculate the efficiency ratios for Comparator equivalent to those


provided by the inter-firm comparison service.

b. Assess the performance of Comparator based on the ratios calculated in


(1) above.

27.2.4 Long-term Solvency: Debt and Gearing Ratios

27.2.4.1 Debt Ratio

Debt ratio = Total debts


Total assets

Assets = non-current assets + current assets


Debts include all payables, whether they are due within one year or after more than
one year.

___________________________________________________________________________

424
27.2.4.2 Gearing

A company can raise money by loans (Debt) or issuing shares (Equity).

Gearing can be calculated either:

_____Debt8 ______
Debt + Equity9

OR

___Debt___
Equity

The gearing ratio is of particular importance to a business as it indicates how risky a


business is perceived to be based on its level of borrowing.

High gearing means high debt (in relation to equity). As borrowing increases so does
the risk as the business is now liable to not only repay the debt but meet any interest
commitments under it. If interest rates increase, then the company could be in
trouble unless they have high enough profits to cover this. In addition, to raise further
debt finance could potentially be more difficult and more expensive.

27.2.4.3 Leverage (equity to sales ratio)

Leverage is the converse of gearing, i.e. the proportion of total assets financed by
equity.

Shareholder’s equity x 100


Shareholders’ equity + total long term debt

OR

Shareholder’s equity x 100


Total assets less current liabilities

8 Debt = Loans + Preference Shares


9 Equity = Ordinary share capital + Reserves + Non-controlling interest
___________________________________________________________________________

425
27.2.4.4 Interest Cover

If a company has a high level of gearing it does not necessarily mean that it will face
difficulties as a result of this.

For example, if the business has a high level of security in the form of tangible non-
current assets and can comfortably cover its interest payments, a high level of
gearing should not give an investor cause for concern.

The interest cover is calculated:

Profit before Interest and Tax (PBIT)


Interest payable

A ratio of at least 3 is deemed to be satisfactory.


The interest coverage ratio is a measurement of the number of times a company
could make its interest payments with its earnings.

It is the equivalent of a person taking the combined interest expense from their
mortgage, credit cards etc, and calculating the number of times they can pay it with
their annual income.

PBIT has its short fallings; companies do pay taxes, therefore it is misleading to act
as if they didn’t. A wise and conservative investor would simply take the company’s
earnings before interest and divide it by the interest expense. This would provide a
more accurate picture of safety.

Lecture Example 5

Following from Lecture Example 1:-

a. Calculate the gearing ratios for Comparator equivalent to those provided


by the inter-firm comparison service.

b. Assess the performance of Comparator based on the ratios calculated in


(1) above.

___________________________________________________________________________

426
Lecture Example 6

Which two of the following are valid reasons why the inventory turnover of a
company increases from one year to the next?

1. A slow down in trading


2. A marketing decision to reduce selling prices
3. Seasonal fluctuations in orders
4. Obsolete goods

A. 1 and 2
B. 2 and 3
C. 1 and 4
D. 3 and 4

Lecture Example 7

A company has increased the length of time allowed for customers to pay their
invoices. This has resulted in an increase in which ratio?

A. Receivables collection period


B. Gearing ratio
C. Interest cover
D. Payables payment period

___________________________________________________________________________

427
__________________________

KEY NOTES
_________________

___________________________________________________________________________

428
1. To summarise and present financial information in a more understandable
form, users need to be properly analysed using accounting ratios and then
compared with either the previous year’s ratios or against averages for the
industry.

2. Ratio analysis has a number of limitations.

3. ROCE is calculated either:

Profit Before Interest and Tax


Total Assets – Current Liabilities (Capital Employed)

OR

Profit before Interest and Tax


Shareholder’s Equity + long-term liabilities

4. Asset turnover =

__________Revenue_________
Total assets – current liabilities

5. Return on equity =

Profit after tax – preference dividends


Equity shareholders funds

6. Gross Profit Margin =

Gross profit
Revenue

7. Net Profit Margin =

Profit before interest and tax


Revenue

___________________________________________________________________________

429
8. ROCE (alternative method) =

Profit Margin x Asset Turnover = ROCE

PBIT x Sales = PBIT


Sales Capital Employed Capital Employed

9. Current Ratio =

___Current Assets___
Current Liabilities

10. Quick Ratio =

Current Assets – Inventories


Current Liabilities

11. Inventory Turnover Period =

__Closing (or average) Inventory_ x 365


COS

12. Receivables Collection Period (in days) =

Trade Receivables x 365


Credit Sales

13. Payables Payment Period (in days) =

__Trade Payables__ x 365


Credit Purchases10

10 Take cost of sales if credit purchases are not given


___________________________________________________________________________

430
14. Working capital cycle (in days) is:

Inventory (in days) + Receivables (in days) – Payables (in days)

15. Debt Ratio =

Total debts
Total assets

16. Gearing =

_____Debt11 ______
Debt + Equity12

OR

___Debt___
Equity

17. Leverage (equity to sales ratio) =

.
Shareholder’s equity x 100
Shareholders’ equity + total long term debt

OR

Shareholder’s equity x 100


Total assets less current liabilities

18. Interest Cover =

Profit before Interest and Tax (PBIT)


Interest payable

11 Debt = Loans + Preference Shares


12 Equity = Ordinary share capital + Reserves + Non-controlling interest
___________________________________________________________________________

431
__________________________

QUESTION BANK
_________________

___________________________________________________________________________

432
1. Xena has the following working capital ratios:

20X9 20X8

Current ratio 1·2:1 1·5:1


Receivables days 75 days 50 days
Payables days 30 days 45 days
Inventory turnover 42 days 35 days

Which of the following statements is correct?

A. Xena’s liquidity and working capital has improved in 20X9


B. Xena is receiving cash from customers more quickly in 20X9 than in 20X8
C. Xena is suffering from a worsening liquidity position in 20X9
D. Xena is taking longer to pay suppliers in 20X9 than in 20X8

2. The following extracts are from Hassan’s financial statements:

$
Profit before interest and tax 10,200
Interest (1,600)
Tax (3,300)
–––––––
Profit after tax 5,300
–––––––
Share capital 20,000
Reserves 15,600
–––––––
35,600
Loan liability 6,900
–––––––
42,500
–––––––

What is Hassan’s return on capital employed?

A. 15%
B. 29%
C. 24%
D. 12%

___________________________________________________________________________

433
3. A company’s gross profit as a percentage of sales increased from 24% in the
year ended 31 December 20X1 to 27% in the year ended 31 December 20X2.

Which of the following events is most likely to have caused the increase?

A. An increase in sales volume


B. A purchase in December 20X1 mistakenly being recorded as happening in
January 20X2
C. Overstatement of the closing inventory at 31 December 20X1
D. Understatement of the closing inventory at 31 December 20X1

4. Which one of the following would cause a company’s gross profit


percentage on sales to fall?

A. A reduction in the total value of goods returned to suppliers.


B. An increase in the costs of delivery of goods to customers.
C. A decline in average inventory levels.
D. An increase in theft of inventory by customers and staff

5. A company’s gross profit as a percentage of sales increased from 28% in the


year ended 31 December 2005 to 33% in the year ended 31 December 2006.

Which one of the following could have caused the increases?

A. An increase in sales volume.


B. Understatement of closing inventory at 31 December 2005.
C. Overstatement of closing inventory at 31 December 2005.
D. Goods received in December 2005 and included in inventory at 31 December
2005 were not recorded in purchases until January 2006.

6. Which of the following events would reduce a company’s gearing?

(i) An issue of loan notes


(ii) A rights issue of equity shares
(iii) A bonus issue of equity shares

A. (i) and (ii)


B. (i) and (iii)
C. (iii) only
D. (ii) only

___________________________________________________________________________

434
__________________________

ANSWER BANK
_________________

___________________________________________________________________________

435
1. C – Liquidity has worsened – current ratio

Working capital has improved – from 40 (50 + 35 – 45) to 87 (75 + 42 -


30)

2. C – (10,200 / 42,500)

3. D – Which one would cause either Gross Profit to increase or Sales to fall?
Lower Closing Inventories in 20X1 would show as lower Opening
Inventories in X2, lower Cost of Sales, higher Gross Profit.

4. C – Lower Inventories would increase Cost of Sales and decrease Gross


Profit.

5. B – (Similar to Question 3).

6. D – An issue of loan notes increase debt (gearing); a bonus issue has no


effect on equity (higher SC, lower SP).

___________________________________________________________________________

436

You might also like